Download as pdf or txt
Download as pdf or txt
You are on page 1of 132

1 Chapter 1 | Fundamentals of Mathematics

Chapter
Quadratic Equations
2
LECTURE-1 SOLVING POLYNOMIAL EQUATIONS

Introduction Step-1 Factorise the constant term of the


A quadratic equation in the variable x is an equation of given quadratic equation.
the form ax2 + bx + c = 0, where a, b, care real Step-2 Express the coefficient of middle term
numbers, a  0. as the sum or difference of the factors
e.g., 2x2 – 3x + 1 = 0, 4x – 3x2 = 0, 1 – x2 = 0 etc. obtained in step 1. Clearly, the product of
When we write the terms of a quadratic equation in these two factors will be equal to the product
descending order of their degrees, then we get the of the coefficient of x2 and constant term.
standard form of the equation. i.e. ax2 + bx + c = 0, Step-3 Split the middle term in two parts
where a, b, c are real numbers and a  0 is called the obtained in step 2.
standard form of a quadratic equation. Step-4 Factorise the quadratic equation
A quadratic equation can be of the following types : obtained in step 3.
(i) b = 0, c  0 i.e. of the type ax2 + c = 0
(ii) b  0, c = 0 i.e. of the type ax2 + bx = 0 (ii) By Completing the Square Method
(iii) b = 0, c = 0 i.e. of the type ax2 = 0 ALGORITHM
(iv) b  0, c  0 i.e. of the type ax2 + bx + c = 0 Step-1 Obtain the quadratic equation. Let the
quadratic equation be ax2 + bx + c = 0, a  0.
Step-2 Make the coefficient of x2 unity, if it is
Methods of Solving b c
not unity. i.e., obtain x 2 + x + = 0.
(a) Roots of a Quadratic Equation a a
c
A real number  is a root of quadratic equation ax2 Step-3 Shift the constant term on R.H.S. to
a
b c
+ bx + c = 0, a  0 ifa2 + b + c = 0. So any real 2
get x + x = −
a a
number which satisfies a given quadratic equation Step-4 Add square of half of the coefficient of
is called the root of the quadratic equation. b 2
In polynomial chapter, we learnt that if x =  x. i.e. ( ) on both sides to obtain
2a
satisfies a quadratic polynomial p(x) i.e., p() = 0, b b 2 b 2 c
x2 + 2 ( ) x + ( ) = ( ) −
then  is the zero of p(x). 2a 2a 2a a

This means that zeros of a quadratic polynomial Step-5 Write L.H.S. as the perfect square of a
ax2 + bx + c and roots of a quadratic equation ax2 binomial expression and simplify R.H.S. to get
b 2 b2 −4ac
+ bx + c = 0 are the same. A quadratic equation (x + 2a) = 4a2 .
can have almost two real roots. Step-6 Take square root of both sides to get
b b2 −4ac
x+ = ±√ .
Note 2a 4a2
1. A quadratic equation is satisfied by exactly two Step-7 Obtain the values of x by shifting the
values of 'x' which may be real or imaginary. b
constant term on RHS.
The equation, ax2 + bx + c = 0 is 2a
(iii) By Using Quadratic Formula
2. A quadratic equation if a  0. [Two roots]
Consider the quadratic equation : ax2 + bx +
3. A linear equation if a = 0, b  0. [One root]
c = 0. a  0
4. A contradiction if a = b = 0, c  0. [No root]
Dividing throughout by a, we get
5. An identity if a = b = c = 0. [Infinite roots] b c b c
6. A quadratic equation cannot have more than x2 + x + = 0 ⇒ x2 + x = −
a a a a
two roots. 2 b b 2 b 2 c
⇒ x + 2( )x + ( ) = ( ) −
7. It follows from the above statement that if a 2a 2a 2a a
b 2 b2 −4ac
quadratic equation is satisfied by more than ⇒ (x + ) =
2a 4a2
two values of x, then it is satisfied by every √b2 −4ac
b
value of x and so it is an identity. ⇒x+ =± , when b2 – 4ac  0
2a 2a
b √b2 −4ac −b±√b2 −4ac
⇒x=− ± ⇒x=
2a 2a 2a
(b) Methods of Solving Quadratic Equation This is known as Quadratic formula or
(i) By Factorization Method Shreedharacharay’s formula for finding the
ALGORITHM roots of a quadratic equation.
Chapter 1 | Fundamentals of Mathematics 2
Hence, if – 4ac  0, then the root of the
b2 1
Solution : Putting x = 5 and x = in the given equation.
2
quadratic equation ax2 + bx + c are given by 2 5 2 5
−b+√b2 −4ac −b−√b2 −4ac
− + 2 and 1 2
− 1 +2
(5)2 5 (2) (2)
and
2a 2a 2 2 5
Discriminant : For the quadratic equation ax2  − 1 + 2 and 1 − 1 +2
25
4 2
+ bx + c = 0, the expression D = (b2 – 4ac) is 2 27
called its discriminant. In terms of  +1 and 8 – 10 + 2  and 0
25 25
1
discriminant D, the two roots are given by i.e. x = 5 does not satisfy but x = satisfies the given
2
−b−√D −b−√D
α= and β = equation.
2a 2a 1
Hence, x = 5 is not a solution but x = is a solution of
Remark : If the discriminate D = b2 – 4ac < 0, then the 2 5
2

quadratic equation ax2 + bx + c = 0 has no real roots. 𝑥 2 − + 2 = 0.


𝑥

Example 1.1 Example 1.4


Which of the following are quadratic equations ? Find the value of K for which the given value is a
(i) x2 – 6√x + 2 = 0 solution of the equation.
1 (i) x2 + Kx + 3 = 0, x = 1
(ii) x + = 1
x (ii) x2 – x(a + b) + K = 0, x = a
Solution : (i) x2 – 6√x + 2 = 0 is not a quadratic Solution : (i) Since x = 1 is a solution of the equation
equation as x2 – 6√x + 2 is not a quadratic polynomial x2 + Kx + 3 = 0
( the power of x is 1/2)  (1)2 + K × 1 + 3 = 0
1  1+K+3=0
(ii) x + = 1
x  K = – 4.
x2 +1
⇒ =1 (ii) Since x = a is a solution of equation
x
 x2 + 1 = x x2 – x(a + b) + K = 0
 x2 – x + 1 = 0  (a)2 – a(a + b) + K = 0
 (x2 – x + 1) is a quadratic polynomial  a2 – a2 – ab + K = 0
 (x2 – x + 1) = 0 is a quadratic equation.  – ab + K = 0
 K = ab.
Example 1.2
Example 1.5
Determine whether the given values are solution of 2
the given equation or not. If x = and x = – 3 are the roots of the equation ax2 +
3
1
(i) 3x2 + 2x – 1 = 0; x = 7x + b = 0, find the value of a and b.
3 2
(ii) a2x2 – 3abx + 2b2 = 0; x = , x =
a b Solution : Since x = is a root of the equation
3
b a
Solution : (i) 3x2 + 2x – 1 = 0 ax2 + 7x + b = 0
1 2 2 2
Putting x = in the LHS of the given equation, we get ∴ a( ) + 7 × + b = 0
3 3 3
1 2 1  4a + 9b = – 42 ... (i)
LHS = 3x2 + 2x – 1 = 3 × ( ) + 2 × − 1
3 3 Again, x = – 3 is a root of the equation ax2 + 7x + b = 0
1 2 1 2 1+2−3
=3× + −1= + −1=
9 3 3 3 3
= 0 = RHS  a(–3)2 + 7(–3) + b = 0
1
 x = is a solution of + 2x – 1 = 0. 3x2  9a – 21 + b = 0
3
 9a + b = 21 ... (ii)
(ii) a x – 3abx + 2b = 0
2 2 2
On solving (i) & (ii) a = 3, b = – 6
We have a2x2 – 3abx + 2b2 = 0
a
Putting x = in the LHS of the given equation.
b
a2 a a4
Example 1.6
LHS = a2 × 2 − 3ab × + 2b2 = 2 − 3a2 + 2b2 
b b b Solve the following quadratic equation by
RHS factorization method :
b
Putting x = in the LHS of the given equation x2 – 2ax + a2 – b2 = 0
a
b2 b Solution : Factors of the constant term a2 – b2 are (a
LHS = a2 × 2 − 3ab × + 2b2 = b2 – 3b2 + 2b2 = – b) & (a + b). Also, coefficient of the middle term
a a
0 = RHS = – 2a = {(a – b) + (a + b)}.
a b
hence x = is not the solution, but x = is a  x2 – 2ax + a2 – b2 = 0
b a
solution of the given equation.  x2 – {(a –b) + (a + b)} x + (a + b) (a – b) = 0
 x2 – (a – b) x – (a + b) x + (a – b) (a + b) = 0
Example 1.3  x{x– (a – b)} – (a + b) {x – (a – b)} = 0
 {x – (a – b)} {x– (a + b)} = 0
Determine whether the given values are the solutions x – (a – b) = 0 or x – (a + b) = 0
of the given equation or not : x = a – b or x = a + b
2 5 1
𝑥2 − + 2 = 0; x = 5 x = –
𝑥 2
OR
3 Chapter 1 | Fundamentals of Mathematics
(x2 – 2ax + a2) – b2 = 0
 (x – a)2 – (b)2 = 0 Example 1.12
 (x – a + b) (x – a – b) = 0
By using the method of completing the square, show
 x – (a – b) = 0 or x – (a – b) = 0
that the equation 4x2 + 3x + 5 = 0 has no real roots.
 x = a – b or a + b
Solution : We have, 4x2 + 3x + 5 = 0 ...(i)
divide (i) by 4 we get
Example 1.7 3
 x2 + x = −
5
4 4
Find the solutions of the quadratic equation x2 + 6x + add square of half of the coefficient of x on both sides
5 = 0. 3 3 2 3 2 5
Solution : The quadratic polynomial x2 + 6x + 5 can be  x2 + x + ( ) = ( ) −
4 8 8 4
factorized as follows 3 2 9 5
 (x + ) = −
x2 + 6x + 5 8 64 4
3 2 71
= x2 + 5x + x + 5  (x + ) = −
8 64
= x(x + 5) + 1(x + 5) Clearly, RHS is negative.
= (x + 5)(x + 1) 3 2
Therefore the given quadratic equation becomes (x + But, (x + ) cannot be negative for any real value of x.
8
5)(x + 1) = 0 Hence, the given equation has no real roots.
This gives x = – 5 or x = –1.
Therefore, x = – 1, – 5 are the required solutions of the Example 1.13
given equation.
Solve : 9x2 – 15x + 6 = 0
Solution : Here, 9x2 – 15x + 6 = 0
Example 1.8 15 6
 x 2 − x + = 0 (Dividing throughout by 9)
Solve : 25x2 – 30x + 9 = 0 9 9
5 2
Solution : 25x2 – 30x + 9 = 0  x2 − x + = 0
3 3
(5x)2 – 2(5x) × 3 + (3)2 = 0 5 2
 x2 − x = −
3 3
(5x – 3)2 = 0
3 3 3 (Shifting the constant term on RHS)
This gives x = , or simply x = as the required 5 −5 2 −5 2 2
5 5 5
 x2 − 2 ( ) x + ( ) = ( ) −
solution. 6 6 6 3
(Adding square of half of coefficient of x on both sides)
5 2 25 2
Example 1.9  (x − ) = −
6 36 3
Solve 64x2 – 625 = 0 5 2 25−24
 (x − ) =
6 36
Solution : We have 64x2 – 625 = 0
5 2 1
or (8x)2 – (25)2 = 0  (x − ) =
6 36
or (8x + 25)(8x – 25) = 0 5 1
 x− =± [Taking square root of both sides]
i.e., 8x + 25 = 0 or 8x – 25 = 0. 6
5 1
6
25
This gives x = − or .
25  x= ±
8 8 6 6
5 1 5 1 4 2
Thus, x = −
25 25
, are solutions of the given equation.  x = + =1 or x = − = =
6 6 6 6 6 3
8 8

Example 1.10 Example 1.14


Solve the quadratic equation : 16x2 – 24x = 0. Solve the quadratic equation x2 – 6x + 4 = 0 by using
Solution : The given equation may be written as 8x(2x quadratic formula (Sridharacharya's Rule).
– 3) = 0 Solution : Solve the quadratic equation x2 – 6x + 4 = 0
3 by using quadratic formula (Sridharacharya's Rule).
This gives x = 0 or x = .
3
2 On comparing the given equation x2 – 6x + 4 = 0 with
x = 0, are the required solutions. the standard form of quadratic equation ax2 + bx + c
2
= 0, we get a = 1, b = – 6, c = 4.
Hence the required roots are
Example 1.11
−(−6) ± √(−6)2 − 4(1)(4) 6 ± √36 − 16
Solve: x2 + 6x + 8 = 0 x= =
2(1) 2
Solution : We have
x2 + 6x + 8 = 0 6 ± √20 6 ± √4 × 5
= =
x2 + 6x = – 8 2 2
2(3±√5)
add square of half of the coefficient of x on both sides = = 3 ± √5
2
x2 + 6x + (3)2 = – 8 + (3)2
Thus, the roots of the equation are 3 + √5and 3 – √5.
(x + 3)2 = – 8 + 9
(x + 3)2 = 1
x+3=1 Example 1.15
x + 3 = 1 or x + 3 = – 1 Solve the quadratic equation x2 – 7x – 5 = 0.
 x = – 2 or – 4 are the solutions.
Chapter 1 | Fundamentals of Mathematics 4
Solution : Comparing the given equation with + bx ax2 Example 1.18
+ c = 0, a = 1, b = –7 and c = –5.
Therefore, D = (–7)2 – 4 × 1 × (–5) = 49 + 20 = 69 > 0
Find the value of √2 + √2 + √2+. . . . . . . . . . . . . . . . . . . . ..
Since D is positive, the equation has two roots given by
−b+√D −b−√D 7+√69 7−√69 Solution : Let
x= , = ,
2a 2a 2 2
⇒x=
7+√69 7−√69
, are the required solutions. x = √2 + √2 + √2+. . . . . . . . . . . . . . . . . . . . . ∞
2 2
x = √2 + x
by squaring on both sides
Example 1.16
 x2 = 2 + x
Solve the quadratic equation 9x2 – 9(a + b)x + 2a2 +  x2 – x – 2 = 0  (x – 2) (x + 1) = 0
5ab + 2b2 = 0 .  x = –1, 2.
Solution : 9x2 – 9 (a + b) x + 2a2 + 5ab + 2b2 = 0
compare the given equation Ax2 + Bx + C = 0
Example 1.19
A=9
B = –9 (a + b) Solve for x : 2x4 + x3 – 6x2 + x + 2 = 0
C = 2a2 + 5ab + 2b2 Solution : 2x4 + x3 – 6x2 + x + 2 = 0
−±√𝐵2 −4𝐴𝐶 divide the equation by x2, we get
x= 1 1
2𝐴 2 (x 2 + 2) + (x + ) − 6 = 0 ...(i)
9(𝑎+𝑏)±√92 (𝑎 + 𝑏)2 −4×9(2 𝑎2 +5 𝑎𝑏 +2 𝑏2 ) x x
x= 1
x + = y then x 2 +
1
= y 2 − 2put these values in (i)
18
x x2
9(a+b)±√81 a2 +81 b2 +162ab−72 a2 −72 b2 −180ab
x=  2(y2 – 2) + y – 6 = 0
18
9(a+b)±√9 a2 +9 b2 −18ab
 2y2 – 4 + y – 6 = 0
x=  2y2 + y – 10 = 0
18

x=
9(a+b)±√9(a − b)2  2y2 + 5y – 4y – 10 = 0
18
9(a+b)±3(a − b)
 y(2y + 5) – 2(2y + 5) = 0
x=
18  (y – 2) (2y + 5) = 0
5
Taking positive value  y = 2, −
9a+9b+3 a −3b 2a+b 2
x= = 1
x+ 0y=2 or x + = y =
1 −5
18 3
x x 2
Taking negative value x2 +1 −5
2a+b a+2b  x2 – 2x + 1 = 0 =
x= , . x 2
3 3  (x – 1)2 = 0
 x = 1,1  2x2 + 2 + 5x = 0
Example 1.17  (x +2) (2x + 1) = 0
1
 x = – 2, − .
2
Solve for x : 9x + 2 – 6 × 3x + 1 + 1 = 0
Solution : We have 9x + 2 – 6 × 3x + 1 + 1 = 0
 9x.92 – 6.3x.3 + 1 = 0
Example 1.20
1
 81(3x)2 – 18 × 3x + 1 = 0 ... (i) Find the value of 2 + 1 .
2+
Let 3x = y, then equation (i) becomes 2+......∞
1
 81 y2 – 18y + 1 = 0 Solution : Let x = 2 +
2+
1

 81y2 – 9y – 9y + 1 = 0
2+......∞
1
 x = 2 +  x2 – 2x – 1 = 0
 9y(9y – 1) – 1(9y – 1) = 0 x
−(−2)±√(−2)2 −4(1)(−1)
 (9y – 1) (9y – 1) = 0  by quadratic formula x =
1 1 2(1)
 y = or y = ... (ii) 2±√8
9 9 =
Putting y = 3x in (ii), we get 2
1
3x = = 3–2  x = 1 +√2, 1 – √2
9
1 – √2 is neglected
x = – 2 and – 2.
so, x = 1 + √2.

WORKED-OUT PROBLEMS - 1

𝟐
1. The number of roots of the equation 𝒙 − =
(𝒙 −𝟏) 𝟏 𝟏
𝟐 3. Solve for 𝒙: 𝟐 (𝒙𝟐 + ) − 𝟗 (𝒙 + 𝒙) + 𝟏𝟒 = 𝟎.
𝟏− is : 𝒙𝟐
(𝒙 −𝟏) 𝟏 𝟏 𝟏 𝟏
(A) , 1, 2 (B) , 1, –2 (C) , 1, 4 (D) , 1, –4
(A) 0 (B) 1 (C) 2 (D) infinite 𝟐 𝟐 𝟐 𝟐

2. The set of all real values of p for which the equation 4. The roots of equation x2 + px + q = 0 are 1 and 2.
x + 1 = √𝒑𝒙 has exactly one root, is : The roots of the equation qx2 – px + 1 = 0 must be
(A) {0} (B) {4} (C) {0, 4} (D) {0, 2} :
5 Chapter 1 | Fundamentals of Mathematics
𝟏 𝟏 𝟏 𝟏
(A) –1, − (B) , 1 (C) − , 1 (D) – 1,
𝟐 𝟐 𝟐 𝟐
12. Solve for x : √2𝑥 + 9+ x = 13
5. If the roots of the equation px2 + qx + r = 0 are in (A) 4, 16 (B) 8 (C) 2 (D) 8, 20
the ratio : m then :
13. Solve for x : √2𝑥 + 9 − √𝑥 − 4 = 3
(A) ( + m)2 pq =  mr2 (A) 4, 16 (B) 8, 20
(B) ( + m)2 pr =  mq (C) 2, 8 (D) None of these

(C) ( + m)2 pr =  mq2 1 1


14. Solve for x : 2 [𝑥 2 + ] − 9 [𝑥 + ] + 14= 0
𝑥2 𝑥
(D) None of these 1 1
(A) ,1, 2 (B) 2, 4,
2 3
1
6. If 2 – √𝟑 is a root of the quadratic equation x2 + (C) , 4, 1 (D) None of these
3
2(√𝟑 −1) x + 3 – 2 √𝟑 = 0, then the second root is
: 15. The equation √𝑥 + 1 − √𝑥 − 1 = √4𝑥 − 1 has
(A) √𝟑 – 2 (B) √𝟑 (C) 2 + √𝟑 (D) – √𝟑 (A) No solution
(B) One solution
7. – 2 + √𝟑 is a solution to which of the following (C) Two solutions
equations ? (D) More than two solutions
(A) 3x2 + 12x + 3 = 0 (B) x2 + 2x + 3 = 0
(C) x2 + 4x + 2 = 0 (D) 3x2 + 2x – 1 = 0 16. The number of real roots of the equation (x – 1)2
+ (x – 2)2 + (x – 3)2 = 0
8. The value (s) of k for which the quadratic equation (A) 0 (B) 2 (C) 3 (D) 6
kx2 – kx + 1 = 0 has equal roots is :
(A) k = 0 only (B) k = 4 only 17. Solve for x : (x + 2) (x – 5) (x – 6) (x + 1) = 144
(C) k = 0, 4 (D) k = – 4 (A) –1, –2, –3 (B) 7, –3, 2
(C) 2, –3, 5 (D) None of these

√𝟔 + √𝟔 + √𝟔 + √𝟔+. . . . . . . = ? 18. The values of x which satisfy the expression (5 +


9. 2 2
2√6)𝑥 −3 + (5 − 2√6)𝑥 −3 =10
(A) 3 (B) 6 (C) 9 (D)  (A) ± 2, ± √3 (B) ±√2, ± 4
(C) ± 2, ±√2 (D) 2, √2, √3
10. If the roots x1 and x2, of the quadratic equation x2
– 2x + c = 0 also satisfy the equation 7x2 – 4x1 = 19. If x2 – (a + b) x + ab = 0, then the value of (x – a)2
47, then which of the following is true? + (x – b)2 is
(A) c = –15 (B) x1 = 5, x2 = 3 (A) a2 + b2 (B) (a + b)2 (C) (a – b)2 (D) a2 – b2
(C) x1 = 4.5, x2 = –2.5 (D) None of these

11. Solve for x : x6 – 26x3 – 27 = 0


(A) –1, 3 (B) 1, 3 (C) 1, –3 (D) –1, –3

Answer Key
WORKED-OUT PROBLEMS - 1

Ques. 1 2 3 4 5 6 7 8 9 10 11 12 13 14 15 16 17 18 19
Ans. A C A A C D A B A A A B B A A A B C C
11 Chapter 1 | Fundamentals of Mathematics

Chapter
Quadratic Equations
2
LECTURE-2 EQUATIONS REDUCIBLE
TO QUADRATIC EQUATION

A. IRRATIONAL OR SURD EQUATIONS WE shall now discuss some problems which give
DEFINITION : If the root of a quantity cannot be rise to quadratic equations.
exactly obtained the root is called a surd.
3 5
Thus √2, √5, √a3 , √a2 + b 2 are surds. Example 2.4
A surd is sometimes called an irrational quantity; A train travels 300 kilometres at a uniform speed; if
and quantities which are not surds are, for the the speed had been 5 kilometres per hour more, the
sake of distinction, termed rational quantities. journey would have taken two hours less: find the
speed of the train.
Sometimes equations are proposed in which the Solution : Suppose the train travels at the speed of x
unknown quantity appears under the radical sign. 300
For a fuller discussion of surd equations the kilometres per hour, then the time occupied is
x
student may consult the Elementary Algebra. hours.
300
Here we shall only consider a few simple cases, On the other supposition the time is hours;
x+5
which can generally be solved by the following 300 300
 = − 2;
method. Bring to one side of the equation a single x+5 x
radical term by itself: on squaring both sides this Hence x2 + 5 – 750 = 0, or (x + 30) (x – 25) = 0,
radical will disappear. By repeating this process  x = 25 or – 30.
any remaining radicals can in turn be removed. Hence the train travels 25 kilometres per hour, the
negative value being inadmissible.
Example 2.1
Example 2.5
Solve 2√x − √4x − 11 = 1.
A man buys a number of articles for Rs. 10, and sells all
Solution : Transposing, 2√x − 1 = √4x − 11
but two for Rs. 10.80 at 10 paise a piece more than
Square both sides; then
they cost; how many did he buy?
4x – 4√x + 1 = 4x – 11, Solution : Let x be the number of articles bought; then
4√x = 12, 1000
the cost price of each is paise, and the sale price is
√x = 3; 1080
x
 x = 9. x−2
paise.
1080 1000
 − = 10;
x−2 x
Example 2.2 That is,
108

100
= 1.
3 x−2 x
Solve 2 + √x − 5 = 13. After simplification, 8x + 20 = x2 – 2x,
3
Solution : Transposing, √x − 5 = 11. or x2 – 10x – 200 = 0;
Here we must cube both sides; thus x – 5 = 1331; that is, (x – 20) (x + 10) = 0;
whence x = 1336.  x = 20, or –10.
Thus the number required is 20.
Example 2.3
6√x−11 2√x+1 Example 2.6
Solve = 1
3√x √x+6
A cistern can be filled by two pipes in 33 minutes; if
Solution : Multiplying across, we have 3
(6√x − 11)(√x + 6) = 3√x(2√x + 1); the larger pipe takes 15 minutes less than the smaller
to fill the cistern, find in what time it will be filled by
that is, 6x − 11√x + 36√x − 66 = 6x + 3√x,
each pipe singly.
−11√x + 36√x − 3√x = 66,
Solution : Suppose that the two pipes running singly
22√x = 66, would fill the cistern in x and x – 15 minutes; then they
√x = 3; 1
will fill and
1
of the cistern respectively in one
 x = 9. x x−15
minute, and therefore when running together they will
1 1
PROBLEMS LEADING TO QUADRATIC EQUATIONS fill + of the cistern in one minute.
x x−15

By : Prashant Jain Sir


Chapter 1 | Fundamentals of Mathematics 12
1 3
But they fill 1 or of the cistern in one minute. Note
33 100
3
1 1 3 1. By a simple reduction many equations can be
Hence + =
x x−15 100 brought to the form in which the above
100(2x – 15) = 3x(x – 15), equation is given. When this is the case, the
3x2 – 245x + 1500 = 0, necessary simplification is readily completed
(x – 75) (3x – 20) = 0; by "multiplying up" or "multiplying across'', as
2
 x = 75, or 6 . it is sometimes called.
3
Thus the smaller pipe takes 75 minutes, the larger 60
2
minutes. The other solution 6 is inadmissible. Example 2.9
3
8x+23 5x+2 2x+3
Solve − = − 1.
20 3x+4 5
Sometimes it will be found convenient to use more Solution : Multiply by 20, and we have
20(5x+2)
than one unknown. 8x + 23 – = 8x + 12 – 20.
3x+4
20(5x+2)
By transposition, 31 = .
3x+4
Example 2.7 Multiplying across, 93x + 124 = 20(5x + 2).
Nine times the side of one square exceeds the 84 = 7x;
perimeter of a second square by one metre, and six  x = 12.
times the area of the second square exceeds twenty- When two or more fractions have the same denominator,
nine limes the area of the first by one square metre; they should be taken together and simplified.
find the length of a side of each square.
Solution : Let x metres and y metres represent the Example 2.10
sides of the two squares; 1 1
then the perimeter of the second square is 4y metres; 13−2x 23x+8 16− x
3 4
Solve + = = 4.
thus x+3 4x+5 x+3

9x – 4y = l. Solution : By transposition, we have


1 1
23x+8 16− x−13+2x
The areas of the two squares are x2 and y2 square 3
−4= 4
4x+5 x+3
metres; thus 35 7x
7x− 3+
6y2 – 29x2 =1.  =3
. 4
9x−1 4x+5 x+3
From the first equation, y = . Multiplying across, we have ·
4
35x 35x
By substitution in the second equation, 7x 2 − + 21x − 35 = 12x + 7x 2 + 15 +
3 4
3(9x−1)2
− 29x 2 = 1; −
137x
= 50;
8 12
that is, 11x2 – 54x – 5 = 0,  x=−
600
.
or (x – 5) (11x + 1) = 0; 137
Whence x = 5, the negative value being inadmissible.
9x−1 Example 2.11
Also, y= = 11.
4
x−8 x−4 x−5 (x−7)
Thus, the lengths are 5 metres and 11 metres. Solve + = + .
x−10 x−6 x−7 x−9
Solution : This equation might be solved by clearing of
fractions, but the work would be very laborious. The
HARDER EQUATIONS solution will be much simplified by proceeding as
In this topic we propose to give a miscellaneous follows:
collection of equations. Some of these will serve as a x−8 x−5 x−7 (x−4)
Transposing, − = − .
useful exercise for revision of the methods already x−10 x−7 x−9 x−6
explained in previous topics; but we also add others Simplifying each side separately, we have
(x−8)(x−7)−(x−5)(x−10) (x−7)(x−6)−(x−4)(x−9)
presenting more difficulty, the solution of which will = ;
(x−10)(x−7) (x−9)(x−6)
often be facilitated by some special artifice. x2 −15x+56−(x2 −15x+50) x2 −13x+42−(x2 −13x+36)
The following examples worked in full will sufficiently  (x−10)(x−7)
= (x−9)(x−6)
illustrate the most useful methods. 6 6
 (x−l0)(x−7)
= (x−9)(x−6).
Hence, since the numerators are equal, the
Example 2.8 denominators must be equal;
6x−3 3x−2 that is, (x – l0)(x – 7) = (x – 9)(x – 6),
Solve = .
2x+7 x+5
x2 – 17x + 70 = x2 – 15x + 54;
Solution : Multiplying up, we have
 16 = 2x;
(6x – 3)(x + 5) = (3x – 2)(2x + 7).
6x2 + 27x – 15 = 6x2 + 17x – 14;  x = 8.
The above equation may also be solved very neatly by
 10x = 1;
1 the following artifice.
 x= . The equation may be written in the form
10
(x−10)+2 (x−6)+2 (x−7)+2 (x−9)+2
+ = + ;
x−10 x−6 x−7 x−9
whence we have
By : Prashant Jain Sir
13 Chapter 1 | Fundamentals of Mathematics
2 2 2 2 3
1+ +1+ =1+ +1+ ; Solution : Transposing √𝑥 − 5 = 11.
𝑥−10 𝑥−6 𝑥−7 𝑥−9
1 1 1 1 Here we must cube both sides; thus x – 5 = 1331;
which gives + = + .
x−10 x−6 x−7 x−9 whence x = 1336.
1 1 1 1
Transposing, − = − ;
x−10 x−7 x−9 x−6
3 3
 = . Example 2.15
(x−10)(x−7) (x−9)(x−6)
and the solution may be completed as before. Solve √𝑥 + 5 + √3𝑥 + 4 = √12𝑥 + 1.
Solution : Squaring both sides,
Example 2.12 x + 5 + 3x + 4 + 2√(𝑥 + 5)(3𝑥 + 4) = 12x + 1.
5x−64 2x−11 4x−55 x−6 Transposing and dividing by 2,
Solve − = − .
x−13 x−6 x−14 x−7 √(𝑥 + 5)(3𝑥 + 4) = 4x – 4 …(1).
Solution : We have Squaring
1 1 1 1
5+ − (2 + )=4+ − (1 + ); (x + 5)(3x + 4) =16x2 – 32x + 16,
x−13 x−6 x−14 x−7

1

1
= −
1
.
1 or 13x2 – 51x – 4 = 0,
x−13 x−6 x−14 x−7 (x – 4) (13x + 1) =0,
The solution may now be completed as before, and we 1
obtain x = 10.  x = 4, or – .
13
If we proceed to verify the solution by substituting
these values in the original equation, it will be found
1
EQUATIONS INVOLVING SURDS that it is satisfied by x = 4, but not by x = − . But this
13
Sometimes equations are proposed in which the latter value will be found on trial to satisfy the given
unknown quantity appears under the radical sign. equation if we alter the sign of the second radical, thus
Such equations are very varied in character and often √𝑥 + 5 − √3𝑥 + 4 = √12𝑥 + 1.
require special artifices for their solution. Here, we On squaring this and reducing, we obtain
shall only consider a few of the simpler cases, which
−√(𝑥 + 5)(3𝑥 + 4) = 4𝑥 − 4 ... (2)
can generally be solved by the following method. Bring
to one side of the equation a single radical term by and a comparison of (1) and (2) shows that in the next
stage of the work the same quadratic equation is
itself. On squaring both sides this radical will 1
disappear. By repeating this process any remaining obtained each case, the roots of which are 4 and − ,
13
radicals can in turn be removed. as already found.

Example 2.13
From this it appears that when the solution of an
Solve 2√𝑥 − √4𝑥 − 11 = 1. equation requires that both sides should be squared,
Solution : Transposing 2√𝑥 − 1 = √4𝑥 − 11. we cannot be certain without trial which of the values
Squaring both sides then found for the unknown quantity will satisfy the
4x – 4√𝑥 + 1 = 4x – 11, original equation.
4√𝑥 = 12, √𝑥 = 3; In order that all the values found by the solution of the
 x = 9. equation may be applicable it will be necessary to take
into account both signs of the radicals in the given
Example 2.14 equation.
3
Solve 2 + √𝑥 − 5 = 13.

WORKED-OUT PROBLEMS - 2
3 2
1. Solve the equations : √5 − 2x = 7. A reaches Q in 3 hours, and B reaches P in 4
7 3
3
hours after they meet: find the speed of each train.
2. Solve the equations : √x − 7 = 2.
8. Solve the equations :
3. Solve the equations : 2√5x − 3 − 7√x = 0. x 9−x x+1 8−x
(1) + = +
x−2 7−x x−1 6−x
m n n m
(2) + = a, + = b.
4. Solve the equations : 3√1 − 7x + 4x 2 = 5 − 6x. x y x y

5.
3
Solve the equations : 1 + √x 3 − 3x 2 + 7x − 11 = x. 9. Solve the equations:
6
(1) √x + 16 − √x = ;
1 √x
2√x−
6. Solve the equations :
2√x−3
= 2
3
. (2) √𝑥 2 + 11𝑥 + 20 − √𝑥 2 + 5𝑥 − 1 = 3;
√x−1 √x+3
(3)√{4x 2 + 20x + 17 + √16x 2 + 11x + 10}
7. Two trains A and B start simultaneously from two +2(x + 2) = 0.
stations P and Q which are 260 kilometres apart.
10. Solve the equations:
By : Prashant Jain Sir
Chapter 1 | Fundamentals of Mathematics 14
a+√x a+√x b+√x 4𝑥 − 17 10𝑥 − 13 8𝑥 − 30 5𝑥 − 4
(1) + + + = + .
(b−√x)(c−√x) (b−√x)(c−√x) (c−√x)(a−√x)
𝑥−4 2𝑥 − 3 2𝑥 − 7 𝑥−1
c+√x
+ = 0.
(a−√x)(b−√x)
3 3 3
14. Solve the following :
(2) √1 + 𝑥 + √1 − 𝑥 = √2. √𝑥 − 4 + 3 = √𝑥 + 11.
11. Solve the following : 15. Solve the following :
𝑥−7 𝑥−9 𝑥 − 13 𝑥 − 15
− = − . √25𝑥 − 29 − √4𝑥 − 11 = 3√𝑥.
𝑥 − 9 𝑥 − 11 𝑥 − 15 𝑥 − 17

12. Solve the following : 16. Solve the following :


𝑥+3 𝑥+6 𝑥+2 𝑥+5 √8𝑥 + 17 − √2𝑥 = √2𝑥 + 9.
− = − .
𝑥+6 𝑥+9 𝑥+5 𝑥+8
17. Solve the following :
13. Solve the following : √𝑎 − 𝑥 + √𝑏 − 𝑥 = √𝑎 + √𝑏.

Answer Key
WORKED-OUT PROBLEMS - 2

1. –22 2. 3. 4. 5. 6.
7. I =60 km/hr and II =80 km/hr. 8. 9. 10. 11. 13
1
12. –7 13. 2 14. 5 15. 9 16. 8 17. 0, a – b.
2

By : Prashant Jain Sir


Title: Quadratic Equation
Chapter: Quadratic Equation
Approximate Time to Complete (to be filled by student): ________________________
Total Marks Scored:
Worksheet Compiled By: Prashant Jain (PJ Sir)

 Attempt the worksheet in one go. See answers in one go at the end.
 All questions carry 4 marks for positive and -1 if you leave and -2 if you attempt
incorrectly.
 All proving or show questions (if done correctly) are of 4 marks. There is no negative
marking.
 If you cheat in worksheet then you are only cheating and such sinners do not get
selection in JEE so for your own sake refrain from cheating.
 Maintain the solutions of this worksheet and share the link of solution pdf in the
tracker if you want to be monitored.

1. Polynomial :
A function f defined by f(x) = anxn + an – 1xn – 1 + .......+ a1x + a0
where a0, a1, a2, ......, an  R is called a polynomial of degree n with real coefficients (an  0,
n  W). If a0, a1, a2, ....., an  C, it is called a polynomial with complex coefficients.
2. Quadratic polynomial & Quadratic equation :
A polynomial of degree 2 is known as quadratic polynomial. Any equation f(x) = 0, where f is a
quadratic polynomial, is called a quadratic equation. The general form of a quadratic equation is
ax2 + bx + c = 0 .......(i)
Where a, b, c are real numbers, a  0.
If a = 0, then equation (i) becomes linear equation.
3. Difference between equation & identity :
If a statement is true for all the values of the variable, such statements are called as identities. If
the statement is true for some or no values of the variable, such statements are called as
equations.

Example : (i) (x + 3)2 = x2 + 6x + 9 is an identity


(ii) (x + 3)2 = x2 + 6x + 8, is an equation having no root.
(iii) (x + 3)2 = x2 + 5x + 8, is an equation having – 1 as its root.

A quadratic equation has exactly two roots which may be real (equal or unequal) or imaginary.
a x2 + b x + c = 0 is:

  a quadratic equation ifa  0 Two Roots


  a linear equation if a = 0, b  0 One Root
  a contradiction if a = b = 0, c  0 No Root
  an identity if a=b=c=0 Infinite Roots
2
If ax + bx + c = 0 is satisfied by three distinct values of ' x ', then it is an identity.

For Unacademy Subscription Use “PJLIVE” Code | Join t.me/pjsir42 for Updates
For More Info: 75970-84242, 95490-43333/2222
Example # 1 : (i) 3x2 + 2x – 1 = 0 is a quadratic equation here a = 3.
(ii) (x + 1)2 = x2 + 2x + 1 is an identity in x.
Solution : Here highest power of x in the given relation is 2 and this relation is satisfied by three
different values x = 0, x = 1 and x = – 1 and hence it is an identity because a
polynomial equation of nth degree cannot have more than n distinct roots.

4. Relation Between Roots & Co-efficients:


(i) The solutions of quadratic equation, a x2 + b x + c = 0, (a  0) is given by
 b  b2  4 a c
x=
2a
The expression, b2  4 a c  D is called discriminant of quadratic equation.
(ii) If ,  are the roots of quadratic equation,
a x2 + b x + c = 0 .......(i)
then equation (i) can be written as
a(x – ) (x – ) = 0 or ax2 – a( + )x + a  = 0 ......(ii)
equations (i) and (ii) are identical,
b coefficient of x
 by comparing the coefficients sum of the roots,  +  = – = –
a coefficient of x 2
c constant term
and product of the roots, = =
a coefficient of x 2
b c
(iii) Dividing the equation (i) by a, x2 + x+ =0 
a a
 b  c
  x2 –   x + = 0  x2 – ( + )x +  = 0
 a  a
 x2 – (sum of the roots) x + (product of the roots) = 0
Hence we conclude that the quadratic equation whose roots are  &  is x2 – ( + )x +  = 0

Example # 2 : If  and  are the roots of ax2 + bx + c = 0, find the equation whose roots are +2 and
+2.
Solution : Replacing x by x – 2 in the given equation, the required equation is
a(x – 2)2 + b(x – 2) + c = 0 i.e., ax2 – (4a – b)x + (4a – 2b + c) = 0.

Example # 3 : The coefficient of x in the quadratic equation x2 + px + q = 0 was taken as 17 in place


of 13, its roots were found to be – 2 and – 15. Find the roots of the original equation.
Solution : Here q = (– 2) × (– 15) = 30, correct value of p = 13. Hence original equation is
x2 + 13x + 30 = 0 as (x + 10) (x + 3) = 0
  roots are – 10, – 3

Self practice problems :


(1) If ,  are the roots of the quadratic equation cx2 – 2bx + 4a = 0 then find the quadratic
equation whose roots are
 
(i) , (ii) 2, 2 (iii)  + 1,  + 1
2 2
1  1    
(iv) , (v) ,
1  1    

For Unacademy Subscription Use “PJLIVE” Code | Join t.me/pjsir42 for Updates
For More Info: 75970-84242, 95490-43333/2222
(r  1)2 b2
(2) If r be the ratio of the roots of the equation ax2 + bx + c = 0, show that = .
r ac

Answers : (1) (i) cx2 – bx + a = 0


(ii) c2x2 + 4(b2 – 2ac)x + 16a2 = 0
(iii) cx2 – 2x(b + c) + (4a + 2b + c) = 0
(iv) (c – 2b + 4a)x2 + 2(4a – c) x + (c + 2b + 4a) = 0
(v) 4acx2 + 4(b2 – 2ac) x + 4ac = 0

For Unacademy Subscription Use “PJLIVE” Code | Join t.me/pjsir42 for Updates
For More Info: 75970-84242, 95490-43333/2222
Exercise – I
Section (A) : Relation between the roots and coefficients ; Quadratic Equation
1. For what value of 'a', the equation (a2 – a – 2)x2 + (a2 – 4)x + (a2 – 3a + 2) = 0, will have more
than two solutions ? Does there exist a real value of 'x' for which the above equation will be an
identity in 'a' ?

2. If  and  are the roots of the equation 2x2 + 3x + 4 = 0, then find the values of
 
(i) 2 + 2 (ii) +
 

3. If  and  are the roots of the equation ax2 + bx + c = 0, then find the equation whose roots are
given by
1 1
(i) + , + (ii) 2 + 2, 2 + 2
 

 
4. If  but 2 = 5 – 3, 2 = 5 – 3, then find the equation whose roots are and .
 

5. In copying a quadratic equation of the form x2 + px + q = 0, the coefficient of x was wrongly


written
as – 10 in place of – 11 and the roots were found to be 4 and 6. Find the roots of the correct
equation.

3  5 1
6. (i) Find the value of the expression 2x3 + 2x2 – 7x + 72 when x = .
2

1  15
(ii) Find the value of the expression 2x3 + 2x2 – 7x + 72 when x =
2

(iii) Solve the following equation 22x + 2x+2 – 32 = 0

7. Let a, b, c be real numbers with a  0 and let ,  be the roots of the equation ax2 + bx + c = 0.

Express the roots of a3x2 + abcx + c3 = 0 in terms of , 

8. If ,  are roots of x2 – px + q = 0 and  – 2,  + 2 are roots of x2 – px + r = 0, then prove that

16q + (r + 4 – q)2 = 4p2.

For Unacademy Subscription Use “PJLIVE” Code | Join t.me/pjsir42 for Updates
For More Info: 75970-84242, 95490-43333/2222
9. If one root of the equation ax2 + bx + c = 0 is equal to nth power of the other root, then show
that
(acn)1/(n + 1) + (anc)1/(n + 1) + b = 0.

10. If the sum of the roots of quadratic equation (a + 1)x2 + (2a + 3)x + (3a + 4) = 0 is –1, then find
the product of the roots.

11. Find the least prime integral value of '2a' such that the roots ,  of the equation 2 x2 + 6 x + a
 
= 0 satisfy the inequality  < 2.
 

Section (B) : Relation between the roots and coefficients quadratic equation
1. The roots of the equation (b – c) x2 + (c – a) x + (a – b) = 0 are
c a ab bc ca
(A) ,1 (B) ,1 (C) ,1 (D) ,1
bc bc ab ab

2. If ,  are the roots of quadratic equation x2 + p x + q = 0 and ,  are the roots of x2 + p x


– r = 0, then () . () is equal to :

(A) q + r (B) q – r (C) – (q + r) (D) – (p + q + r)

3. Two real numbers  &  are such that  +  = 3,   = 4, then  &  are the roots of the
quadratic equation:
(A) 4x2  12x  7 = 0 (B) 4x2  12x + 7 = 0 (C) 4x2  12x + 25 = 0 (D) none of these

4. For the equation 3x2 + px + 3 = 0, p > 0 if one of the roots is square of the other, then p is equal
to:
(A) 1/3 (B) 1 (C) 3 (D) 2/3

5. Consider the following statements :


S1 : If the roots of x2 – bx + c = 0 are two consecutive integers, then value of b2 – 4c is equal
to 1.
S2 : If  are roots of x2 – x + 3 = 0 then value of 4 is equal 7.

S3 : If  are the roots of x3 – 7x2 + 16 x – 12 = 0 then value of 2 + 2 +  is equal to


17.
State, in order, whether S1, S2, S3 are true or false
(A) TTT (B) FTF (C) TFT (D) FTT

For Unacademy Subscription Use “PJLIVE” Code | Join t.me/pjsir42 for Updates
For More Info: 75970-84242, 95490-43333/2222
Answer Key
Section (A) : Relation between the roots and coefficients ; Quadratic Equation
7 7
1. a = 2; No real value of x. 2. (i) – (ii) –
4 8

3. (i) ac x2 + b(a + c) x + (a + c)2 = 0 (ii) a2 x2 + (2ac – 4a2 – b2) x + 2b2 + (c – 2a)2 = 0

4. 3x2 – 19x + 3 = 0. 5. 8, 3

6. (i) 4 (ii) 72 (iii) 2

7.  = 2 and  = 2 or  = 2 and = 2

10. 2 11. 11

Section (B) : Relation between the roots and coefficients quadratic equation
1. (B) 2. (C) 3. (A) 4. (C) 5. (A)

Solution
Section (A) : Relation between the roots and coefficients ; Quadratic Equation
1. a2 – a – 2 = 0, a2 – 4 = 0, a2 – 3a + 2 = 0  a = 2, – 1 and a = ± 2 and a = 1, 2  a = 2
Now (x2 + x + 1) a2 – (x2 + 3) a – (2x2 + 4x – 2) = 0 will be an identity if x2 + x + 1 = 0 &
x2 + 3 = 0 & 2x2 + 4x – 2 = 0 which is not possible.
2
 3  7    2  2 7
2. (i) 2 + 2 = ( + )2 – 2 =   – 2(2) = (ii) + = =–
 2  4    8

b c
3.  +  =– and  =
a a
1 1   b b / a b b (a  c)
(i)  + + + =++ = + =     = b
   a c/a a c ac
 1  1 1 c a (a  c)2
and         =  + +2 = + +2=
     a c ac
1 1
 equation whose roots are  + and  + is
 
1 1
  + + acx2 + b(a + c)x + (a + c)2 = 0
 
b2 2ac 4a2  b2  2ac
(ii)  2 + 2 + 2 + 2 = ( + )2 – 2 + 4 = 2 – 2 + 4 = 2
a a a
c2 2(b2  2ac)
and (2 + 2) (2 + 2) = 22 + 2(2 + 2) + 4 = + +4
a2 a2
  equation whose roots are 2 + 2 & 2 + 2 is

For Unacademy Subscription Use “PJLIVE” Code | Join t.me/pjsir42 for Updates
For More Info: 75970-84242, 95490-43333/2222
a2 x2 + (2ac – b2 – 4a2)x + 2b2 + 4a2 + c2 – 4ac = 0 a2 x2 + (2ac – b2 – 4a2)x + 2b2 + (2a
– c)2 = 0

4. given 2 = 5 – 3 and 2 = 5 – 3   &  are the roots of x2 – 5x + 3 = 0


2 2
    25  6 19  
 + = = = and . =1
   3 3  
 
 equation have , as its roots is 3x2 – 19x + 3 = 0
 

5.  p = –11, q = 24

then correct equation will be x2 – 11x + 24 = 0


 (x – 8)(x –3) = 0  x = 3, 8

6. (i) E = 2x3 + 2x2 – 7x + 72


3  5i
Given, x =  2x – 3 = 5i  4x2 + 9 – 12x = – 25
2
  4x2 – 12x + 34 = 0  2x2 – 6x + 17 = 0 ........(i)
Given expression can be written as
E = (2x2 – 6x + 17) (x + 4) + 4 = 4 (using (i))
 1 15 1 15 14 7
(ii) x  2 =  x2 + x + =  x2 + x =  x2 + x =   
  2 4 4 4 2

2x3 + 2x2 – 7x + 72 = 2x (x2 + x) – 7x + 72 = 2x   – 7x + 72 = 7x – 7x + 72 =


7
2  
72.
(iii) 2x = y  y2 + 22 y – 32 = 0  y2 + 8y – 4y – 32 = 0
  y = 4 = 2x    2x  –8  x = 2.

b c
7.  ax2 + bx + c = 0  + =–   =
a a
 Let a3 x2 + (a b c) x + c3 = 0 

=      = ( ) ( + ) = 2 + 2
abc b c
  +=– 3
v ....(i)
a  a a
3
c
 =   = ()3 = (2) (2) ....(ii)
a 
From (i) and (ii) we can say that  = 2 and  = 2 and  = 2 and  = 2

8.  +  = p,  = q  ( – 2) ( + 2) = r   + 2 – 2 – 4 = r
q + 2( – ) – 4 = r  2 – 2 = r + 4 – q  2 + 2 = 2p
4 = r + 4 – q + 2p  4 = 2p – (r + 4 – q)  16 = 4p2 – (r + 4 – q)2
16q + (r + 4 – q)2 = 4p2.

1 1 n

=    c  n 1  c  n 1
c c n 1 b b
9.  . n =     + n = –  a + a =–
a a a     a
1 1 n n n 1 1 n
1 1
a n 1 . c n1  c n 1 . a n 1 b  0   a n 1 . c n1  a n 1 . c n 1  b  0 

For Unacademy Subscription Use “PJLIVE” Code | Join t.me/pjsir42 for Updates
For More Info: 75970-84242, 95490-43333/2222
1 1

a n
c  n 1

 a cn  n 1 b  0 Proved.

(2a  3) 3a  4 6  4
10. S= = –1  2a + 3 = a + 1  a = –2 ; p= = =2
a 1 a 1 2  1

11. 2x2 + 6x + a = 0
Its roots are ,    + = – 3 & =
 
 < 2
 
(  )2 – 2 9–a 2a – 9
 < 2  <1  > 0
 a a
9 
 a (–, 0)   ,    2a = 11 is least prime.
2 

Section (B) : Relation between the roots and coefficients quadratic equation
1. x = 1 is root. Let other root = 
ab ab
Product of the roots = (1) () =  roots are 1,
bc bc

2.  + = – p  = q   + = – p  = – r


(– ) (– ) = 2 – (+ ) + = 2 + p– r = (+ p) – r = – – r = –q – r = – (q +
r)

3. ( – ) = 4   ( – )2 = 16  ( + )2 – 4 = 16


7 7
 9 – 4 = 16   = –  equation is x2 – 3x – = 0
4 4

p
4. 3x2 + px + 3 = 0   + 2 = – .... (i)
3
3 = 1,    = 1, , 2   1
 =  or =2 put is (i)   p=3

5. S1 : x2 – bx + c = 0  | – | = 1  ( – )2 = 1  b2 – 4c = 1.
S2 :    +  = 1 and vkSj  = 3 
    4 + 4 = (2 + 2)2 – 22 2 = [( + )2 – 2]2 – 2()2 = (1 – 6)2 –
2(9) = 25 – 18 = 7
S3 :  = 7  = 16  = 12 2 = ()2 – 2 () =
49 – 32
 2 + 2 + 2 = 17

For Unacademy Subscription Use “PJLIVE” Code | Join t.me/pjsir42 for Updates
For More Info: 75970-84242, 95490-43333/2222
Title: Quadratic Equation
Chapter: Quadratic Equation
Approximate Time to Complete (to be filled by student): ________________________
Total Marks Scored:
Worksheet Compiled By: Prashant Jain (PJ Sir)

 Attempt the worksheet in one go. See answers in one go at the end.
 All questions carry 4 marks for positive and -1 if you leave and -2 if you attempt
incorrectly.
 All proving or show questions (if done correctly) are of 4 marks. There is no negative
marking.
 If you cheat in worksheet then you are only cheating and such sinners do not get
selection in JEE so for your own sake refrain from cheating.
 Maintain the solutions of this worksheet and share the link of solution pdf in the
tracker if you want to be monitored.

5. Theory Of Equations :
If 1, 2, 3,......n are the roots of the equation;
f(x) = a0xn + a1xn-1 + a2xn-2 +.... + an-1x + an = 0 where a0, a1,....,an are all real & a0  0
then, 
a a a a
 1 =  1 ,  1 2 = + 2 , 1 2 3 =  3 ,....., 1 2 3........n = (1)n n
a 0 a 0 a 0 a 0

Note : (i) If  is a root of the equation f(x) = 0, then the polynomial f(x) is exactly divisible by (x
) or (x ) is a factor of f(x) and conversely.
(ii) Every equation of nth degree (n 1) has exactly n roots & if the equation has more than
n roots, it is an identity.
(iii) If the coefficients of the equation f(x) = 0 are all real and  + i is its root, then   i is
also a root. i.e. imaginary roots occur in conjugate pairs.
(iv) An equation of odd degree will have odd number of real roots and an equation of even
degree will have even numbers of real roots.
(v) If the coefficients in the equation are all rational &  +  is one of its roots, then 
  is also a root where ,   Q &  is not square of a rational number.
(vi)If there be any two real numbers 'a' & 'b' such that f(a) & f(b) are of opposite signs, then
f(x) = 0 must have odd number of real roots (also atleast one real root) between ' a ' and
' b '.
(vii) Every equation f(x) = 0 of degree odd has atleast one real root of a sign opposite to that
of its last term. (If coefficient of highest degree term is positive).
Example # 4 : If 2x3 + 3x2 + 5x + 6 = 0 has roots , ,  then find  +  + ,  + +  and .
Solution : Using relation between roots and coefficients, we get
3 5 6
+  +  = = – , +  +  = ,  = – = – 3.
2 2 2

For Unacademy Subscription Use “PJLIVE” Code | Join t.me/pjsir42 for Updates
For More Info: 75970-84242, 95490-43333/2222
Self practice problems :
(3) If 2p3 – 9pq + 27r = 0 then prove that the roots of the equations rx3 – qx2 + px – 1 = 0
are in H.P.
(4) If , ,  are the roots of the equation x3 + qx + r = 0 then find the equation whose roots
are
(a)  + 2,  + , 
r r r
(b)  ,  , 
  
(c) 2, 2, 2
(d) 3, 3, 3

Answers: (4) (a) x3 + qx – r = 0 (b) x3 – qx2 – r2 = 0


(c) x3 + 2qx2 + q2 x – r2 = 0 (d) x3 – 3x2r + (3r2 +
q 3) x – r3 = 0

For Unacademy Subscription Use “PJLIVE” Code | Join t.me/pjsir42 for Updates
For More Info: 75970-84242, 95490-43333/2222
Exercise – I
Section (A) : Relation between roots and coefficients ; Higher Degree Equations
1. If  and  be two real roots of the equation x3 + px2 + qx + r = 0 (r 0) satisfying the relation
 + 1 = 0, then prove that r2 + pr + q + 1 = 0.

2. If , ,  are the roots of the equation x3 + px2 + qx + r = 0, then find the value of
 1  1   1 
         .
        
3. (i) Solve the equation 24x3 – 14x2 – 63x +  = 0, one root being double of another. Hence find
the value(s) of .
(ii) Solve the equation 18x3 + 81x2 + x + 60 = 0, one root being half the sum of the other two.
Hence find the value of    

4. If , ,  are roots of equation x3 – 6x2 + 10x – 3 = 0, then find cubic equation with roots 2
+ 1, 2 + 1, 2 + 1.

 
5. If ,  and  are roots of 2x3 + x2 – 7 = 0, then find the value of     .
, ,  
 

6. Find the roots of 4x3 + 20x2 – 23x + 6 = 0 if two of its roots are equal.

Section (B) : Relation between roots and coefficients ; Higher Degree Equations
1. If two roots of the equation x3  px2 + qx  r = 0, (r  0) are equal in magnitude but opposite in
sign, then:
(A) pr = q (B) qr = p (C) pq = r (D) None of these

1  1  1 
2. If , &  are the roots of the equation x3  x  1 = 0 then, + + has the value
1  1  1 
equal to:
(A) zero (B)  1 (C)  7 (D) 1

3. Let , , be the roots of (x – a) (x – b) (x – c) = d, d  0, then the roots of the equation


(x – ) (x – ) (x – ) + d = 0 are :
a b c
(A) a + 1, b + 1, c + 1 (B) a, b, c (C) a – 1, b – 1, c – 1 (D) , ,
b c a
 3  3   3
4. If , ,  are the roots of the equation x3 + ax + b = 0 then value of is equal to :
 2  2   2
3b 3b
(A) (B) (C) 3b (D) 2b
2a 2a
5. If two of the roots of equation x4 – 2x3 + ax2 + 8x + b = 0 are equal in magnitude but opposite in
sign, then value of 4a + b is equal to :
(A) 16 (B) 8 (C) –16 (D) –8

For Unacademy Subscription Use “PJLIVE” Code | Join t.me/pjsir42 for Updates
For More Info: 75970-84242, 95490-43333/2222
Answer Key
Section (A) : Relation between roots and coefficients ; Higher Degree Equations
(r  1)3
2. –
r2
3 3 5 1 25
3. (i) roots are , , ,  = 45 or ,  1, ,  = –25.
4 2 3 2 12
4 3 5
(ii) roots are , , ,  = 121
3 2 3
1 1
4. x3 – 15x2 + 67x – 77 = 0. 5. –3 6. , ,–6
2 2

Section (B) : Relation between roots and coefficients ; Higher Degree Equations
1. (C) 2. (C) 3. (B) 4. (A) 5. (C)

Solution
Section (A) : Relation between roots and coefficients ; Higher Degree Equations
1. Let 3rd root be  then  = –r But  = –1 (given)   = r
substituting x =  = r in the given equation we get r2 + pr + q + 1 = 0.

 1 1  1 
2. x3 + px2 + qx + r  = – r             
        
3
       1 (r  1)3 (r  1)3
=             =   1   = – r 3
=–
 r  r r  r r r2

4. 3 – 62 + 10 – 3 = 0.
x 1 (x  1)3 6(x  1)2
Let x = 2 + 1 new root  =  – + 5(x – 1) – 3 = 0
2 8 4
(x3 – 3x2 + 3x – 1) – 12 (x2 – 2x + 1) + 40 (x – 1) – 24 = 0 x3 – 15x2 + 67x – 77 = 0.

5. 2x3 + x2 – 7 = 0   +  +  = – 1/2,    0 ,  = 7/2


        1 1 1
      =      + 
+ +
 
+ = ( + ) + ( + ) + ( + )
   
1  1  1  1  1  1  1  1 1 1 1   
=     +     +     =–     –1–1–1=–   –3=–3
  2    2    2  2    2   

6. Let roots be ,  and   


20
  ++=–  2 +  = – 5 .............(1)
4
23 23
  .  +  +  = –  2 + 2 = – .............(2)
4 4
6 3
and 2 = – = – .............(3)
4 2
23
from equation (1) put  = – 5 – 2 in (2), we get 2 + 2 (– 5 – 2) = –
4
23
 122 + 40 – 23 = 0   = 1/2, –
6

For Unacademy Subscription Use “PJLIVE” Code | Join t.me/pjsir42 for Updates
For More Info: 75970-84242, 95490-43333/2222
1
(i) If  = then from (1), we get  = – 6
2
23 8
(ii) If  = – then from (1), we get  =
6 3
1 23 8
Note :  = and  = – 6 also satisfy (3) but  =  and  = does not satisfy (3)
2 6 3
1 1
 required roots are , , –6
2 2

Section (B) : Relation between roots and coefficients ; Higher Degree Equations
1. Let the roots be , , – then  +  –  = p
 =p ...(1) and   –  – 2 = q  2 = –q
...(2)
2
also –  = r  pq = r [using (1)].

2. 3
x –x–1=0  then 3 –  – 1 = 0 .............(1)

3
1  y 1  y  1  y  1 
Let  y    from equation (1)     1  0 y3 + 7y2 – y + 1 = 0
1  y 1  y  1  y  1 

1  1  1 
then   =–7
1  1  1 

3. Clearly (x – a) (x – b) (x – c) = –(x – ) (x – ) (x – )  if , ,  are the roots of given


equation
then (x – ) (x – ) (x – ) + d = 0 will have roots a, b, c.

 3  3   3 3  3b
4.   = =
 2  2   2 2(     ) 2a

5. Let roots are , – then  +  = 2 and –2 ( + ) = –8

 2 = 4   ±2

 24 – 2(23) + a(2)2 + 8(2) + b = 0  4a + b = –16

For Unacademy Subscription Use “PJLIVE” Code | Join t.me/pjsir42 for Updates
For More Info: 75970-84242, 95490-43333/2222
Title: Quadratic Equation
Chapter: Quadratic Equation
Approximate Time to Complete (to be filled by student): ________________________
Total Marks Scored:
Worksheet Compiled By: Prashant Jain (PJ Sir)

 Attempt the worksheet in one go. See answers in one go at the end.
 All questions carry 4 marks for positive and -1 if you leave and -2 if you attempt
incorrectly.
 All proving or show questions (if done correctly) are of 4 marks. There is no negative
marking.
 If you cheat in worksheet then you are only cheating and such sinners do not get
selection in JEE so for your own sake refrain from cheating.
 Maintain the solutions of this worksheet and share the link of solution pdf in the
tracker if you want to be monitored.

6. Nature of Roots:
Consider the quadratic equation, a x2 + b x + c = 0 having ,  as its roots;
D  b2  4 a c

D=0 D0
Roots are equal i.e.  =  =  b/2a Roots are unequal
& the quadratic expression can be expressed
as a perfect square of a linear polynomial

a, b, c  R & D > 0 a, b, c  R & D < 0


Roots are real Roots are imaginary  = p + i q,  = p  i q

a, b, c  Q & a, b, c  Q &
D is square of a rational number D is not square of a rational number
 Roots are rational  Roots are irrational
 i.e.  = p + q ,  = p  q
a = 1, b, c   & D is square of an integer
Roots are integral.

Example # 5 : For what values of m the equation (1 + m) x2 – 2(1 + 3m)x + (1 + 8m) = 0 has equal
roots.

For Unacademy Subscription Use “PJLIVE” Code | Join t.me/pjsir42 for Updates
For More Info: 75970-84242, 95490-43333/2222
Solution : Given equation is (1 + m) x2 – 2(1 + 3m)x + (1 + 8m) = 0 ........(i)
Let D be the discriminant of equation (i).
Roots of equation (i) will be equal if D = 0.
or 4(1 + 3m)2 – 4(1 + m) (1 + 8m) = 0
or 4(1 + 9m2 + 6m – 1 – 9m – 8m2) = 0
or m2 – 3m = 0 or, m(m – 3) = 0
  m = 0, 3.

Example # 6 : Find all the integral values of a for which the quadratic equation (x – a) (x – 10) + 1 = 0
has integral roots.
Solution : Here the equation is x2 – (a + 10)x + 10a + 1 = 0. Since integral roots will always be
rational it means D should be a perfect square.
From (i) D = a2 – 20a + 96.
  D = (a – 10)2 – 4  4 = (a – 10)2 – D
If D is a perfect square it means we want difference of two perfect square as 4
which is possible only when (a – 10)2 = 4 and D = 0.
 (a – 10) = ± 2  a = 12, 8

Example # 7 : If the roots of the equation (x – a) (x – b) – k = 0 be c and d, then prove that the roots of
the equation (x – c) (x – d) + k = 0, are a and b.
Solution : By given condition (x – a) (x – b) – k  (x – c) (x – d)
or (x – c) (x – d) + k  (x – a) (x – b)
Above shows that the roots of (x – c) (x – d) + k = 0 are a and b.

Example # 8 : Determine 'a' such that x2 – 11x + a and x2 – 14x + 2a may have a common factor.
Solution : Let x –  be a common factor of x2 – 11x + a and x2 – 14x + 2a.
Then x =  will satisfy the equations x2 – 11x + a = 0 and x2 – 14x + 2a = 0.
  2 – 11 + a = 0 and 2 – 14 + 2a = 0
Solving (i) and (ii) by cross multiplication method, we get a = 0, 24.

Example # 9 : Show that the expression x2 + 2(a + b + c)x + 3(bc + ca + ab) will be a perfect square
if a = b = c.
Solution : Given quadratic expression will be a perfect square if the discriminant of its
corresponding equation is zero.
i.e. 4(a + b + c)2 – 4.3 (bc + ca + ab) = 0
or (a + b + c)2 – 3(bc + ca + ab) = 0
1
or ((a – b)2 + (b – c)2 + (c – a)2) = 0
2
which is possible only when a = b = c.

For Unacademy Subscription Use “PJLIVE” Code | Join t.me/pjsir42 for Updates
For More Info: 75970-84242, 95490-43333/2222
Self practice problems :

(5) For what values of 'k' the expression (4 – k)x2 + 2(k + 2)x + 8k + 1 will be a perfect
square ?

(6) If (x – ) be a factor common to a1x2 + b1x + c and a2x2 + b2x + c, then prove that
(a1 – a2) = b2 – b1.

(7) If 3x2 + 2xy + 2y2 + 2ax – 4y + 1 can be resolved into two linear factors, Prove that
'' is a root of the equation x2 + 4ax + 2a2 + 6 = 0.

(8) Let 4x2 – 4( – 2)x +  – 2 = 0 (  R) be a quadratic equation. Find the values of ''
for which
(i) Both roots are real and distinct.
(ii) Both roots are equal.
(iii) Both roots are imaginary
(iv) Both roots are opposite in sign.
(v) Both roots are equal in magnitude but opposite in sign.

(9) If P(x) = ax2 + bx + c, and Q(x) = – ax2 + dx + c, ac  0 then prove that P(x) . Q(x) = 0
has atleast two real roots.

Answers. (5) 0, 3
(8) (i) (– , 2)  (3, ) (ii)  {2, 3} (iii) (2, 3) (iv) (– , 2) (v) 

For Unacademy Subscription Use “PJLIVE” Code | Join t.me/pjsir42 for Updates
For More Info: 75970-84242, 95490-43333/2222
Exercise – I
Section (A) : Nature of Roots
1. If 2 + i 3 is a root of the equation x2 + px + q = 0 (where p, q  R and i2 = –1), then find the
ordered pair (p, q).

2. If the roots of the equation x2 – 2cx + ab = 0 are real and unequal, then prove that the roots of
x2 – 2(a + b) x + a2 + b2 + 2c2 = 0 will be imaginary.

3. For what values of k the expression kx2 + (k + 1)x + 2 will be a perfect square of a linear
polynomial.

4. Show that if roots of equation (a2 – bc) x2 + 2(b2 – ac) x + c2 – ab = 0 are equal, then either
b = 0 or a3 + b3 + c3 = 3abc
1 1 1
5. If a, b, c  R, then prove that the roots of the equation + + = 0 are always real
xa x b x c

and cannot have roots if a = b = c.

1 1 1
6. If the roots of the equation + = are equal in magnitude but opposite in sign, then
(x  p) (x  q) r

show that p + q = 2 r and that the product of the roots is equal to (1/2) (p² + q²).

7. (i) If – 2 + i is a root of x3 + 63x +  = 0 (where  R – {0},  R and i2 = –1), then find
roots of equation.
1
(ii) If + i, is a root of 2x3 + bx2 + 3x + 1 = 0 (where b,  R – {0} and i2 = –1), then find
2

the value(s) of b.

8. Solve the equation x4 + 4x3 + 5x2 + 2x – 2 = 0, one root being – 1 + 1 .

9. Draw graph of y = 12x3 – 4x2 – 3x + 1. Hence find number of positive zeroes.

Section (B) : Nature of Roots


1. If one roots of equation x2 – 3 x +  = 0 ,   R is 3 + 2 then other root is

For Unacademy Subscription Use “PJLIVE” Code | Join t.me/pjsir42 for Updates
For More Info: 75970-84242, 95490-43333/2222
(A) 3 – 2 (B) – 2
(C) 2 – 3 (D) 2

2. If roots of equation 2x2 + bx + c = 0 ; b, c  R, are real & distinct then the roots of equation

2cx2 + (b – 4c) x + 2c – b + 1 = 0 are


(A) imaginary (B) equal
(C) real and distinct (D) can't say

3. Let one root of the equation x2 + x + m = 0 is square of other root. If m R then

(A)    – ,   {1}
1
(B) (–0]
 4

(C)    – ,  (D)    ,1


1 1
 9  4 

4. If a, b, c are integers and b2 = 4(ac + 5d2), d  N, then roots of the quadratic equation ax2 + bx
+ c = 0 are
(A) Irrational (B) Rational & different
(C) Complex conjugate (D) Rational & equal

5. Let a, b and c be real numbers such that 4a + 2b + c = 0 and ab > 0. Then the equation
ax2 + bx + c = 0 has
(A) real roots (B) imaginary roots
(C) exactly one root (D) none of these

6. Consider the equation x2 + 2x – n = 0, where n  N and n  [5, 100]. Total number of different
values of 'n' so that the given equation has integral roots, is
(A) 4 (B) 6 (C) 8 (D) 3

For Unacademy Subscription Use “PJLIVE” Code | Join t.me/pjsir42 for Updates
For More Info: 75970-84242, 95490-43333/2222
Answer Key
Section (A) : Nature of Roots
1. (– 4, 7) 3. 3±2 2

7. (i) 4, – 2 ± i 5 3 (ii) 3 or 4

8. – 1 ± 2 , – 1 ± 1 9. . Two positive roots.

Section (B) : Nature of Roots


1. (B) 2. (C) 3. (A) 4. (A) 5. (A)
6. (C)

Solution
Section (A) : Nature of Roots
1. 2 + i 3 and 2 – i 3 are the roots of x2 + px + q = 0
 –p = 4  p = –4 & q = 7.

2. x2 – 2cx + ab = 0 has roots real and unequal i.e. D1 > 0  4c2 – 4ab > 0  c2 – ab > 0
............(1)
Now, x2 – 2(a + b) x + (a2 + b2 + 2c2) = 0  D2 = 4(a + b)2 –4 (a2 + b2 + 2c2) = –8
(c2 – ab)
by (1) D2 < 0 roots will be imaginary.

6  36  4
3. D = 0  (k + 1)2 – 8k = 0  k2 + 1 – 6k = 0  k = k=3±2 2.
2

4. D=0  4(b2 – ac)2 – 4 (a2 – bc) (c2 – ab) = 0  b(a3 + b3 + c3 – 3abc) = 0


 Either b = 0 or a3 + b3 + c3 = 3abc.
1 1 1
5. + + =0 ...(1)
xa x b x c
 (x – b)(x – c) + (x – a) (x – c) + (x – a) (x – b) = 0
 3x2 – 2(a + b + c)x + ab + bc + ac = 0 ...(2)
D = 4(a + b + c) – 12(ab + bc + ac) = 4[a + b + c – ab – bc – ac]= 2[(a – b)2 + (b – c)2 +
2 2 2 2
(c – a)2]
 D  0  roots are always real But if a = b = c
1 1 1 3
Then + + =0 = 0
x a x b xc xa
which has no real 'x'

For Unacademy Subscription Use “PJLIVE” Code | Join t.me/pjsir42 for Updates
For More Info: 75970-84242, 95490-43333/2222
 this equation cannot have roots if a = b = c.

1 1 1
6. + =  x2 + x (p +q – 2r) + (pq – pr – qr) = 0
(x  p) (x  q) r
   (  )  (p  q  2r)  0  p + q = 2r
(p  q)2 1
&Product of roots = pq – r(p + q) = pq – r (p + q) = pq  =  (p2  q2 )
2 2

7. (i) Roots are – 2 + i, – 2 – i,  (say) ; Sum of roots (– 2 + i) + (– 2 – i) +  = 0 ;  = 4.
Sum of products taken two at a time.
4(– 2 + i) + 4(– 2 – i) + (4 + 2) = 63; – 16 + 4 + 2 = 63; 2 = 75
 = ± 5 3 . Roots are 4, – 2 ± i 5 3 .
1 1
(ii) Call roots as , + i, – i
2 2
b
–1= .........(1)
2
 1   1  1 3
    i  +    i  + + 2 = .......(2)
 2   2  4 2
1  1
   2  = ........(3)
4  2
1 3
(2)  + 2 = +
4 2
3  1 1
Put in (3)     = ;  (2 + 3) = – 1.   = – 1, .
2  2 2
1
If  = – 1, (3)  b = 4 =  b=3
2
Put in (1) b = 3 or 4

8. Given one root is –1 + i  2nd root will be –1 – i 


  x + 2x + 2 will be one factor of x + 4x + 5x + 2x – 2 = 0 and x2 + 2x –1 will be
2 4 3 2
another factor
 The roots of given equation are –1 ± 2 and –1 ± i.

1 1 1
9. y = (2x – 1)(6x2 + x – 1) = (2x –1)(2x + 1)(3x – 1). Hence roots are x =  , ,
2 3 2

f(1) > 0

Section (B) : Nature of Roots


1. += 3
3 +2 +  = 3 = – 2

2. D1 = b2 – 4.2.c > 0  b2 – 8c > 0


D2 = (b – 4c)2 4.2c. (2c – b + 1)
= b2 + 16c2 – 8bc – 16c2 + 8bc – 8c

For Unacademy Subscription Use “PJLIVE” Code | Join t.me/pjsir42 for Updates
For More Info: 75970-84242, 95490-43333/2222
= b2 – 8c > 0

3.  + 2 = – , 3 = m
6 + 3 + 32 ( + 2) = – 3
 m2 + m + 3m (–) + 3 = 0
 m2 + m (1– 3) + 3 = 0
 (1–3)2 – 43  0 {because m R}
 43 – 9 2 + 6 – 1  0
 ( –1)2 (4 –1)  0
1
 (– , ]  {1}
4

4. D = b2 – 4ac = 20d2  D  2 5d So roots are irrational.

5. D = b2 – 4ac = b2 – 4a (–4a –2b) = b2 + 16a2 + 8ab


Since ab > 0  D > 0. So equation has real roots.

6. For integral roots, D of equation should be perfect sq.  D = 4(1+n)


By observation, for n  N, D should be perfect sq. of even integer.
So D = 4(1+n) = 62, 82, 102,122,142, 162, 182, 202 . No. of values of n = 8.

For Unacademy Subscription Use “PJLIVE” Code | Join t.me/pjsir42 for Updates
For More Info: 75970-84242, 95490-43333/2222
Title: Quadratic Equation
Chapter: Quadratic Equation
Approximate Time to Complete (to be filled by student): ________________________
Total Marks Scored:
Worksheet Compiled By: Prashant Jain (PJ Sir)

 Attempt the worksheet in one go. See answers in one go at the end.
 All questions carry 4 marks for positive and -1 if you leave and -2 if you attempt
incorrectly.
 All proving or show questions (if done correctly) are of 4 marks. There is no negative
marking.
 If you cheat in worksheet then you are only cheating and such sinners do not get
selection in JEE so for your own sake refrain from cheating.
 Maintain the solutions of this worksheet and share the link of solution pdf in the
tracker if you want to be monitored.

7. Graph of Quadratic Expression :

 the graph between x, y is always a parabola.

 b D 
 the coordinate of vertex are   , 
 2a 4a 

 If a > 0 then the shape of the parabola is concave upwards & if a < 0 then the shape of
the parabola is concave downwards.

 the parabola intersect the yaxis at point (0, c).

 the xcoordinate of point of intersection of parabola with xaxis are the real roots of
the quadratic equation f (x) = 0. Hence the parabola may or may not intersect the xaxis.

8. Range of Quadratic Expression f(x) = ax2 + bx + c.

(i) Range :
 D 
If a > 0   f (x)   , 
 4 a 
 D 
If a < 0   f (x)    ,  
 4a 

For Unacademy Subscription Use “PJLIVE” Code | Join t.me/pjsir42 for Updates
For More Info: 75970-84242, 95490-43333/2222
D
Hence maximum and minimum values of the expression f (x) is  in respective
4a
b
cases and it occurs at x =  (at vertex).
2a
(ii) Range in restricted domain:
Given x  [x1, x2]
b
(a) If   [x1, x2] then,
2a
f (x) [ min { f(x1 ),f(x 2 )} , max { f(x1 ), f(x 2 )}]
b
(b) If   [x1, x2] then,
2a
  D   D 
f (x)  min  f ( x1 ) , f ( x 2 ) ,   , max  f ( x1 ) , f ( x 2 ) ,  
  4a   4 a  

9. Sign of Quadratic Expressions :


The value of expression f (x) = a x2 + b x + c at x = x0 is equal to ycoordinate of the point
on parabola y = a x2 + b x + c whose xcoordinate is x0. Hence if the point lies above the
xaxis for some x = x0, then f (x0) > 0 and viceversa.

We get six different positions of the graph with respect to xaxis as shown.

(i) Conclusions :

(a) a>0
(b) D>0
(c) Roots are real & distinct.
(d) f(x) > 0 in x  (– , )  (, )
(e) f(x) < 0 in x  (, )
(ii) (a) a>0

(b) D=0
(c) Roots are real & equal.
(d) f(x) > 0 in x  R – {}

(iii) (a) a>0

(b) D<0
(c) Roots are imaginary.
(d) f(x) > 0 x  R.

(iv) (a) a<0

(b) D>0
(c) Roots are real & distinct.
(d) f(x) < 0 in x  (– , )  (, )
(e) f(x) > 0 in x  (, )

For Unacademy Subscription Use “PJLIVE” Code | Join t.me/pjsir42 for Updates
For More Info: 75970-84242, 95490-43333/2222
(v) (a) a<0

(b) D=0
(c) Roots are real & equal.
(d) f(x) < 0 in x  R – {}

(vi) (a) a<0

(b) D<0
(c) Roots are imaginary.
(d) f(x) < 0 x  R.

Example # 10 : If c < 0 and ax2 + bx + c = 0 does not have any real roots then prove that
(i) a–b+c<0 (ii) 9a + 3b + c < 0.

Solution : c < 0 and D < 0   f(x) = ax2 + bx + c < 0 for all x  R


  f(– 1) = a – b + c < 0 and f(3) = 9a + 3b + c < 0

Example # 11 : Find the range of f(x) = x2 – 5x + 6.


25  24 
= – 
D b 5 1
Solution : minimum of f(x) =– at x = –  at x = =–
4a 2a  4  2 4
maximum of f(x)  
 1 
Hence range is   ,  
4
 
x2  x  4
Example # 12 : Find the range of rational expression y = if x is real.
x2  x  4
x2  x  4
Solution : y=  (y – 1)x 2 + (y + 1) x + 4(y – 1) = 0 ........(i)
x2  x  4

case- : if y  1, then equation (i) is quadratic in x


and  x is real
  D0   (y + 1) 2 – 16(y – 1) 2  0  (5y – 3) (3y – 5)  0

 3 5
 y  ,  – {1}
5 3
case- : if y = 1, then equation becomes
2x = 0  x = 0 which is possible as x is real.
Range is  ,
3 5

5 3 

x3
Example # 13 : Find the range of y = 2
, if x is real.
2x  3x  9
x3
Solution : y=
2x 2  3x  9
  2yx2 + (3y – 1)x + 3(3y – 1) = 0   .......(i)
case- : if y  0, then equation (i) is quadratic in x
 
For Unacademy Subscription Use “PJLIVE” Code | Join t.me/pjsir42 for Updates
For More Info: 75970-84242, 95490-43333/2222
 x is real
 D0
  (3y – 1)2 – 24y (3y – 1)  0
  (3y – 1) (21y + 1)  0
 1 1
y   ,  – {0}
 21 3 
case- : if y = 0, then equation becomes
x = –3 which is possible as x is real
Range y  
1 1
  , 
 21 3 

Self practice problems :

(10) If c > 0 and ax2 + 2bx + 3c = 0 does not have any real roots then prove that
(i) 4a – 4b + 3c > 0 (ii) a + 6b + 27c > 0 (iii) a + 2b + 6c > 0

(a  b)2
(11) If f(x) = (x – a) (x – b), then show that f(x)  – .
4
(12) Find the least integral value of 'k' for which the quadratic polynomial
(k – 1) x2 + 8x + k + 5 > 0  x  R.

x 2  34x  71
(13) Find the range of the expression , if x is a real.
x 2  2x  7
mx 2  3x  4
(14) Find the interval in which 'm' lies so that the expression can take all real
4x 2  3x  m
values,x  R.

Answers : (12) k = 4 (13) (– , 5]  [9, ) (14) m  (1, 7)

For Unacademy Subscription Use “PJLIVE” Code | Join t.me/pjsir42 for Updates
For More Info: 75970-84242, 95490-43333/2222
Exercise – I
Section (A) : Range of quadratic expression and sign of quadratic expression
1. Draw the graph of the following expressions:
(i) y = x2 + 4x + 3 (ii) y = 9x2 + 6x + 1 (iii) y = – 2x2 + x – 1

2. Find the range of following quadratic expressions :


(i) f(x) = –x2 + 2x + 3  x  R
(ii) f(x) = x2 – 2x + 3 x  [0, 3]
(iii) f(x) = x2 – 4x + 6 x  (0, 1]

3. If x be real, then find the range of the following rational expressions :


x2  x  1
(i) y=
x2  1
x 2  2x  9
(ii) y=
x 2 – 2x  9

kx 2  2(k  1)x  (9k  4)


4. Find the range of values of k, such that f(x) = is always negative.
x 2  8x  17

5. x2 + (a  b) x + (1  a  b) = 0, a, b  R. Find the condition on ' a ' for which


(i) Both roots of the equation are real and unequal  b  R .
(ii) Roots are imaginary  b  R

Section (B) : Range of quadratic expression and sign of quadratic expression


1. If  &  ( < ) are the roots of the equation x2 + bx + c = 0, where c < 0 < b, then
(A) 0 <  <  (B)  < 0 < 2< 2
(C) <  < 0 (D) < 0 < 2 < 2

2. Which of the following graph represents the expression f(x) = a x2 + b x + c (a  0) when


a > 0, b < 0 & c < 0 ?

(A) (B)

(C) (D)

3. The expression y = ax2 + bx + c has always the same sign as of 'a' if :


(A) 4ac < b2 (B) 4ac > b2
(C) 4ac = b2 (D) ac < b2
For Unacademy Subscription Use “PJLIVE” Code | Join t.me/pjsir42 for Updates
For More Info: 75970-84242, 95490-43333/2222
4. The entire graph of the expression y = x2 + kx – x + 9 is strictly above the x-axis if and only if
(A) k < 7 (B) –5 < k < 7
(C) k > – 5 (D) none of these

5. If a, b  R, a  0 and the quadratic equation ax2  bx + 1 = 0 has imaginary roots then a + b + 1


is:
(A) positive (B) negative
(C) zero (D) depends on the sign of b

6. If a and b are the non-zero distinct roots of x2 + ax + b = 0, then the least value of x2 + ax + b
is
3 9
(A) (B)
2 4
9
(C) – (D) 1
4

7. If y = – 2x2 – 6x + 9, then
(A) maximum value of y is –11 and it occurs at x = 2
(B) minimum value of y is –11 and it occurs at x = 2
(C) maximum value of y is 13.5 and it occurs at x = –1.5
(D) minimum value of y is 13.5 and it occurs at x = –1.5

8. If f(x) = x2 + 2bx + 2c2 and g(x) = – x2 – 2cx + b2 are such that min f(x) > max g(x), then the
relation between b and c, is

(A) no relation (B) 0 < c < b/2 (C) c2 < 2b (D) c2 > 2b2

For Unacademy Subscription Use “PJLIVE” Code | Join t.me/pjsir42 for Updates
For More Info: 75970-84242, 95490-43333/2222
Answer Key
Section (A) : Range of quadratic expression and sign of quadratic expression

1. (i) (ii) (iii)

2. (i) (–, 4] (ii) [2, 6] (iii) [3, 6)

1 3  4 
3. (i) 2, 2 (ii)   , 5   (1, )
   

 1
4.  ,  2  5. (i) a > 1 (ii) a .
 

Section (B) : Range of quadratic expression and sign of quadratic expression


1. (B) 2. (B) 3. (B) 4. (B) 5. (A)
6. (C) 7. (C) 8. (D

Solution
Section (A) : Range of quadratic expression and sign of quadratic expression

1. (i) (ii) (iii)

2. (i) y = –x2 + 2x + 3 = – (x2 – 2x – 3) = – (x – 3) (x + 1)


Here a < 0 and D>0 Range is (–, 4]
(ii) f(x) = x2 – 2x + 3  x  [0, 3]

For Unacademy Subscription Use “PJLIVE” Code | Join t.me/pjsir42 for Updates
For More Info: 75970-84242, 95490-43333/2222
y  [2, 6]  x  [0, 3] Ans.

Aliter :
f(x) = x2 – 2x + 3 = (x – 1)2 + 2
Since 0  x  3  –1  x – 1  2 0  (x – 1)2  4 
  2  (x – 1)2 + 2  6 2  f(x)  6
  Range of f(x) is [2, 6].

(iii) y = x2 – 4x + 6 ; x  (0, 1]
Here a > 0 and D < 0
f(0) = 6 f(1) = 3 Clearly for x  (0, 1]
 y  [3, 6)

3. (i) (y – 1)x2 – x + y – 1 = 0  xR  D0


1
  1 – 4(y – 1)2  0  (1 + 2y – 2) (1 – 2y + 2) 0  (2y – 1) (2y – 3)  0   y
2
3
 
2
(ii) y(x2 – 2x – 9) = x2 – 2x + 9  (y – 1) x2 – 2(y – 1) x – (y + 1)9 = 0
If y = 1  –(2) 9 = 0 contradiction.
y = 1  –(2) 9 = 0
  y1 D0   (5y + 4) (y – 1)  0
 4 
y    ,   (1, )
 5 

4. We can see for x2 – 8x + 17 D = 64 – 4(17) < 0


 x2 – 8x + 17 is always +ve If f(x) < 0
 kx2 + 2(k + 1)x + (9k + 4) < 0  k < 0 ......(1)
& 4(k + 1) – 4 k(9k + 4) < 0  k + 1 + 2k – 9k2 – 4k < 0 –8k2 – 2k + 1 < 0
2 2
8k2 + 2k – 1 > 0  8k2 + 4k – 2k – 1 > 0  4k(2k + 1) – 1(2k + 1) > 0
(2k + 1)(4k – 1) > 0 .......(2)
 1
combining (1) & (2) we get k   ,  
 2 

For Unacademy Subscription Use “PJLIVE” Code | Join t.me/pjsir42 for Updates
For More Info: 75970-84242, 95490-43333/2222
5. (i) x2 + (a – b) x + (1 – a – b) = 0  D>0
  2
(a – b) – 4 × 1 × (1 – a – b) > 0   a2 + b2 – 2ab – 4 + 4a + 4b > 0
  b2 + 2b (2 – a) + (a2 + 4a – 4) > 0  4(2 – a)2 – 4 × 1 × (a2 + 4a – 4) < 0
4 + a2 – 4a – a2 – 4a + 4 < 0   8a – 8 > 0   a>1
(ii) 2
(a – b) – 4 · 1 · (1 – a – b)  0    b + (4 – 2a)b + (a2 + 4a – 4)  0,
2
bR
as coefficient of b2 = 1, positive it in not possible.  a .

Section (B) : Range of quadratic expression and sign of quadratic expression


1. x2 + bx + c = 0  +=–b    = c
 Sum is +ve and product is – ve.    < 0 <  < ||

2. a > 0 & c < 0 is satisfied by (B) only [ f(0) = 0 & a > 0] Further in (B)
b
– >0  b<0 [ a > 0].
2a

3. For y = ax2 + bx + c to have the sign always same of 'a' b2 – 4ac < 0  4ac > b2

4. Here for D < 0 , entire graph will be above x-axis ( a > 0)


 (k – 1)2 – 36 < 0  (k – 7) (k + 5) < 0  –5<k<7

5. Let f(x) = ax2 – bx + 1. Given D < 0 & f(0) = 1 > 0


 possible graph is as shown
i.e. f(x) > 0 x  R or f(–1) > 0 f(–1) = a + b+ 1 > 0

6. x2 + ax + b = 0  a+b =–a  2a + b = 0 and ab = b


ab – b = 0  b (a – 1) = 0  Either b = 0 or a=1
But b  0 (given)  a=1  b=–2
 2
f(x) = x + x – 2
1
Least value occurs at x = –
2
1 1 9
Least value = – –2 =–
4 2 4

7.

y = –2x2 – 6x + 9 
b 6 3
   =  = –1.5
2a 2( 2) 2

For Unacademy Subscription Use “PJLIVE” Code | Join t.me/pjsir42 for Updates
For More Info: 75970-84242, 95490-43333/2222
& D = 36 – 4(–2)(9) = 36 + 72 = 108 
D 108 108
     = 13.5  y  ( ,13.5]
4a 4( 2) 8

8. min. f(x) > max. g(x)  – b2 + 2c2 > b2 + c2  c2 > 2b2  |c| > |b| 2

For Unacademy Subscription Use “PJLIVE” Code | Join t.me/pjsir42 for Updates
For More Info: 75970-84242, 95490-43333/2222
Title: Quadratic Equation
Chapter: Quadratic Equation
Approximate Time to Complete (to be filled by student): ________________________
Total Marks Scored:
Worksheet Compiled By: Prashant Jain (PJ Sir)

 Attempt the worksheet in one go. See answers in one go at the end.
 All questions carry 4 marks for positive and -1 if you leave and -2 if you attempt
incorrectly.
 All proving or show questions (if done correctly) are of 4 marks. There is no negative
marking.
 If you cheat in worksheet then you are only cheating and such sinners do not get
selection in JEE so for your own sake refrain from cheating.
 Maintain the solutions of this worksheet and share the link of solution pdf in the
tracker if you want to be monitored.

10. Location of Roots :


Let f (x) = ax² + bx + c, where a > 0 & a, b, c  R.

(i) (ii) (iii)

(i) Conditions for both the roots of f (x) = 0 to be greater than a specified number ‘x0’ are
b²  4ac  0 & f (x0) > 0 & ( b/2a) > x0.
(ii) Conditions for both the roots of f (x) = 0 to be smaller than a specified number ‘x0’ are
b²  4ac  0 & f (x0) > 0 & ( b/2a) < x0.
(iii) Conditions for a number ‘x0’ to lie between the roots of f (x) = 0 is f (x0) < 0.

(iv) (v)
(iv) Conditions that both roots of f (x) = 0 to be confined between the numbers x1 and
x2, (x1 < x2) are b²  4ac  0 & f (x1) > 0 & f (x2) > 0 & x1 < ( b/2a) < x2.

(v) Conditions for exactly one root of f (x) = 0 to lie in the interval (x1, x2) i.e.
x1 < x < x2 is f (x1). f (x2) < 0.

Example # 14 : Let x2 – (m – 3) x + m = 0 (m  R) be a quadratic equation, then find the values of 'm'


for which
(a) both the roots are greater than 2.
(b) both roots are positive.
(c) one root is positive and other is negative.
(d) One root is greater than 2 and other smaller than 1
(e) Roots are equal in magnitude and opposite in sign.
(f) both roots lie in the interval (1, 2)
For Unacademy Subscription Use “PJLIVE” Code | Join t.me/pjsir42 for Updates
For More Info: 75970-84242, 95490-43333/2222
Solution : (a)

Condition - :D  0  (m – 3)2 – 4m  0  m2 – 10m + 9  0


     (m – 1) (m – 9)  0 
      m  (– , 1]  [9, ) ......(i)

Condition - : f(2) > 0  4 – (m – 3)2 + m > 0 m < 10 ......(ii)


b m3
Condition - : – >2  2  m>7 ......(iii)
2a 2
Intersection of (i), (ii) and (iii) gives m  [9, 10)

(b)
Condition -  D  0  m  (– , 1] [9, )
Condition -  f(0) > 0  m>0
b m3
Condition -   >0  >0  m>3
2a 2
intersection gives m  [9, ) Ans.

(c)

Condition -  f(0) < 0   m < 0 Ans.

(d)

Condition -  f(1) < 0  4<0   m 


Condition -  f(2) < 0  m > 10 
Intersection gives m   Ans.
(e) sum of roots = 0  m=3
and f(0) < 0  m<0  m 

(f)

Condition -  D  0   m  (– , 1]  [9, )


Condition - f(1) > 0  1 – (m – 3) + m > 0  4 > 0which is true  m  R
Condition -  f(2) > 0  m < 10
b m3
Condition - V 1<– <2  1< <2  5<m<7
2a 2
intersection gives m 

Example # 15 : Find all the values of 'a' for which both the roots of the equation (a – 2)x2 – 2ax + a
= 0 lies in the interval (– 2, 1).
Solution : Case-I : f(–2) > 0  4(a – 2) + 4a + a > 0
8
9a – 8 > 0  a>
9
f(1) > 0  a – 2 – 2a + a > 0

For Unacademy Subscription Use “PJLIVE” Code | Join t.me/pjsir42 for Updates
For More Info: 75970-84242, 95490-43333/2222
– 2 > 0 not possible  a 
Case-II : a–2<0  a<2
8
f(–2) < 0  a<
9
f(1) < 0   aR
b 4
–2 < –<1  a<
2a 3
D0   a0
intersection gives a  0, 
8
 9
 8
complete solution a  0,   {2}
9 

Self practice problems :

(15) Let x2 – 2(a – 1)x + a – 1 = 0 (a  R) be a quadratic equation, then find the value of 'a'
for which
(a) Both the roots are positive (b) Both the roots are negative
(c) Both the roots are opposite in sign. (d) Both the roots are greater than 1.
(e) Both the roots are smaller than 1.
(f) One root is small than 1 and the other root is greater than 1.

(16) Find the values of p for which both the roots of the equation 4x2 – 20px + (25p2 + 15p
– 66) = 0 are less than 2.

(17) Find the values of '' for which 6 lies between the roots of the equation x2 + 2( – 3)x +
9 = 0.

(18) Let x2 – 2(a – 1)x + a – 1 = 0 (a  R) be a quadratic equation, then find the values of 'a'
for which
(i) Exactly one root lies in (0, 1). (ii) Both roots lies in (0, 1).
(iii) Atleast one root lies in (0, 1).
(iv) One root is greater than 1 and other root is smaller than 0.

(19) Find the values of a, for which the quadratic expression ax2 + (a – 2) x – 2 is negative
for exactly two integral values of x.

Answers : (15) (a) [2, ) (b)  (c) (– , 1) (d)  (e) (– , 1] (f) (2, )

 3
(16) (– , –1) (17)  ,  
 4 
(18) (i) (– , 1)  (2, ) (ii)  (iii) ( – , 1)  (2, ) (iv) 

(19) [1, 2)

11. Common Roots:


Consider two quadratic equations, a1 x2 + b1 x + c1 = 0 & a2 x2 + b2 x + c2 = 0.

For Unacademy Subscription Use “PJLIVE” Code | Join t.me/pjsir42 for Updates
For More Info: 75970-84242, 95490-43333/2222
(i) If two quadratic equations have both roots common, then the equations are identical and
their co-efficient are in proportion.
a1 b1 c1
i.e. = =
a2 b2 c2
(ii) If only one root is common, then the common root '  ' will be :
c1 a2  c 2 a1 b1 c 2  b2 c1
= =
a1 b2  a2 b1 c1 a2  c 2 a1
Hence the condition for one common root is :
  c1 a2  c 2 a1 2 =  a1 b2  a2 b1  b1 c 2  b2 c1 
Note : If f(x) = 0 & g(x) = 0 are two polynomial equation having some common root(s) then those
common root(s) is/are also the root(s) of h(x)  a f(x) + bg (x) = 0.

Example # 16 : If x2 – ax + b = 0 and x2 – px + q = 0 have a root in common and the second equation


has
ap
equal roots, show that b + q = .
2
Solution : Given equations are : x2 – ax + b= 0 ........ (i)
and 2
x – px + q = 0. ........ (ii)
Let  be the common root. Then roots of equation (ii) will be  and . Let  be the
other root of equation (i). Thus roots of equation (i) are ,  and those of equation (ii)
are , .
Now +=a ........ (iii)
 = b ........ (iv)
2 = p ........ (v)
2 = q ........ (vi)
L.H.S. = b + q =  + 2 = ( + ) ........ (vii)
ap (  ) 2
and R.H.S. = = =  ( + ) ........ (viii)
2 2
from (vii) and (viii), L.H.S. = R.H.S.

Example # 17 : If a, b, c  R and equations ax2 + bx + c = 0 and x2 + 2x + 9 = 0 have a common


root, show that a : b : c = 1 : 2 : 9.
Solution : Given equations are : x2 + 2x + 9 = 0 ........(i)
and ax2 + bx + c = 0 ........(ii)
Clearly roots of equation (i) are imaginary since equation (i) and (ii) have a common
root, therefore common root must be imaginary and hence both roots will be common.
Therefore equations (i) and (ii) are identical
a b c
 = =
1 2 9
 a:b:c=1:2:9

Self practice problems :


(20) If the equations ax2 + bx + c = 0 and x3 + x – 2 = 0 have two common roots then show
that 2a = 2b = c.

For Unacademy Subscription Use “PJLIVE” Code | Join t.me/pjsir42 for Updates
For More Info: 75970-84242, 95490-43333/2222
a b c
(21) If ax2 + 2bx + c = 0 and a1x2 + 2b1x + c1 = 0 have a common root and , , are in
a1 b1 c1

A.P. show that a1, b1, c1 are in G.P.

12. Graphs of Polynomials


y = anxn + ............ + a1x + a0. The points where y' = 0 are called turning points which are
critical in plotting the graph.

Example # 18 : Draw the graph of y = 2x3 – 15x2 + 36x + 1


Solution. y = 6x2 – 30x + 36 = 6(x – 3) (x – 2)

x 2 3  –

y 29 28  –

Example # 19 : Draw the graph of y = –3x4 + 4x3 + 3,

Solution. y = –12x3 + 12x

y = –12x2 (x – 1)

x 0 1  –
y 3 4 – –

For Unacademy Subscription Use “PJLIVE” Code | Join t.me/pjsir42 for Updates
For More Info: 75970-84242, 95490-43333/2222
Exercise – I
Section (A) : Location of Roots
1. If both roots of the equation x2 – 6ax + 2 – 2a + 9a2 = 0 exceed 3, then show that a > 11/9.

2. Find all the values of 'K' for which one root of the equation x²  (K + 1) x + K² + K  8 = 0,
exceeds 2 & the other root is smaller than 2.

3. Find all the real values of 'a', so that the roots of the equation
(a2 – a + 2) x2 + 2(a – 3) x + 9 (a4 – 16) = 0 are of opposite sign.

4. Find all the values of 'a', so that exactly one root of the equation x2 – 2ax + a2 – 1 = 0, lies
between the numbers 2 and 4, and no root of the equation is either equal to 2 or equal to 4.

5. If  &  are the two distinct roots of x² + 2 (K  3) x + 9 = 0, then find the values of K such that 
,   ( 6, 1).

Section (B) : Common Roots & Graphs of Polynomials


1. If one of the roots of the equation ax2 + b x + c = 0 be reciprocal of one of the roots of
a1 x2 + b1 x + c1 = 0, then prove that (a a1  c c1)2 = (b c1  a b1) (b1c  a1b).

2. Find the value of 'a' so that x2 – 11 x + a = 0 and x2 – 14x + 2a = 0 have a common root.

3. If ax2 + bx + c = 0 and bx2 + cx + a = 0 have a common root and a, b, c are non-zero real
a3  b 3  c 3
numbers, then find the value of .
abc

4. If x2 + px + q = 0 and x2 + qx + p = 0, (p  q) have a common root, show that 1 + p + q = 0 ;


show that their other roots are the roots of the equation x2 + x + pq = 0.

5. Draw the graphs of following :


(i) y = 2x3 + 9x2 – 24x + 15 (ii) y = – 3x4 + 4x3 + 12x2 – 2

6. Find values of ‘k’ if equation x3 – 3x2 + 2 = k has


(i) 3 real roots
(ii) 1 real root

Section (C) : Location of Roots


1. If b > a, then the equation (x  a) (x  b)  1 = 0, has:
(A) both roots in [a, b] (B) both roots in (a)
(C) both roots in [b) (D) one root in (a) & other in (b, )
For Unacademy Subscription Use “PJLIVE” Code | Join t.me/pjsir42 for Updates
For More Info: 75970-84242, 95490-43333/2222
2. If , are the roots of the quadratic equation x2  2p (x  4)  15 = 0, then the set of values of
'p' for which one root is less than 1 & the other root is greater than 2 is:
(A) (7/3, ) (B) (, 7/3) (C) x  R (D) none of these

3. If ,  be the roots of 4x2 – 16x +  = 0, where   R, such that 1 <  < 2 and 2 <  < 3, then
the number of integral solutions of  is
(A) 5 (B) 6 (C) 2 (D) 3

4. Set of real values of k if the equation x2 – (k–1)x + k2 = 0 has atleast one root in (1,2) is
(A) (2, 4) (B) [–1, 1/3] (C) {3} (D) 

Section (D) : Common Roots & Graphs of Polynomials


1. If the equations k (6x2 + 3) + rx + 2x2 – 1 = 0 and 6k (2x2 + 1) + px + 4x2 – 2 = 0 have both
roots common, then the value of (2r – p) is
(A) 0 (B) 1/2 (C) 1 (D) none of these

2. If 3x2 – 17x + 10 = 0 and x2 – 5x +  = 0 has a common root, then sum of all possible real
values of  is
29 26 29
(A) 0 (B)  (C) (D)
9 9 3

3. If a, b, p, q are nonzero real numbers, then two equations 2a2 x2  2 ab x + b2 = 0 and


p2 x2 + 2 pq x + q2 = 0 have :
(A) no common root (B) one common root if 2 a2 + b2 = p2 + q2
(C) two common roots if 3 pq = 2 ab (D) two common roots if 3 qb = 2 ap

x3 – 4x
4. The graphs of y = is
4
y

127

2
(A) 15

2
x
–4 O 1

For Unacademy Subscription Use “PJLIVE” Code | Join t.me/pjsir42 for Updates
For More Info: 75970-84242, 95490-43333/2222
y

30

3
(B) O
–1 2 x

–2

y
(C)

–2 2 x

(D)

5. The graphs of y = x4 – 2x2 + 5 is


y

30
y

127

2 3
(A) 15 (B) O
–1 2
2 x
x
–4 O 1 –2

y
(C) (D)

–2 2 x

For Unacademy Subscription Use “PJLIVE” Code | Join t.me/pjsir42 for Updates
For More Info: 75970-84242, 95490-43333/2222
Answer Key
Section (A) : Location of Roots
2. K  ( 2, 3) 3. a  (–2, 2) 4. a  (1, 5) – {3}
5. 6 < K < 6.75

Section (B) : Common Roots & Graphs of Polynomials


2. a = 0, 24 3. 3

5. (i) (ii)

6. (i) k[–2,2] (ii) k(–,–2)  (2,)

Section (C) : Location of Roots


1. (D) 2. (B) 3. (D) 4. (D)

Section (D) : Common Roots & Graphs of Polynomials


1. (A) 2. (C) 3. (A) 4. (C)
5. (D)

Solution
Section (A) : Location of Roots
1. For both roots to exceed 3
(i) D  0  36a2 – 8 + 8a – 36a2  0  a  1

(ii) f(3) > 0  9 – 18a + 2 – 2a + 9a2 > 0  9a2 – 20a + 11 > 0  a  (–, 1) 
 11 
 9 , 
 
b 11
(iii) > 3  3a > 3  a > 1   (i)  (ii)  (iii)  a > .
2a 9

2. Here for one root to exceed 2 and other to be smaller than 2, f(2) < 0
 4 – 2k – 2 + k2 + k – 8 < 0
 k2 – k – 6 < 0
 –2 < k < 3.

For Unacademy Subscription Use “PJLIVE” Code | Join t.me/pjsir42 for Updates
For More Info: 75970-84242, 95490-43333/2222
3. Here coefficient of x2 is always positive
 f(0) < 0
 (a2 + 4) (a – 2) (a + 2) < 0
 a (–2, 2)

4. (i) D>0
4a2 – 4(a2 – 1) > 0
4>0 xR
(ii) f(2) f(4) < 0
(4 – 4a + a2 – 1) (16 – 8a + a2 – 1) < 0
(a – 3)2 (a – 1) (a – 5) < 0
a  (1, 5) – {3}

5. x2 + 2(k – 3) x + 9 = 0 ......(i)
Roots , of equation (i) are distinct & lies between –6 and 1
D > 0  4(K – 3)2 – 36 > 0  k(k – 6) > 0
 k (– , 0) (6, ) ......(ii)
f(1) > 0 1 + 2 (k – 3)+ 9 > 0
 2k + 4 > 0
 k (–2, ) ......(iii)
f(–6) > 0 36 – 12 (k – 3) + 9 > 0
 27 
 4k – 27 < 0  k   – , ......(iv)
 4 
b –2(K – 3)
–6 < – < 1 – 6 < < 1  –1 < k – 3 < 6 2<k<9 ......(v)
2a 2
 27 
(ii)  (iii) (iv) (v) ls   k   6, .
 4 

Section (B) : Common Roots & Graphs of Polynomials


1
1. If  is one of the root of a1x2 + b1x + c1 = 0. Then will be a root of ax2 + bx + c = 0  c2

+ b + a = 0
& a12 + b1 + c1 = 0 have one common root. 
  applying the condition for one common root we get (aa1 – cc1)2 = (bc1 – ab1) (b1c – a1b)

For Unacademy Subscription Use “PJLIVE” Code | Join t.me/pjsir42 for Updates
For More Info: 75970-84242, 95490-43333/2222
2. Given equation are x2 – 11x + a = 0 .........(i) x2 – 14x + 2a = 0 .........(ii)
2
Multiplying equation (i) by 2 and then subtracting, we get x – 8x = 0  x = 0, 8
If x = 0, a = 0 If x = 8, a = 24

3. ax2 + bx + c = 0 bx2 + cx + a = 0 have a common root, say 


2  1
 a2 + b + c = 0  b2 + c + a = 0    
ab  c bc  a 2
ac  b2
2
ab  c 2 bc  a2  ab  c 2   bc  a2 
2
  ,    2
   2
  (ab – c2) (ac – b2) = (bc – a2)2
ac  b2 ac  b2  ac  b   ac  b 
a3  b 3  c 3
a3 + b3 + c3 = 3abc [ a 0]   3  Ans.
abc
Aliter :
By observation, x = 1 is the common root
 a + b + c = 0  a3 + b2 + c3 = 3abc or = 3.

4. Let  is the common root hence 2 + p + q = 0 2 + q + p = 0


2  1
  2 = –(p + q),  = 1  – (p + q) = 1 p + q + 1 = 0
2
p q 2 qp qp
Let other roots be  and  then  +  = – p ,  = q  +  = – q,  = p
 q  qp qp qp
 –  = q – p,        = p  = q
 p  p  p
Equation having ,  as roots
x2 – ( + ) x +  = 0  x2 – (p + q) x + pq = 0  x2 + x + pq = 0 [ p + q =
– 1]

5. (i) (ii)

6. f(x) = x3 – 3x2 + 2
f’(x) = 3x2 – 6x = 3x (x – 2) = 0

f(0) = 2
f(2) = 8 – 12 + 2 = – 2
(i) k[–2,2]
(ii) k(–,–2) (2,)

For Unacademy Subscription Use “PJLIVE” Code | Join t.me/pjsir42 for Updates
For More Info: 75970-84242, 95490-43333/2222
Section (C) : Location of Roots
1. (x – a) (x – b) – 1 = 0. Let f(x) = (x – a) (x – b) – 1 f(a) = –1  f(b) = – 1
 the graph will be mouth opening upwards.

 (D) will be correct

2. x2 – 2px + (8p – 15) = 0


f(1) < 0 and f(2) < 0
 f(1) = 1 – 2p + 8p – 15 < 0
 p < 7/3
and f(2) = 4 – 4p + 8p – 15 < 0
11
 4p – 11 < 0  p 
4
Hence p  ( , 7 / 3) Ans.

1 2
 

3. 4x2 –16x +  = 0
f(1) > 0 and f(2) < 0 and f(3) > 0

1 2 3
  
O 12 16

f(1) = 4 – 16 +  > 0   > 12 ...(i)


f(2) = 16 – 32 +  < 0   < 16 ...(ii)
f(3) = 36 – 48 +  > 0   > 12 ...(iii)
by (i)  (ii) (iii)
12 <  < 16. So  = 13,14,15 has 3 integral solutions.

4. D0
(k–1)2 – 4k2  0  (k + 1) (3k – 1)  0

–1 1/3
Case- I Exactly one in (1,2)
f(1) f(2) < 0  (1– k+1+1) (4–2k+2+k2)<0
 (3 – k) (k2 – 2k + 6) < 0
3–k<0 k>3
if one roots is – 1 then k = 3
– 1 × k = 9  k = – 9  k 3

if one root is 2 then k2 – 2k + 6 = 0 not possible

For Unacademy Subscription Use “PJLIVE” Code | Join t.me/pjsir42 for Updates
For More Info: 75970-84242, 95490-43333/2222
 k
Case-II If both roots lie in (1,2)
f(1) > 0 & f(2) > 0
3 – k > 0  k < 3 & k2 – 2k + 6 > 0
 k

1 1

Section (D) : Common Roots & Graphs of Polynomials


1. x2(6k + 2) + rx + (3k –1) = 0  x2(12k + 4) + px + 6k – 2 = 0
6k  2 r 3k  1 r 1
For both roots common,      2r – p = 0 Ans.
12k  4 p 2(3k  1) p 2

2
2. 3x2 – 17x + 10 = 0   x= or 5
3
If x = 5 is comman  =0
2 26 26
If x= is comman  = ; Sum =
3 9 9

3. D1 = 4a2b2 – 8a2b2 = – 4a2b2 < 0 img. root ; D2 = 4p2q2 – 4p2q2 = 0 equal, real
roots
So no common roots.

For Unacademy Subscription Use “PJLIVE” Code | Join t.me/pjsir42 for Updates
For More Info: 75970-84242, 95490-43333/2222
Title: Quadratic Equation
Chapter: Quadratic Equation
Approximate Time to Complete (to be filled by student): ________________________
Total Marks Scored:
Worksheet Compiled By: Prashant Jain (PJ Sir)

 Attempt the worksheet in one go. See answers in one go at the end.
 All questions carry 4 marks for positive and -1 if you leave and -2 if you attempt
incorrectly.
 All proving or show questions (if done correctly) are of 4 marks. There is no negative
marking.
 If you cheat in worksheet then you are only cheating and such sinners do not get
selection in JEE so for your own sake refrain from cheating.
 Maintain the solutions of this worksheet and share the link of solution pdf in the
tracker if you want to be monitored.

Exercise – I
ONLY ONE OPTION CORRECT TYPE
1. Let a > 0, b > 0 & c > 0. Then both the roots of the equation ax2 + bx + c = 0
(A) are real & negative (B) have negative real parts
(C) are rational numbers (D) have positive real parts

2. If the roots of the equation x2 + 2ax + b = 0 are real and distinct and they differ by atmost 2m,
then b lies in the interval

(A) (a2 – m2, a2) (B) [a2 – m2, a2)


(C) (a2, a2 + m2) (D) none of these

3. The set of possible values of  for which x2 – (2 – 5 + 5)x + (22 – 3 – 4) = 0 has roots,
whose sum and product are both less than 1, is
(A)  1 ,  (C) 1 ,  (D)  1 ,
5 5 5
(B) (1, 4)
 2   2  2 

4. If p, q, r, s  R, then equaton (x2 + px + 3q) (–x2 + rx + q) (–x2 + sx – 2q) = 0 has

(A) 6 real roots (B) atleast two real roots


(C) 2 real and 4 imaginary roots (D) 4 real and 2 imaginary roots

5. Find the set of all real values of  such that the root of the equation
x2 + 2(a + b + c)x + 3 (ab + bc + ca) = 0 are always real for any choice of a, b, c (where a, b,
c represents sides of scalene triangle).
 4  4  1 5 4 5
(A)  ,  (B)  ,   (C)  ,  (D)  , 
 3  3  3 3 3 3

6. If coefficients of biquadratic equation are all distinct and belong to the set {–9, – 5, 3, 4, 7}, then
equation has

For Unacademy Subscription Use “PJLIVE” Code | Join t.me/pjsir42 for Updates
For More Info: 75970-84242, 95490-43333/2222
(A) atleast two real roots
(B) four real roots, two are conjugate surds and other two are also conjugate surds
(C) four imaginary roots
(D) None of these

7. Let p, q, r, s  R, x2 + px + q = 0, x2 + rx + s = 0 such that 2 (q + s) = pr then

(A) atleast one of the equation have real roots.


(B) either both equations have imaginary roots or both equations have real roots.
(C) one of equations have real roots and other equation have imaginary roots
(D) atleast one of the equations have imaginary roots.

8. The equation, x =  2x2 + 6x  9 has:


(A) no solution (B) one solution
(C) two solutions (D) infinite solutions

9. If (2 +  – 2)x2 + ( + 2) x < 1 for all x  R, then  belongs to the interval


 2 2 
(A) (–2, 1) (B)  2, (C)  , 1 (D) none of these
 5  5 

10. Let conditions C1 and C2 be defined as follows : C1 : b2 – 4ac  0, C2 : a, –b, c are of same
sign. The roots of ax2 + bx + c = 0 are real and positive, if
(A) both C1 and C2 are satisfied (B) only C2 is satisfied
(C) only C1 is satisfied (D) none of these

x2  x  c
11. If 'x' is real, then can take all real values if :
x 2  x  2c
(A) c  [0, 6] (B) c  [ 6, 0] (C) c  (  6)  (0, ) (D) c  ( 6, 0)

12. If both roots of the quadratic equation (2  x) (x + 1) = p are distinct & positive, then p must lie
in the interval:
(A) (2, ) (B) (2, 9/4) (C) (– , – 2) (D) (– , )

13. If two roots of the equation (a – 1) (x2 + x + 1)2 – (a + 1) (x4 + x2 + 1) = 0 are real and distinct,
then 'a' lies in the interval

(A) (–2, 2) (B) (– , –2)  (2, ) (C) (2, ) (D) (––2)

14. The equations x3 + 5x2 + px + q = 0 and x3 + 7x2 + px + r = 0 have two roots in common. If
the third root of each equation is represented by x1 and x2 respectively, then the ordered pair
(x1, x2) is:
(A) ( 5,  7) (B) (1,  1) (C) ( 1, 1) (D) (5, 7)

15. If a, b, c are real and a2 + b2 + c2 = 1, then ab + bc + ca lies in the interval:

1   1  1
(A)  , 2 (B) [0, 2] (C)   , 1 (D)  1, 
2   2   2

For Unacademy Subscription Use “PJLIVE” Code | Join t.me/pjsir42 for Updates
For More Info: 75970-84242, 95490-43333/2222
Answer Key
ONLY ONE OPTION CORRECT TYPE
1. (B) 2. (B) 3. (D) 4. (B) 5. (A)
6. (A) 7. (A) 8. (A) 9. (B) 10. (A)
11. (D) 12. (B) 13. (B) 14. (A) 15. (C)

Solution
ONLY ONE OPTION CORRECT TYPE
1. a > 0, b > 0 and c > 0  ax2 + bx + c = 0
c
 +  = – b/a = – ve,  = = + ve
a
–ve real part


2. x2 + 2ax + b = 0  0 < |  –  |  2m  0 < (   )2 – 4   2m

0 < 4a2 – 4b  4m2  a 2 – m2  b < a 2  b  [a2 – m2, a2)

3. Sum of roots < 1


 2 – 5 + 5 < 1  ( – 1)( – 4) < 0  1<<4 ...(1)
 Product of roots < 1
5
 22 – 3 – 5 < 0  (2 – 5) ( + 1) < 0  –1 <  < ...(2)
2
5
 (1) & (2)  1<< .
2

4. Dis. of x2 + px + 3q is p2 – 12q  D1
Dis. of –x2 + rx + q is r2 + 4q  D2
Dis. of –x2 + sx – 2q is s2 – 8q  D3
Case 1 : If q < 0, then D1 > 0, D3 > 0 and D2 may or may not be positive
Case 2 : If q > 0, then D2 > 0 and D1, D3 may or may not be positive
Case 3 : If q = 0, then D1  0, D2  0 and D3  0
from Case 1, Case 2 and Case 3 we can say that the given equation has atleast two real roots.

5. We, know that a + b > c, b + c > a and c + a > b  c – a < b, a – b < c, b – c < a
squaring on both sides and adding (c – a)2 + (a – b)2 + (b – c)2 < a2 + b2 + c2
a2 + b2 + c2 – 2(ab + bc + ca) < 0  (a + b + c)2 – 4(ab + bc + ca) < 0
(a  b  c)2
 <4 ....(i)
ab  bc  ca
Now roots of equation x2 + 2(a + b + c) x + 3 (ab + bc + ca) = 0 are real, then D  0
(a  b  c)2
 4 (a + b + c)2 – 4. 3 (ab + bc + ca)  0   3
ab  bc  ca

For Unacademy Subscription Use “PJLIVE” Code | Join t.me/pjsir42 for Updates
For More Info: 75970-84242, 95490-43333/2222
(a  b  c)2 4
So 3  < 4  <
ab  bc  ca 3

6. Let biquadratic is ax4 + bx3 + cx2 + dx + e = 0


 a + b + c + d + e = 0 as a, b, c, d, e  {–9, – 5, 3, 4, 7}
Hence x = 1 is a root. So real root will be atleast two.

7. x2 + px + q = 0  D1 = p2 – 4q .....(1)
x2 + rx + s = 0  D2 = r2 – 4s .....(2)
D1 + D2 = p2 + r2 – 4 (q + s) [ pr = 2(q + s)]
= (p – r)2 > 0
Since D1 + D2 is +ve, so atleast
one of the equation has real roots.

8. x = – 2x2 + 6x – 9  D = 36 – 4(–2) (–9) = 36 – 72 < 0 & a < 0


So quadratic expression – 2x2 + 6x – 9 is always negative whereas x is always +ve
 Equation will not hold for any x.  x  So x = – 2x2 + 6x – 9 has no solution.

9. ( + 2) ( – 1)x2 + ( + 2)x – 1 < 0  x  R  ( + 2) ( – 1) < 0 


 –2 <  < 1 ...(1)
(a < 0) and ( + 2)2 + 4( + 2) ( – 1) < 0 (D < 0)
 ( + 2) ( + 2 + 4 – 4) < 0  ( + 2) (5 – 2) < 0
2
 –2 <  < ...(2)
5
 2
(1) & (2)      2, Also  = –2  0 < 1 which is true
 5 
 2
 Required interval is    2,
 5 

10. C1 : b2 – 4a c  0 ; C2 : a, – b, c are of same sign


ax2 + bx + c = 0 has real roots then D  0 i.e. C1 must be satisfied
b b
(i) Let a, – b, c > 0 then – > 0 (ii) Let a, – b, c < 0 then – >0
2a 2a
Hence, for roots to be + ve, C2 must be satisfied. Thus both C1, C2 are satisfied

x2  x  c
11. Let y = ; x  R and y  R   (y – 1) x2 + (y + 1)x + 2y c – c = 0
x 2  x  2c
 xR  D0  (y + 1)2 – 4 c(y – 1) (2y – 1)  0
  y2 + 1 + 2y – 4c [2y2 – 3y + 1]  0
 (1 – 8c)y2 + (2 + 12c) y + 1 – 4c  0 ....... (1)

For Unacademy Subscription Use “PJLIVE” Code | Join t.me/pjsir42 for Updates
For More Info: 75970-84242, 95490-43333/2222
1
Now for all y  R (1) will be true if 1 – 8c > 0  c < and D  0
8
  4 (1 + 6c)2 – 4 (1 – 8c) (1 – 4c)  0   1 + 36c2 + 12c – 1 – 32c2 + 12c  0
  4c2 + 24c  0  –6c0
But c = –6 and c = 0 will not satisfy given condition  c (–6, 0)

12. (2 – x) (x + 1) = p  x2 – x + (p – 2) = 0 ...(1)
(1) has both roots distinct & positive
b
 (i) D > 0 (ii) f(0) > 0 (iii) >0
2a
9
(i) D>0p< (ii) f(0) > 0  p>2
4
b 1
(iii) = > 0 (always true)
2a 2

(i)  (ii)  (iii)  p   2,


9
 .
 4 

13. (a – 1) (x2 + x + 1)2 – (a + 1) (x4 + x2 + 1) = 0 ........(1)


 x4 + x2 + 1 = (x2 + x + 1) (x2 – x + 1)
 (1) becomes
 (x2 + x + 1) [(x2 + x + 1) (a – 1) – (a + 1) (x2 – x + 1)] = 0  (x2 + x + 1) (x2 – ax +
1) = 0
Here two roots are imaginary and for other two roots to be real D>0
 a2 – 4 > 0  a  (–, –2) (2, )

14. x3 + 5x2 + px + q = 0    + + x1 = – 5,  + x1 +x1 = p ...(1)
x1

x3 + 7x2 + px + r = 0    + + x2 = – 7,  + x2 + x2 = p ...(2)
x2
Subtracting (2) from (1)
    x 2  x 2  p
 + x1 +x1 = p   (x1 – x2) ( – ) = 0 [x1 
 (x1 – x 2 )   (x1 – x 2 )  0
x2]
 +=0  x1 = – 5   x2 = – 7

15.  a2 + b2 + c2 = 1   (a + b + c)2 = a2 + b2 + c2 + 2 (ab + bc +


ca)  0
1
 1 + 2 (ab + bc + ca)  0   (ab + bc + ca)  – ........(1)
2
 a2 + b2 + c2 – (ab + bc + ca)  0  (ab + bc + ca)  1 ........(2)
 1 
 From (1) and (2) we can say that (ab + bc + ca)    , 1
 2 

For Unacademy Subscription Use “PJLIVE” Code | Join t.me/pjsir42 for Updates
For More Info: 75970-84242, 95490-43333/2222
Title: Quadratic Equation
Chapter: Quadratic Equation
Approximate Time to Complete (to be filled by student): ________________________
Total Marks Scored:
Worksheet Compiled By: Prashant Jain (PJ Sir)

 Attempt the worksheet in one go. See answers in one go at the end.
 All questions carry 4 marks for positive and -1 if you leave and -2 if you attempt
incorrectly.
 All proving or show questions (if done correctly) are of 4 marks. There is no negative
marking.
 If you cheat in worksheet then you are only cheating and such sinners do not get
selection in JEE so for your own sake refrain from cheating.
 Maintain the solutions of this worksheet and share the link of solution pdf in the
tracker if you want to be monitored.

Exercise – I
SINGLE AND DOUBLE VALUE INTEGER TYPE
1. Find number of integer roots of equation x (x + 1) (x + 2) (x + 3) = 120.
2 2
3 3
2. Find product of all real values of x satisfying (5  2 6 )x  (5  2 6 )x = 10

3. If a, b are the roots of x2 + px + 1 = 0 and c, d are the roots of x2 + qx + 1 = 0. Then find the
value of
(a  c) (b  c) (a + d) (b + d)/(q2  p2).

4. ,  are roots of the equation  (x2 – x) + x + 5 = 0. If 1 and 2 are the two values of  for
 1  2 
     
which the roots ,  are connected by the relation + = 4, then the value of  2 1 
is
   14 
 
 

5. Let one root of equation ( – m) x2 + x + 1 = 0 be double of the other. If  be real and 8m  k


then find the least value of k.

6. Let  be the roots of the equation x2 + ax + b = 0 and  be the roots of x2 – ax + b – 2 = 0.
1 1 1 1 5
If  = 24 and     , then find the value of a.
    6

7. If a > b > 0 and a3 + b3 + 27ab = 729 then the quadratic equation ax2 + bx – 9 = 0 has roots 
,  ( < ). Find the value of 4 – a.

8. Let  and  be roots of x2 – 6(t2 – 2t + 2)x – 2 = 0 with  > . If an = n – n for n  1, then

For Unacademy Subscription Use “PJLIVE” Code | Join t.me/pjsir42 for Updates
For More Info: 75970-84242, 95490-43333/2222
a100  2a98
find the minimum value of (where t  R)
a99

9. If , , ,  are the roots of the equation x4  Kx3 + Kx2 + Lx + M = 0, where K, L & M are
real numbers, then the minimum value of 2 + 2 + 2 + 2 is – n. Find the value of n.

2x
10. Consider y = , where x is real , then the range of expression y2 + y  2 is [a, b]. Find the
1 x2
value of (b – 4a).

11. If the roots of the equation x3 + Px2 + Qx  19 = 0 are each one more than the roots of the
equation x3  Ax2 + Bx  C = 0, where A, B, C, P & Q are constants, then the value of A + B
+ C is equal to :

12. If one root of the equation t2 – (12x)t – (f(x) + 64x) = 0 is twice of other, then find the maximum
value of the function f(x), where x  R.

13. The values of k, for which the equation x2 + 2 (k  1) x + k + 5 = 0 possess atleast one positive
root, are (– , – b]. Find value of b.

14. Find the least value of 7a for which atleast one of the roots of the equation x2 – (a – 3) x + a =
0 is greater than 2.

15. If the quadratic equations 3x2 + ax + 1 = 0 & 2x2 + bx + 1 = 0 have a common root, then the
value of the expression 5ab  2a2  3b2 is

16. The equations x2  ax + b = 0, x3  px2 + qx = 0, where a, b, p, q  R – {0} have one common


ap
root & the second equation has two equal roots. Find value of .
qb

16
17. If x – y and y – 2x are two factors of the expression x3 – 3x2y + xy2 + y3, then  4 is
11

For Unacademy Subscription Use “PJLIVE” Code | Join t.me/pjsir42 for Updates
For More Info: 75970-84242, 95490-43333/2222
Answer Key
SINGLE AND DOUBLE VALUE INTEGER TYPE
1. 2 2. 8 3. 1 4. 73 5. 9
6. 10 7. 13 8. 6 9. 1 10. 9
11. 18 12. 32 13. 1 14. 63 15. 1
16. 2 17. 1

Solution
SINGLE AND DOUBLE VALUE INTEGER TYPE
1.  x2  3x  2  x2  3x  = 120
Let x2 + 3x = y  y2 + 2y – 120 = 0  (y + 12) (y – 10)= 0
 2
y = –12  x + 3x + 12 = 0   x 
2
y = 10  x + 3x – 10 = 0   (x + 5) (x – 2) = 0  x = {–5, 2}
x = 2, – 5 are only two integer roots.

5  2 6 
x2 3 1 1
2.   10  t = 10  t2 – 10t + 1 = 0
5  2 6 
x 2 3 t

10  96 2 1
t= 52 6  (5  2 6 )x 3
= (5  2 6 ) or
2 52 6
 x2 – 3 = 1 or x2 – 3 = –1
 x = 2 or –2 or  2 or 2
Product 8

3. x2 + px + 1 = 0 a + b = – p, ab = 1 ; x2 + qx + 1 = 0 c + d = – q, cd = 1
a + b = –p, ab = 1 c + d = –q, cd = 1
RHS = (a – c) (b – c) (a + d) (b + d) = (ab – ac – bc + c2) (ab + ad + bd + d2)
= (1 – ac – bc + c2) (1 + ad + bd + d2)
= 1 + ad + bd + d2 – ac – a2cd – abcd – acd2 – bc – abcd – b2cd – bcd2 + c2 + adc2 + bdc2 +
c2d2
= 1 + ad + bd + d2 – ac – a2 – 1 – ad – bc – 1 – b2 – bd + c2 + ac + bc + 1 [
ab = cd = 1]
= c2 + d2 – a2 – b2 = (c + d)2 – 2cd – (a + b)2 + 2ab = q2 – 2 – p2 + 2 = q2 – p2 = LHS.
Proved.
Aliter :
RHS = (ab – c(a + b) + c2) (ab + d(ab + d(a + b) + d2) = (c2 + pc + 1) (1 – pd + d2)
...(1)
Since c & d are the roots of the equation x2 + qx + 1 = 0
 c2 + qc + 1 = 0  c2 + 1 = –qc & d2 + qd + 1 = 0  d2 + 1 = –qd.
 (i) Becomes = (pc – qc) (–pd – qd) = c(p – q) (–d) (p + q) = –cd (p2 – q2)
= cd (q2 – p2) = q2 – p2 = LHS. Proved.

For Unacademy Subscription Use “PJLIVE” Code | Join t.me/pjsir42 for Updates
For More Info: 75970-84242, 95490-43333/2222
 1 5
4.  are roots of x2 – ( – 1) x + 5 = 0  += and  =
 
   2  2
 + =4   =4  ( + )2 = 6 
  
(  1)2 30
  =   2 – 32 + 1 = 0 ............(1)
2 
 1, 2 are roots of (1)  1 + 2 = 32 and 12 = 1
 1  2 
1 2 (1   2 )2  212 (32)2  2    
 + = = = 1022   2 1 
= 73
2 1 1 2 1  14 
 
 

  1 2 2 1
5.  + 2 = –   =– Also 22 =  =
m 3(  m)  m 9(  m) 2 m
9
  22 – 9 + 9m = 0  R  D0  81 – 72m  0  m  .
8

6.  = b ;  = b – 2   = b(b – 2) = 24


1 1 1 1 a
 bx2 + ax + 1 = 0 has roots ,   
    b
1 1 1 1 a
(b – 2)x2 – ax + 1 = 0 has root ,   
    b2
1 1 1 1 a a 5 2a 5 2a 5
      ;  ;  ; a = 10.
    b b2 6 b(b  2) 6 24 6

7. a3 + b3 + (– 9)3 = 3 · a · b (– 9)  a + b – 9 = 0 or a = b = – 9. Which is
rejected.
As a>b>–9   a+b–9=0  x = 1 is a root
9 9  9 
other root = .   = ,  = 1 4 – a = 4 – a   = 4 + 9 = 13.
a a  a 

8. Let t 2 – 2t + 2 = k  2 – 6k – 2 = 0  2 – 2 = 6 k
a100 – 2a98 = 100 – 2.98 – 100 + 2.98 = 98(2 – 2) – 98(2 – 2) = 6k(99 – 99)
a100 – 2a98 = 6k.a99
a100  2a98
= 6k = 6(t2 – 2t + 2) = 6[(t – 1)2 + 1]
a99
a100  2a98
 min. value of is 6.
a99

9. x4 – Kx3 + Kx2 + Lx + M =    = K,   = K,    = –L

= M  2 + 2 + 2 + 2 = ( +  +  + )2 – 2    
 K2 – 2K = (K – 1)2 – 1  (2 + 2 + 2 + 2)min = – 1

2 x
10. y=  x2y – 2x + y = 0  x  R
1  x2

For Unacademy Subscription Use “PJLIVE” Code | Join t.me/pjsir42 for Updates
For More Info: 75970-84242, 95490-43333/2222
D0
4 – 4y2  0  y  [– 1, 1]
2
Now f(y) = y + y – 2
9 9
 f(y)   – 
, b = 0  b – 4a = 0 – 4   = 9 . Ans.
9
, 0 a=
 4  4  4 

11. Let , ,  be the roots of x3 – Ax2 + Bx – C = 0 ...(1)


3 2
the roots of x + Px + Qx – 19 = 0 will be ( + 1), ( + 1), ( + 1)
 ( + 1) ( + 1) ( + 1) = 19  ( +  +  + 1) ( + 1) = 19
  +  +  +  +  +  +  + 1 = 19  C + B + A = 18
[using (1)].

12.  + 2 = 12x     = 4x    ()(2) = – f(x) – 64x 


  2
f(x) = – (32x + 64x)  2
f(x) = – 32(x + 2x)  f(x) = – 32((x + 1)2 – 1)

  f(x)  32.  Maximum value of f(x) is 32

13. Case-I : Both the roots are positive


x2 + 2 (K – 1) x + (K + 5) = 0
(i) D0  4(K – 1)2 – 4 (K + 5)  0  (K + 1) (K – 4)  0

(ii) f(0) > 0   K+5>0    K>–5


b 2(1  k)
(iii) – >0  > 0  K<1
2a 2

–5 –1 0 1 4
K  (– , – 1] ... (i)
Case-II : One root is +ve and other root is –ve
f(0) < 0 k + 5 < 0  K<–5 ... (ii)

Case-III : One root is zero and other is +ve


b
f(0) = 0 & >0 K=–5 ... (iii)
2a

For Unacademy Subscription Use “PJLIVE” Code | Join t.me/pjsir42 for Updates
For More Info: 75970-84242, 95490-43333/2222
Union of all the three cases give K  (– , – 1] = (– , – b]  b = 1.

14. case- I : Both roots are greater than 2.


or one root is 2 & other is greater than 2
D0
 (a – 3)2 – 4a 0
 a2 – 10 a + 9  0
(a – 1) (a – 9)  0
a (–, 1] [9, ) ... (i)
–b a–b
 2   >2 
2a 2
  a>7 ... (ii)
f(2)  0   4 – 2(a – 3) + a 0
– a + 10 0  a 10 ... (iii)
(i)(ii) (iii) gives
a [9, 10] ... (iv)

Case-II : One root is greater than 2


f(2) < 0   – a + 10 < 0
 a > 10   a (10, ) ....(v)
(iv)(v) gives final answer as
a [9, )
 Least value of 7a is 63.

2
3 a a 1 3 1
15. =  (3b – 2a) (a – b) = (3 – 2)2 
2 b b 1 2 1
  5ab – 3b2 – 2a2 = 1

16. x3 – px2 + qx = 0 ...(1)


x(x2 – px + q) = 0 ; x = 0, x2 – px + q = 0  0, ,  are the roots of equation
(1)
2 = p   = p/2 ...(2) & 2 = q ...(3)
Since  is the root of the equation x2 – ax + b = 0 also,

For Unacademy Subscription Use “PJLIVE” Code | Join t.me/pjsir42 for Updates
For More Info: 75970-84242, 95490-43333/2222
 2 – a + b = 0
a p ap
q– +b=0 [using (2) & (3)]   ap = 2(b + q) 2 = .
2 qb

17. Given expression is f(x, y) = x3 – 3x2y + xy2 + y3 .....(i)


since (x – y) is a factor of (i) 
  x3 – 3x3 + x3 + x3 = 0  +  – 2 = 0 .....(ii)
(y – 2x) is also a factor of (i)  3 2 2 3
x – 3x (2x) + x (4x ) + (8x ) = 0
 4+ 8– 5 = 0 .....(iii)
11 3 16 16 11  3 
Solving (ii) & (iii) we get = and = –   4 =  4  = 4 – 3 = 1.
4 4 11 11 4  4 

For Unacademy Subscription Use “PJLIVE” Code | Join t.me/pjsir42 for Updates
For More Info: 75970-84242, 95490-43333/2222
Title: Quadratic Equation
Chapter: Quadratic Equation
Approximate Time to Complete (to be filled by student): ________________________
Total Marks Scored:
Worksheet Compiled By: Prashant Jain (PJ Sir)

 Attempt the worksheet in one go. See answers in one go at the end.
 All questions carry 4 marks for positive and -1 if you leave and -2 if you attempt
incorrectly.
 All proving or show questions (if done correctly) are of 4 marks. There is no negative
marking.
 If you cheat in worksheet then you are only cheating and such sinners do not get
selection in JEE so for your own sake refrain from cheating.
 Maintain the solutions of this worksheet and share the link of solution pdf in the
tracker if you want to be monitored.

Exercise – I
ONE OR MORE THAN ONE OPTIONS CORRECT TYPE
1. Possible values of 'p' for which the equation (p2 – 3p + 2)x2 – (p2 – 5p + 4)x + p – p2 = 0 does
not possess more than two roots is/are
(A) 0 (B) 1 (C) 2 (D) 4

2. If a, b are non-zero real numbers and ,  the roots of x2 + ax + b = 0, then


(A) 2, 2 are the roots of x2 – (2b – a2) x + a2 = 0
1 1
(B) , are the roots of bx2 + ax + 1 = 0
 
 
(C) , are the roots of bx2 + (2b – a2) x + b = 0
 
(D) ( – 1), ( – 1) are the roots of the equation x2 + x (a + 2) + 1 + a + b = 0

3. If ,  are the roots of ax2 + bx + c = 0 (a  0) and  + ,  +  are the roots of,


Ax2 + Bx + C = 0 (A  0) for some constant , then
1 B b 1 b B 
(A)  =  (B)  = 
2  A a  2  a A 
b2  4 a c B2  4 A C b2  4ac B2  4 A C
(C) 2
= 2
(D) 2
=
a A a A2

4. If one root of the equation 4x2 + 2x – 1 = 0 is ‘’, then


1  5 1 5
(A)  can be equal to (B)  can be equal to
4 4
(C) other root is 43 – 3. (D) other root is 43 + 3

5. If ,  are roots of x2 + 3x + 1 = 0, then


(A) (7 – ) (7 – ) = 0 (B) (2 – ) (2 – ) = 11
2 2 2 2
       
(C)  = –2 (D)     = 18
3  1 3  1  1      1

For Unacademy Subscription Use “PJLIVE” Code | Join t.me/pjsir42 for Updates
For More Info: 75970-84242, 95490-43333/2222
6. If both roots of x2 – 32x + c = 0 are prime numbers then possible values of c are

(A) 60 (B) 87 (C) 247 (D) 231

7. Let f(x) = x2 – a(x + 1) – b = 0, a, b  R – {0}, a + b  0. If  and  are roots of equation f(x) =


1 1 2
0, then the value of + – is equal to
2
  a 2
  a ab
a a2
(A) 0 (B) f(a) + a + b (C) f(b) + a + b (D) f   + +a+b
2 4

8. If f(x) is a polynomial of degree three with leading coefficient 1 such that f(1) = 1, f(2) = 4, f(3)
= 9, then
3
6
(B) f   =  
6
(A) f(4) = 22
5 5
(C) f(x) = x3 holds for exactly two values of x. (D) f(x) = 0 has a root in interval (0, 1).

9. Let P(x) = x32 – x25 + x18 – x11 + x4 – x3 + 1. Which of the following are CORRECT ?
(A) Number of real roots of P(x) = 0 are zero.
(B) Number of imaginary roots of P(x) = 0 are 32.
(C) Number of negative roots of P(x) = 0 are zero.
(D) Number of imaginary roots of P(x) + P(–x) = 0 are 32.

10. If ,  are the real and distinct roots of x2 + px + q = 0 and 4, 4 are the roots of x2 – rx + s =
0, then the equation x2 – 4qx + 2q2 – r = 0 has always (given   –)

(A) two real roots (B) two negative roots


(C) two positive roots (D) one positive root and one negative root

11. x2 + x + 1 is a factor of a x3 + b x2 + c x + d = 0, then the real root of above equation is


(a, b, c, d  R)
(A)  d/a (B) d/a (C) (b – a)/a (D) (a – b)/a

12. If – 5 + i, – 5 + i (where 2  2 ; ,  R and i2 = –1) are roots of x3 + 15x2 + cx + 860 =
0, c  R, then
(A) c = 222
(B) all the three roots are imaginary
(C) two roots are imaginary but not complex conjugate of each other.
(D) – 5 + 7i 3 , – 5 – 7i 3 are imaginary roots.

13. Let f(x) = ax2 + bx + c > 0,  x  R or f(x) < 0,  x  R. Which of the following is/are
CORRECT ?
(A) If a + b + c > 0 then f(x) > 0,  x  R (B) If a + c < b then f(x) < 0,  x  R
(C) If a + 4c > 2b then f(x) < 0, x  R (D) ac > 0.

For Unacademy Subscription Use “PJLIVE” Code | Join t.me/pjsir42 for Updates
For More Info: 75970-84242, 95490-43333/2222
14. Let x1 <  <  <  < x4, x1 < x2 < x3. If f(x) is a cubic polynomial with real coefficients such
that
(f())2 + (f())2 + (f())2 = 0, f(x1) f(x2) < 0, f(x2) f(x3) < 0 and f(x1) f(x3) > 0 then which of
the following are CORRECT ?
(A)  (x1, x2),  (x2, x3) and  (x3, x4) (B)  (x1, x3), ,  (x3, x4)
(C) ,  (x1, x2) and   (x4, ) (D)  (x1, x3),  (x2, x3) and  (x2,
x4)

15. If f(x) is cubic polynomial with real coefficients,  <  <  and x1 < x2 be such that f() = f()
= f() = f(x1) = f  (x2) = 0 then possible graph of y = f(x) is (assuming y-axis vertical)

(A) (B)

(C) (D)

3 4 5
16. Let f(x) = + + , then f(x) = 0 has
x2 x 3 x4
(A) exactly one real root in (2, 3) (B) exactly one real root in (3, 4)
(C) 3 different roots (D) atleast one negative root

17. If the quadratic equations ax2 + bx + c = 0 (a, b, c  R, a  0) and x2 + 4x + 5 = 0 have a


common root, then a, b, c must satisfy the relations:
(A) a > b > c (B) a < b < c
(C) a = k; b = 4k; c = 5k (k  R, k  0) (D) b2  4ac is negative.

18. If the quadratic equations x2 + abx + c = 0 and x2 + acx + b = 0 have a common root, then the
equation containing their other roots is/are :
(A) x2 + a (b + c) x  a2bc = 0 (B) x2  a (b + c) x + a2bc = 0
(C) a (b + c) x2  (b + c) x + abc = 0 (D) a (b + c) x2 + (b + c) x  abc = 0

19. Consider the following statements.


S1 : The equation 2x2 + 3x + 1 = 0 has irrational roots.
S2 : If a < b < c < d, then the roots of the equation (x – a) (x – c) + 2 (x – b) (x – d) = 0 are
real and distinct.
S3 : If x2 + 3x + 5 = 0 and ax2 + bx + c = 0 have a common root and a, b, c  N, then the
minimum value of (a + b + c) is 10.

S4 : The value of the biquadratic expression x4  8 x3 + 18 x2  8 x + 2, when x = 2 + 3 ,


is 1
Which of the following are CORRECT ?
(A) S2 and S4 are true. (B) S1 and S3 are false.
(C) S1 and S2 are true. (D) S3 and S4 are false.

For Unacademy Subscription Use “PJLIVE” Code | Join t.me/pjsir42 for Updates
For More Info: 75970-84242, 95490-43333/2222
20. If the equations x2 + a x + 12 = 0, x2 + b x + 15 = 0 & x2 + (a + b) x + 36 = 0 have a common
positive root, then which of the following are true ?

(A) ab = 56 (B) common positive root is 3


(C) sum of uncommon roots is 21. (D) a + b = 15.

21. If x2 + x + 1 = 0,  (–2 , 2) and 4x3 + 3x + 2c = 0 have common root then c +  can be
1 1 3
(A) (B) – (C) 0 (D)
2 2 2

For Unacademy Subscription Use “PJLIVE” Code | Join t.me/pjsir42 for Updates
For More Info: 75970-84242, 95490-43333/2222
Answer Key
ONE OR MORE THAN ONE OPTIONS CORRECT TYPE
1. (A) 2. (B), (C), (D) 3. (B), (C) 4. (A), (C)
5. (B), (C) ,(D) 6. (B), (C) 7. (A), (B), (D) 8. (A), (B), (C), (D)
9. (A), (B), (C), (D) 10. (A), (D) 11. (A), (D) 12. (A), (D)
13. (A), (B), (D) 14. (A), (D) 15. (A), (C) 16. (A), (B)
17. (C), (D) 18. (B), (D) 19. (A), (B) 20. (A), (B), (C)
21. (A), (B)

Solution
ONE OR MORE THAN ONE OPTIONS CORRECT TYPE
1. p = 0  2x2 – 4x – 0 = 0 two roots (A)
2
p = 1  0x – (0)x + 0 = 0 identity more than two roots (B)
2
p = 2  0x – (–2)x + (–2) = 0 x=+1 one root (C)
2
p = 4  6x – 0x – 12 = 0 two root (D)

2. (A) S = 2 + 2 = a2 – 2b ; P = 2 2 = b2  equation is x2 – (a2


– 2b) x + b2 = 0
1 1 a 1 1 1 a 1
(B) S= + =– ,P= . =  x2 + x+ = 0
  b   b b b
 bx2 + ax + 1 = 0
   2  2 a2  2b  
(C) S= + = = ; P= . =1
   b  
a2  2b
x2 – x+1=0   bx2 – (a2 – 2b) x + b = 0
b
(D) S=+–2=–a–2 ; P = ( – 1) ( – 1)
=  – ( + ) + 1 = b + a + 1 
   equation is x2 + (a + 2)x + (a + b + 1) = 0.

b c
3. ax2 + bx + c = 0 +  =  ,  = Ax2 + Bx + C = 0
a a

B C
( + ) + ( + ) = – , ( + ) ( + ) =  |(+ ) – ( + )| = |( – )|
A A

B2 4C b 2 4c b2  4ac B2  4AC
  =   = Hence proved
A2 A a2 a a2 A2

4. 4x2 + 2x – 1 = 0 

  42 + 2 – 1 = 0 ....(1)
Let 3
 = 4 – 3
with the help of equation (1)
(1  2 )
 =  [42 – 3] = [1 – 2 – 3] = – 22 – 2 = –2 –2 [using (1)]
4
 = – – 1/2
 +  = – 1/2 which is given. hence second root is 43 – 3.

For Unacademy Subscription Use “PJLIVE” Code | Join t.me/pjsir42 for Updates
For More Info: 75970-84242, 95490-43333/2222
5. x2 + 3x + 1 = (x – ) (x – ). Put x = 2  11 = (2 – ) (2 – ) (B)
2
 + 3 + 1 = 0, 2
 + 3 + 1 = 0
2
 = – (3 + 1), 2 = – (3 + 1)
2 2 2 2
= – 1, = –1  + =–2 (C)
3  1 3  1 3  1 3  1
2 2
      2 2 (3  1) (3  1)  (3  1)  (3  1)
  +  = + = + =
 1    1   1  2   2
1  2   2   

=
3( 2  2 )  (  )
=
 2
3 (   )  2  ( 3)  = 3 (7) – 3 = 18.
1 1

6. Split 32 into sum of two primes 32 = 2 + 30 = 3 + 29 = 5 + 27 = 7 + 25 = 11 + 21 = 13 + 19.


32 = 2 + 30 = 3 + 29 = 5 + 27 = 7 + 25 = 11 + 21 = 13 + 19.

7. 2 – a( + 1) – b = 0 .......(i)
2 – a( + 1) – b = 0 .......(ii)
by (i) & (ii)
1 1 2 1 1 2
(A)   =   0 (hence A)
2
  a 2
  a ab ab ab ab
(B) f(a) + a + b = –(a + b) + (a + b) = 0 (hence B)
f(b) + a + b = b2 – ab – a – b  0
 a  a2 a2 a  a2
(D) f  ab=  a   1  b  ab  0
2 4 4 2  4

8. Let f(x) = x3 + bx2 + cx + d


b+c+d=0 ......(i)
4b + 2c + d = –4 ......(ii)
9b + 3c + d = –18 ......(iii)
by (i), (ii) and (iii)
b = –5, c = 11, d = –6
 f(x) = x3 – 5x2 + 11x – 6
Alter : f(x) = (x – 1)(x – 2)(x – 3) + x2 = x3 – 5x2 + 11x – 6 = x3 – (x – 1)(5x – 6)
 f(4) = (3)(2)(1) + 16 = 22
3
6 6 
f     Now f(x) = x3  x = 1 or
5 5 5
f(0) f(1) = (–6)(1) < 0
one root in (0, 1)

9. Case-I (i) x>1


p(x) = x25 (x7 – 1) + x11(x7 – 1) + x3(x – 1) + 1
p(x) > 0 no root for x  (1, )
(ii) 0<x<1
p(x) = x32 + x18 (1 – x7) + x4 (x – x7) + (1 – x3)
p(x) > 0 not root for (0, 1)
(iii) x = 1 ; P(x) = 1
hence no real root for x > 0
Case-II : for x < 0
let x = – is root ( > 0)
For Unacademy Subscription Use “PJLIVE” Code | Join t.me/pjsir42 for Updates
For More Info: 75970-84242, 95490-43333/2222
p() = 32 + 25 + 18 + 11 + 4 + 3 + 1
p()  0
Hence no negative root
All roots are imaginary
p(x) + p(–x) = 2(x32 + x18 + x4 + 1)  0 x  R
Hence imaginary roots.


10. x2 + px + q = 0  +  = – p,  = q and p2 – 4q > 0  x2 – rx + s = .....(1)

Now  4 + 4 = r  4 + 4 = r , ()4 = s = q4  (2 +2)2 – 2()2 = r 


  [( + )2 – 2]2 – 222 = r (p2 – 2q)2 – 2q2 = r
(p2 – 2q)2 = 2q2 + r > 0 .....(2)
Now, for x2 – 4qx + 2q2 – r = 0
D = 16q2 – 4(2q2 – r) by equation (2) = 8q2 + 4r = 4(2q2 + r) > 0
D > 0 two real and distinct roots
Product of roots = 2q2 – r = 2q2 – [(p2 – 2q)2 – 2q2] = 4q2 – (p2 – 2q)2 = – p2 (p2 – 4q) < 0
from (1)
So product of roots is – ve. hence roots are opposite in sign

11. ax3 + bx2 + cx + d = 0

Let ax3 + bx2 + cx + d  (x2 + x + 1) (Ax + B)


Roots of x2 + x + 1 = 0 are imaginary, Let these are , 
So the third root '' will be real.
b b ab
++=  –1 +  = =
a a a
d d
Also  = . But  = 1  =  Ans are (A) & (D).
a a

12. If – 5 + i is a root then other root is – 5 – i and  = 0


 roots are – 5 + i, – 5 – i, – 5
Product of roots (25 + 2) (– 5) = – 860 ; 25 + 2 = 172 ; 2 = 147; = ± 7 3
 roots are – 5 + 7i 3 , – 5 – 7i 3 , – 5
and c = – 5(– 5 + 7i) – 5 (– 5 – 7i 3 ) + (– 5 + 7i 3 ) (– 5 – 7i 3 )
c = 50 + (250 + 147) = 222.

13. f(x) > 0 x  R or f(x) < 0 x  R hence D < 0


its graph can be

(A) f(1) > 0 graph (i) will be possible


so f(x) > 0 x  R
(B) f(–1) < 0 graph (ii) will be possible so f(x) < 0 x  R
 1
(C) f   > 0 so f(x) < 0 x  R
 2

For Unacademy Subscription Use “PJLIVE” Code | Join t.me/pjsir42 for Updates
For More Info: 75970-84242, 95490-43333/2222
so not possible
(D) a > 0 c > 0 (graph (i))
a < 0 c < 0 (graph (ii))
in both cases ac > 0

14. f() = f() = f() = 0


hence f(x) has three real roots  possible graphs of f(x) are

or

 (x1 x2), (x2 x3) and (x3 x4) or  (x1 x3), (x2 x3) and (x2 x4)
hence A and D are correct
B is wrong as  (x3, x4)
C is wrong as  (x1, x2)

15. only A and C are correct as in these graphs


f() = f() = f() = f'(x1) = f'(x2) = 0
In option B f() < 0 and f() > 0 (can't be equal).
In option D f() > 0 and f() < 0 (can't be equal).

3 4 5  f(2 )   
16. f(x) = + +  f(x) = 0 has exactly one root in (2, 3).
x2 x 3 x4 and f(3  )   
 f(3  )   
again    f(x) = 0 has exactly one root in (3, 4).
and f(4  )   

17.  D of x2 + 4x + 5 = 0 is less than zero


 both the roots are imaginary  both the roots of quadratic are same
a b c
 b2 – 4ac < 0 & = = = k   a = k, b = 4k, c = 5k.
1 4 5


18. x2 + abx + c = 0 ...(1)  +  = – ab,  = c

x2 + acx + b = ...(2)  +  = – ac,   = b


2 + ab  + c = 0
2 + ac  + b = 0
2  1 a (b2 – c 2 )
= = 2 = = – (b + c)
ab2 – ac 2 c–b a c – ab a(c – b)
c –b 1 1
& = =  common root,  =
a(c – b) a a
1
 – (b + c) =  a2 (b + c) = – 1
a2
Product of the roots of equation (1) & (2) gives
1 1
× = c   = ac & × = b   = ab.
a a
 equation having roots ,  is

For Unacademy Subscription Use “PJLIVE” Code | Join t.me/pjsir42 for Updates
For More Info: 75970-84242, 95490-43333/2222
x2 – a (b + c) x + a2bc = 0  a (b + c) x2 – a2 ( b + c)2 x + a.(b + c) a2bc = 0
a (b + c) x2 + (b + c) x – abc = 0.

19. S1 : 2x2 + 3x + 1 = 0
 D=9–4×1×1=1
Which is perfect square of a rational number
 roots will be rational.
S2 :  Let f(x) = (x – a)(x – c) + 2(x – b) (x – d)
 f(a) > 0
f(b) < 0
f(c) < 0
f(d) > 0
 two real and distinct roots.
S3 : x2 + 3x + 5 = 0 .....(i) and
ax2 + bx + c = 0 .....(ii)
for equation (i) , D < 0
 Roots are imaginary and they occur in conjugate pair
 Roots of equation (i) and (ii) will be identical
a b c
   = , (N)  a =, b = 3, c = 5  a + b + c = 9
1 3 5
 least value is 9.

20. x2 + ax + 12 = 0 .....(1)
x2 + bx + 15 = 0 .....(2)
x2 + (a + b) x + 36 = 0 .....(3)
(1) + (2) – (3) gives x2 – 9 = 0  x = ± 3 given that common root will be +ve
so x = 3 put in equation (3) 9 + 3 (a + b) + 36 = 0  a + b = – 15
by equation (1) 9 + 3a + 12 = 0  a = – 7 & b = – 8

21. 4x3 + 3x + 2c = (4x + 2c)(x2 + x + 1)


comparing co-efficients
1 1
 c = 1 and  = – or c = –1 and  = 
2 2
1 1
  c+= or –
2 2

For Unacademy Subscription Use “PJLIVE” Code | Join t.me/pjsir42 for Updates
For More Info: 75970-84242, 95490-43333/2222
Title: Quadratic Equation
Chapter: Quadratic Equation
Approximate Time to Complete (to be filled by student): ________________________
Total Marks Scored:
Worksheet Compiled By: Prashant Jain (PJ Sir)

 Attempt the worksheet in one go. See answers in one go at the end.
 All questions carry 4 marks for positive and -1 if you leave and -2 if you attempt
incorrectly.
 All proving or show questions (if done correctly) are of 4 marks. There is no negative
marking.
 If you cheat in worksheet then you are only cheating and such sinners do not get
selection in JEE so for your own sake refrain from cheating.
 Maintain the solutions of this worksheet and share the link of solution pdf in the
tracker if you want to be monitored.

Exercise – I
COMPREHENSION
Comprehension # 1 (Q. No. 1 & 2)
If x, y  R then some problems can be solved by direct observing extreme cases
e.g. (i) (x – 3)2 + (y – 2)2 = 0 is possible only for x = 3 and y = 2
(ii) if x  3, y  2 and xy  6 then x = 3 & y = 2
1. The least value of expression x2 + 2 xy + 2 y2 + 4 y + 7 is :
(A) 1 (B) 2 (C) 3 (D) 4

2. Let P(x) = 4x2 + 6x + 4 and Q(y) = 4y2 – 12y + 25. If x, y satisfy equation P(x).Q(y) = 28, then
the value of 11y – 26x is -
(A) 6 (B) 36 (C) 8 (D) 42

Comprehension # 2 (Q. No. 3 & 4)


In the given figure OBC is an isosceles right triangle in which AC is a median, then answer
the following questions:
Y

y = x2 + bx + c
C

O A B X

3. Roots of y = 0 are
(A) {2, 1} (B) {4, 2} (C) {1, 1/2} (D) {8, 4}

4. The equation whose roots are ( + ) & ( – ), where , ( > ) are roots obtained in previous
question, is
(A) x2 – 4x + 3 = 0 (B) x2 – 8x + 12 = 0 (C) 4x2 – 8x + 3 = 0 (D) x2 – 16x + 48 = 0

For Unacademy Subscription Use “PJLIVE” Code | Join t.me/pjsir42 for Updates
For More Info: 75970-84242, 95490-43333/2222
Comprehension # 3 (Q. No. 5 to 7)
Consider the equation x4 – x2 + 9 = 0. This can be solved by substituting x2 = t such equations
are called as pseudo quadratic equations.
5. If the equation has four real and distinct roots, then  lies in the interval
(A) (–, –6)  (6, ) (B) (0, ) (C) (6, ) (D) (–, –6)

6. If the equation has no real root, then  lies in the interval


(A) (–, 0) (B) (–, 6) (C) (6, ) (D) (0, )

7. If the equation has only two real roots, then set of values of  is
(A) (–, –6) (B) (–6, 6) (C) {6} (D) 

Comprehension # 4
To solve equation of type,
ax2m + bx2m – 1 + cx2m – 2 + ......... + kxm + ......... + cx2 + bx + a = 0,(a  0)  ()
divide by xm and rearrange terms to obtain
 1 
b  xm1  m1  + c  xm2  m2  + ......... + k = 0
1 1
a  xm  m  +
 x   x   x 
Substitutions like
1 1
t=x+ or t=x– helps transforming equation into a reduced degree equation.
x x

8. Roots of equation x4 – 10x3 + 26x2 – 10x + 1 = 0 are

(A) 2 ± 3 , 3 ± 2 (B) 2 ± 3 , 3 ± 2 2 (C) 3 ± 2 , 3 ± 2 2 (D) 8 ± 3 , 3 ± 2

9. Roots of equation x5 – 5x4 + 9x3 – 9x2 + 5x – 1 = 0 are


3 5 1 i 3 5 3 3i
(A) 1, , (B) 1, ,
2 2 2 2
3 5 3i 5 3 1 i 3
(C) 1, , (D) 1, ,
2 2 2 2

10. Roots of equation x6 – 4x4 + 4x2 – 1 = 0 are

1 i 5 1  5 1 5 1  i 5
(A) ± 1, , (B) ± 1, ,
2 2 2 2
1  5 1  5 1  5 1  i 5
(C) ± 1, , (D) ± 1, , .
2 2 2 2

For Unacademy Subscription Use “PJLIVE” Code | Join t.me/pjsir42 for Updates
For More Info: 75970-84242, 95490-43333/2222
Answer Key
COMPREHENSION
1. (C) 2. (B) 3. (A) 4. (A) 5. (C)
6. (B) 7. (D) 8. (B) 9. (A) 10. (C)

Solution
COMPREHENSION
Comprehension # 1 (Q. No. 1 & 2)
1. x2 + 2xy + 2y2 + 4y + 7 = (x + y)2 + (y + 2)2 + 3  0 + 0 + 3  Least value = 3.

2
  3 7 7
2. P(x) = 4x2 + 6x + 4 = 4  x     P(x) = 
 4  4 4
2
Q(y) = 4y2 – 12y + 25 = 4  y    16
3
 Q(y)  16
 2
p(x).Q(y)  28 but it is given P(x).Q(y) = 28 p(x).Q(y)  28
7
 P(x) = & Q(y) = 16
4
3 3 3 ( 3) 33 39 72
 x= , y= ; 11y – 26x = 11 × – 26 =  = = 36. Ans.
4 2 2 4 2 2 2

Comprehension # 2 (Q. No. 3 & 4)


(3 & 4)
Let the coordinates of A(, 0), B(2, 0), C(0, 2). Now y = x2 + bx + c passes through C(0,
2)
 given equation of curve reduces to y = x2 + bx + 2Now it also passes through A &
B
 0 = 2 + b + 2  0 =  + b + 2 ..... (i)
& 2
0 = 4 + 2b + 2  0 = 2 + b + 1 ..... (ii)
On solving (i) & (ii) for  & b we get  = 1, b = – 3
 given curve is y = x2 – 3x + 2
3. roots of y = 0 are {2, 1}
4. ( + )  3 (  = 2,  = 1)   – )  1
 equation whose roots are 3, 1 is x2 – 4x + 3 = 0

Comprehension # 3 (Q. No. 5 to 7)


(5 to 7)
x4 – x2 + 9 = 0  x2 = t 0  f(t) = t2 – t + 9 = 0

5. given equation has four real & distinct roots


D > 0  2 – 36 > 0
–b 
>0     > 0
2a 2
f (0) > 0    9>0
  (6,)

For Unacademy Subscription Use “PJLIVE” Code | Join t.me/pjsir42 for Updates
For More Info: 75970-84242, 95490-43333/2222

6. Equation has no real roots.


case-I D 0 2 – 36     
–b
     < 0
2a
f (0) > 0    9 > 0.
   (– , – 6]
case-II D < 0 
  2 – 36 < 0   (– 6, 6)
union of both cases gives (-I)  (-II)
(– , 6) 

7. Equation has only two real roots


case-I f (0) < 0 9<0
which is false
case-II f (0) = 0
–b
and < 0
2a
 No solution
 Final answer is 

Comprehension # 4
+ 2 = 0 x2 + 2 – 10  x   + 26 = 0
10 1 1 1
8. Divide by x2 x2 – 10x + 26 –
x x x  x
1 1
t=x+  t2 – 2 = x 2 + 2 t2 – 2 – 10t + 26 = 0  t2 – 10t + 24 = 0
x x
1
t=4 x+ =4  x2 – 4x + 1 = 0  x=2± 3
x
1
t=6 x+ =6  x2 – 6x + 1 = 0  x=3±2 2.
x

9. By trail x = 1 is a root divide by x – 1


1 5 9 9 5 1
1
 1 4 5 4 1
1 4 5 4 1 0
(x – 1) (x4 – 4x3 + 5x2 – 4x + 1) = 0  x = 1 or x4 – 4x3 + 5x2 – 4x + 1 = 0
4 1 1 1
x2 – 4x + 5 –  2=0  t=x+  t2 = x2 + +2
x x x x2

For Unacademy Subscription Use “PJLIVE” Code | Join t.me/pjsir42 for Updates
For More Info: 75970-84242, 95490-43333/2222
1 1
t2 – 2 – 4t + 5 = 0  t2 – 4t + 3 = 0  x+ = 1, x + =3
x x
1 i 3 3 5
x2 – x + 1 = 0, x2 – 3x + 1 = 0  x= ,x=
2 2
1 i 3 3 5
 roots are 1, , .
2 2

4 1 1  1
10. Divide by x3   x3 – 4x + – = 0; x3 – – 4 x  = 0
x x3 x3  x 

t3 = x3 – 3x2 + 3x 2 – 3 = x3 – 3  x   – 3
1 1 1 1 1 1
Put t=x– 
x x x x  x x
1
t3 + 3t = x3 –
x3
Put in equation above t3 + 3t – 4t= 0 t3 – t = 0  t = 0, 1, –1
1 1 1
x– = 0, x– = 1, x– = –1 ; x = ± 1, x2 – x – 1 = 0,
x x x
x2 + x – 1 = 0
1 5 1  5
x = ± 1, x= , x= .
2 2

For Unacademy Subscription Use “PJLIVE” Code | Join t.me/pjsir42 for Updates
For More Info: 75970-84242, 95490-43333/2222
Title: Quadratic Equation
Chapter: Quadratic Equation
Approximate Time to Complete (to be filled by student): ________________________
Total Marks Scored:
Worksheet Compiled By: Prashant Jain (PJ Sir)

 Attempt the worksheet in one go. See answers in one go at the end.
 All questions carry 4 marks for positive and -1 if you leave and -2 if you attempt
incorrectly.
 All proving or show questions (if done correctly) are of 4 marks. There is no negative
marking.
 If you cheat in worksheet then you are only cheating and such sinners do not get
selection in JEE so for your own sake refrain from cheating.
 Maintain the solutions of this worksheet and share the link of solution pdf in the
tracker if you want to be monitored.

Exercise – I
* Marked Questions may have more than one correct option.
1. The smallest value of k, for which both the roots of the equation x2 – 8kx + 16(k2 – k + 1) = 0
are real, distinct and have values atleast 4, is

2. Let p and q be real numbers such that p  0, p3  q and p3  – q. If  and  are nonzero complex
 
numbers satisfying  +  = – p and 3 + 3 = q, then a quadratic equation having and as
 
its roots is
(A) (p3 + q) x2 – (p3 + 2q)x + (p3 + q) = 0 (B) (p3 + q) x2 – (p3 – 2q)x + (p3 + q) = 0

(C) (p3 – q) x2 – (5p3 – 2q)x + (p3 – q) = 0 (D) (p3 – q) x2 – (5p3 + 2q)x + (p3 – q) =
0

3. Let  and  be the roots of x2 – 6x – 2 = 0, with  >  . If an = n – n for n  1, then the value
a10  2a8
of is
2a9
(A) 1 (B) 2 (C) 3 (D) 4

4. A value of b for which the equations


x2 + bx – 1 = 0
x2 + x + b = 0
have one root in common is

(A) – 2 (B) – i 3 (C) i 5 (D) 2

5. The quadratic equation p(x) = 0 with real coefficients has purely imaginary roots. Then the
equation p(p(x)) = 0 has

For Unacademy Subscription Use “PJLIVE” Code | Join t.me/pjsir42 for Updates
For More Info: 75970-84242, 95490-43333/2222
(A) only purely imaginary roots (B) all real roots
(C) two real and two purely imaginary roots (D) neither real nor purely imaginary roots

6. Let S be the set of all non-zero real numbers  such that the quadratic equation x2 – x +  = 0
has two distinct real roots x1 and x2 satisfying the inequality |x1 – x2| < 1. Which of the
following intervals is(are) a subset(s) of S ?

 1 1   1   1   1 1
(A)  – , –  (B)  – , 0 (C)  0,  (D)  , 
 2 5  5   5  5 2

 
7. Let – <  < – . Suppose 1 and 1 are the roots of the equation x2 – 2x sec  + 1 = 0 and
6 12
2 and  are the roots of the equation x2 + 2x tan – 1 = 0. If 1 > 1 and 2 > 2 , then 1
+ 2 equals
(A) 2(sec – tan ) (B) 2sec  (C) – 2tan  (D) 0

Comprehension (Q-8 & 9)


Let p, q be integers and let , be the roots of the equation, x2 – x – 1 = 0 where   . For n
= 0,1,2,...., let an = pn + qn.
FACT : If a and b are rational numbers and a + b 5 = 0, then a = 0 = b.
8. a12 =
(A) a11 + 2a10 (B) 2a11 + a10 (C) a11 – a10 (D) a11 + a10

9. If a4 = 28, then p + 2q =
(A) 14 (B) 7 (C) 21 (D) 12

For Unacademy Subscription Use “PJLIVE” Code | Join t.me/pjsir42 for Updates
For More Info: 75970-84242, 95490-43333/2222
Answer Key
1. 2 2. (B) 3. (C) 4. (B) 5. (D)
6. (A, D) 7. (C) 8. (D) 9. (D)

Solution
1. (i) x2 – 8kx + 16(k2 – k + 1) = 0 D = 64 (k2 – (k2 – k + 1)) = 64 (k – 1) > 0

 k>1 .......(1)
b 8k
(ii) – >4  >4  k > 1 .......(2)
2a 2
(iii) f(4)  0
  16 – 32k + 16 (k2 – k + 1)  0   k2 – 3k + 2  0
  (k – 2) (k – 1)  0  k  1 or k  2 .......(3)
(1)  (2)  (3). Hence k = 2

2. Product = 1
 2  2 (  )2  2
Sum = =
 
Since 3 + 3 = q  – p (2 + 2 – ) = q
2 q q
(( + ) – 3 ) = –  p2 + = 3
p p
 2 2  p  q  
3
p     3p
 3  p   p3  2q
Hence sum = =
(p3  q) p3  q
 p3  2q 
so the equation is x2 –  3
 x+1=0
 p q 
 (p3 + q) x2 – (p3 – 2q)x + (p3 + q) = 0

3. x2 – 6x – 2 = 0 having roots and  2 – 6– 2 = 0


  – 6 – 28 = 0
10 9  10 – 28 = 69 .... (i)
10 8
similarly  – 2 = 6 9 .... (ii)
by (i) and (ii)
a10 – 2a8
(10 – 10) – 2(8 – 8) = 6 (9 – 9)  a10 – 2a8 = 6a9  =
2a9
3
Aliter
10  10  2(8  8 ) 10  10  (8  8 )  9 (    )  9 (    )  6
9 9
= 9 9
= 9 9
= = =3
2(   ) 2(   ) 2(   ) 2 2

x 2  bx  1  0
x2  x  b  0 b2  1 (b  1)
4.  x =   (b2+1)(1–b) =(b+1)2
x 2
x 1 (b  1) 1 b
 
b  1 1  b 1  b
2

For Unacademy Subscription Use “PJLIVE” Code | Join t.me/pjsir42 for Updates
For More Info: 75970-84242, 95490-43333/2222
 b2 –b3 + 1– b = b2 + 2b +1  b3 + 3b = 0  b = 0 ; b2 = –3  b = 0  3 i ,

5. p(x) will be of the form ax2 + c. Since it has purely imaginary roots only.
Since p(x) is zero at imaginary values while ax2 + c takes real value only at real 'x', no root is
real.
Also p(p(x)) = 0  p(x) is purely imaginary  ax2 + c = purely imaginary
Hence x can not be purely imaginary since x2 will be negative in that case and ax2 + c will be
real.
Thus .(D) is correct.

6. (x1 + x2)2 – 4x1x2 < 1


1
–4<1
2
1
 5– >0
2
5 2  1
>0
2

 1   1 
  ,   ,  ...(1)
 5  5 
D>0
1 – 42 > 0
 1 1
   ,  ...(2)
2 2 
(1) & (2)
 1 1   1 1 
   ,   , 
 2 5   5 2

7. x2 – 2xsec + 1 = 0
2sec   4 sec 2  – 4
x=  x = sec + tan , sec – tan  1 = sec – tan
2
–2 tan   4 tan2   4
now x2 + 2x tan– 1 = 0  x =
2
 x = – tan ± sec  2 = (sec – tan)  2 = – (sec + tan)
  1 + 2 = – 2 tan

8. As  and  are roots of equation x2 – x –1 = 0, we get :


– – 1 = 0  + 1
– – 1 = 0  + 1
a11 + a10 = p11 + q+ p+ q
= p10 (+1) + q10 ( +1)
= p10  + q 

For Unacademy Subscription Use “PJLIVE” Code | Join t.me/pjsir42 for Updates
For More Info: 75970-84242, 95490-43333/2222
= p12  q
 =a 
12

9. an+2 = an+1 + an
a4 = a3 + a2 = 3a1 + 2a0 = 3p+ 3q+ 2(p + q)
1 5 1 5
As   ,  , we get
2 2
 1 5   
a4 = 3p   3q 1  5  + 2p + 2q = 28
  2 
 2   
 3p 3q 
    2p  2q  28   0 …….(i)
 2 2 
3p 3q
and   0 …………….(ii)
2 2
p = q (from (ii))
7p = 28(from (i) and (ii))
p = 4
q = 4
p + 2q = 12

For Unacademy Subscription Use “PJLIVE” Code | Join t.me/pjsir42 for Updates
For More Info: 75970-84242, 95490-43333/2222
Title: Quadratic Equation
Chapter: Quadratic Equation
Approximate Time to Complete (to be filled by student): ________________________
Total Marks Scored:
Worksheet Compiled By: Prashant Jain (PJ Sir)

 Attempt the worksheet in one go. See answers in one go at the end.
 All questions carry 4 marks for positive and -1 if you leave and -2 if you attempt
incorrectly.
 All proving or show questions (if done correctly) are of 4 marks. There is no negative
marking.
 If you cheat in worksheet then you are only cheating and such sinners do not get
selection in JEE so for your own sake refrain from cheating.
 Maintain the solutions of this worksheet and share the link of solution pdf in the
tracker if you want to be monitored.

Exercise – I
1. Sachin and Rahul attempted to solve a quadratic equation. Sachin made a mistake in writing
down the constant term and ended up in roots (4, 3). Rahul made a mistake in writing down
coefficient of x to get roots (3, 2). The correct roots of equation are :
(A) 6, 1 (B) 4, 3 (C) –6 , –1 (D) –4 , –3

2. Let for a  a1  0, f(x) = ax2 + bx + c, g(x) = a1x2 + b1x + c1 and p(x) = f(x) – g(x). If
p(x) = 0 only for x = –1 and p(–2) = 2, then the value of p(2) is :
(A) 3 (B) 9 (C) 6 (D) 18

3. The equation esinx – e–sinx – 4 = 0 has :


(A) infinite number of real roots (B) no real roots
(C) exactly one real root (D) exactly four real roots

4. If the equations x2 + 2x + 3 = 0 and ax2 + bx + c = 0, a,b,c R, have a common root, then a : b
: c is
(A) 1 : 2 : 3 (B) 3 : 2 : 1 (C) 1 : 3 : 2 (D) 3 : 1 : 2

5. If a  R and the equation – 3(x – [x])2 + 2 (x – [x]) + a2 = 0 (where [x] denotes the greatest
integer  x) has no integral solution, then all possible values of a lie in the interval :
(A) (–2, –1) (B) (–, –2)  (2, ) (C) (–1, 0)  (0, 1) (D) (1, 2)

1 1
6. Let  and  be the roots of equation px2 + qx + r = 0, p  0. If p, q ,r are in the A.P. and 
 
= 4, then the value of | – | is :
34 2 13 61 2 17
(A) (B) (C) (D)
9 9 9 9

For Unacademy Subscription Use “PJLIVE” Code | Join t.me/pjsir42 for Updates
For More Info: 75970-84242, 95490-43333/2222
7. Let  and  be the roots of equation x2 – 6x – 2 = 0. If an = n – n, for n  1, then the value
a10 – 2a8
of is equal to :
2a9
(A) 6 (B) – 6 (C) 3 (D) –3

8. The number of all possible positive integral values of  for which the roots of the quadratic
equation, 6x2 – 11x +  = 0 are rational numbers is :
(A) 3 (B) 4 (C) 5 (D) 2

9. If  be the ratio of the roots of the quadratic equation in x, 3m2x2 + m(m – 4)x + 2 = 0, then the
1
least value of m for which  + = 1, is :

(A) –2 + 2 (B) 4 – 3 2 (C) 2 – 3 (D) 4 – 2 3

For Unacademy Subscription Use “PJLIVE” Code | Join t.me/pjsir42 for Updates
For More Info: 75970-84242, 95490-43333/2222
Answer Key
1. (A) 2. (D) 3. (B) 4. (A) 5. (C)
6. (B) 7. (C) 8. (A) 9. (B)

Solution
1. Let the correct equation be ax2 + bx + c = 0

now Sachin’s equation  ax2 + bx + c = 0

Rahul’s equation  ax2 + b x + c = 0


b c
– =7 ........ (i) =6 ........ (ii)
a a
from (i) and (ii)
correct equation is x2 – 7x + 6 = 0 roots are 6 and 1.

2. P(x) = 0  f(x) = g(x)  ax2 + bx + c = a1x2 + b1x + C,


 (a – a1) x2 + (b – b1) x + (c – c1) = 0.
It has only one solution x = – 1
 b – b 1 = a – a1 + c – c 1 .... (1)
b – b1
vertex (–1, 0)  =–1  b – b1 = 2(a – a1) .... (2)
2(a – a1 )
 f(–2) – g(–2) = 2  4a – 2b + c – 4a1 + 2b1 – c1 = 2
 4(a – a1) – 2(b – b1) + (c – c1) = 2 .... (3)
1
by (1), (2) and (3) (a – a1) = (c – c1) = (b – b1) = 2
2
Now P(2) = f(2) – g(2) = 4 (a – a1) + 2 (b – b1) + (c – c1) = 8 + 8 + 2 = 18

4  16  4
3. Let esinx = t  t2 – 4t – 1 = 0  t=  t = esinx = 2 ± 5
2
  esin x = 2 – 5 , esin x = 2 + 5   esin x 2 – 5 < 0, 
  sinx = ln(2 + 5 ) > 1 so rejected so rejected
hence no solution

4. x2 + 2x + 3 = 0 ...(i) ax2 + bx + c = 0 ...(ii)


Since equation (i) has imaginary roots.
So equation (ii) will also have both roots same as (i).
a b c
Thus    a = , b = 2, c = 3 Hence 1 : 2 : 3
1 2 3

5. a2 = 3{x}2 – 2{x} [ x – [x] = {x}]


Let {x} = t  t  (0, 1) As x is not an integer
 2  2
 a2 = 3t2 – 2t f(t) = 3t  t    a2 = 3t  t  
 3  3 
Clearly by graph

For Unacademy Subscription Use “PJLIVE” Code | Join t.me/pjsir42 for Updates
For More Info: 75970-84242, 95490-43333/2222
2
  a2  1
3
 a  (–1, 1) – {0} (As x  integer) Ans. (3)
Note : It should have been given that the solution exists else answer will be a  R – {0}

6. px2 + qx + r = 0 ; p, q, r  A.P. ; 2q = p + r
1 1   q
 = 4 ; = 4  =4
   r
q = – 4r ..... (i)
 –8r = p + r
p = – 9r ..... (ii)
q2 4r
|–| = (   )2  4 = 2
 by (i) and (ii)
p p

q2  4pr 16r 2  36r 2 2 13


= = =
|p| | 9r | 9

7. x2 – 6x – 2 = 0
an = n – n
a10  2a8 10  10  2(8  8 )

2a9 2( 9  9 )
8 ( 2  2)  8 (2  2) 69  69
= =
2(9  9 ) 2( 9  9 )
 6
= =3 Ans. (3)
2 2

8. For rational roots D must be perfect square


D = 121 – 24 = k2
for 121 – 24 to be perfect square  must be equal to 3, 4, 5 (observation) so number of possible
values of  is 3.

9. Let roots are  & now


  
 1    1 2 + 2 = 
  

(+)2 = 3
2
 m(m  4)  2
 3m2   3. 3m2
 
m2 – 8m –2 = 0
m=4±3 2
so least value of m = 4 – 3 2
For Unacademy Subscription Use “PJLIVE” Code | Join t.me/pjsir42 for Updates
For More Info: 75970-84242, 95490-43333/2222



    

     
=    
     



 
 

 

− −


= 

 
= 

 

 


  
=   = 
 
 
 
− = =
 

  
=   = 
   


=

 
= 
  
 
 
= 
 
 
 
= 
 
 
 
=  − 
 
 

=
= − = + +
− − − −

+ − + +

− = =
+ =

  

+ =
+ =
+ =
+ + =

= +

 

+ +  − + 

+ −  + + 

+ =

− − − −
− − − −
− −

+ =

 − −   
 
 

+ = +
NSEP Vector Assignment-2
Time : 60 Minutes
Single Correct Type:

1. A particle passes a point moving in north direction with uniform velocity 2 3 m/s. After some time
it turns in the east and moves with uniform velocity 6 m/s. Direction of change in its velocity
vector is
(A) 30° east of north (B) 60° east of north
(C) 60° south of east (D) 30° south of east

2. A vector of magnitude 10 units has initial point at (4, 3). Terminal point of its x-component, when
drawn from the initial point of the vector is (–4, 3). What is the terminal point of the vector?
(A) (–4, –3) (B) (–4, 3) (C) (4, –3) (D) (8, 0)

3. A co-ordinate system consisting of x-y axis, is rotated by an angle  in anticlockwise direction in


the same plane. The unit vector along new set of axes x̂ , and ŷ , are respectively,

(A) cos ˆi & sin ˆj (B) cos ˆi + sin ˆj & − sin ˆi + cos ˆj

(C) cos ˆi + sin ˆj & sin ˆi + cos ˆj (D) sin ˆi + cos ˆj & cos ˆi + sin ˆj

4. A vector is turned 45° along a plane & then 45° in a plane perpendicular to its initial plane. The
angle between initial & final vectors is
(A) 15° (B) 45° (C) 60° (D) 90°

5. Two forces f1 = 2N and f2 = 3N are acting on a particle along x = 0 and y = 0 lines, respectively. The
resultant force on the particle will be–
(A) 2î + 3 ĵ (B) 3î + 2 ĵ (C) 2î − 3 ĵ (D) −2î + 3jˆ

Multiple Correct Type:

6. The value of | A + B − C + D | can be zero, if :

(A) | A |= 5,| B |= 3,| C |= 4;| D |= 13 (B) | A |= 2 2,| B |= 2,| C |= 2;| D |= 5

(C) | A |= 2 2,| B |= 2,| C |= 2;| D |= 10 (D) | A |= 5,| B |= 4,| C |= 3;| D |= 8

Space for Rough Work

1
7. Four vectors (A,B,C,D) all have the same magnitude and lie in a plane. The angle
between adjacent vectors is 45° as shown. Which of the following equation
is/are correct ?

(A) A − C = − 2D (B) B + D − 2 C = 0

(C) A + B = B + D (D) (A + C) / 2 = B

8. The angle between two vectors a & b is  and the magnitude of b is half of magnitude of a . If
c = a − b & | a |= a , then choose the correct statements

a 5 a 3
(A) if c = then  will be 90°. (B) if c = then  will be 60°.
2 2
a 3a
(C) if c = then  will be 45° (D) if c = then  will be 180°
2 2

9. A vector A of magnitude 3 units is added to another vector B which has magnitude 6 units but
has random direction anywhere in the same plane. When B has such a direction that angle made
by resultant with B is largest,
(A) magnitude of resultant of A and B is 3 3 units.
(B) angle between A and B is 120°
(C) angle between B and resultant is 30°
(D) angle between A & resultant is 60°.

10. A body acted upon by 3 given forces is under equilibrium as shown in figure. If F1 = 10 N and

F2 = 6N, direction of force F3 is along –Y axis, then which of the following is/are CORRECT :

(A) Magnitude of F3 is 136 N (B) Angle between F1 and F2 is 127°

(C) F2 is –6N î (D) F1 + F2 + F3 is a zero vector

Space for Rough Work

2
11. Given a + b + c + d = 0 , which of the following statements is/are correct :

(A) a,b,c and d must each be a zero vector

(B) The magnitude of ( a + c ) equals to the magnitude of ( b + d ).

(C) The magnitude of a can never be greater than the sum of the magnitudes of b, c and d .

(D) b + c must lie in the plane of a and d if a and d are not collinear, and in the line of a and d
if they are collinear

12. The following forces act on an object at some instant of time. F1 = ( −10iˆ + 2ˆj)N; F2 = (4iˆ + 6ˆj)N . Which
of the following statements is/are true for the net force on the object?

(A) Unit vector in the direction of the net force is (−0.6iˆ + 0.8j)N
ˆ .

(B) Unit vector in the direction of the net force is (−0.6iˆ + 0.8j)
ˆ .

(C) Angle of the net force vector with the positive x-axis is 127°.
(D) Angle of the net force vector with the positive y-axis is 37°.

Linked Comprehension Type


(Single Correct Answer Type)
Paragraph for Question 13 to 15
A boy A starts from a point P runs some distance towards east then turns 53° towards north and
runs 75 m further to reach point Q. The boy maintains constant speed of 5 m/s in running from P
to Q. Another boy B starts 2 s after A from point P and runs 100 m in a direction 37° north of east
with a constant speed. Both of them meet at point Q.

13. How far in the east direction, has the boy A ran?
(A) 25 m (B) 30 m (C) 35 m (D) 40 m

14. How long the boy A has to run to reach point Q?


(A) 20 s (B) 22 s (C) 24 s (D) 25 s

15. Magnitude of average velocity of the boy A is closest to :


(A) 5 m/s (B) 4.45 m/s (C) 4.54 m/s (D) 3.75 m/s

Space for Rough Work

3
Paragraph for Question 16 to 18
If two vectors are represented by two adjacent sides of a parallelogram which are directed away
from their common point then their sum (i.e. resultant vector) is given by the diagonal of the
parallelogram passing away through that common point. On the basis of above theory , answer the
following questions.

16. If two vectors of magnitude of 5 and 3 are added such that angle between resultant and vector of
magnitude 5 is maximum then the angle will be :
(A) 37° (B) 53° (C) 90° (D) 180°

17. If two vectors of magnitude of 5 and 3 are added such that angle between resultant and vector of
magnitude 3 is maximum then the angle will be :
(A) 37° (B) 53° (C) 90° (D) 180°

18. A vector A of unknown magnitude makes 127° or 37° with another vector of magnitude 5. What is
the minimum possible magnitude of resultant vector?
(A) 3 or 5 (B) 4 or 5 (C) 0 or 3 (D) Data insufficient

Numerical Type :

19. A train moving with a constant speed of 60 mile/h moves eastward for 40 min, then in a direction
45° east of north for 20 min, and finally westward for 50 min. What is the magnitude of average
velocity of the train during this run in mile/h. (closest integer)

Space for Rough Work

4
9 1 .
Find
dy at t =
2 see .

af


y E
:

③ 3th
y
=


y =
① I
y
=

3/2
+

& Find
.
2
& for


y =
=

③ Y
% E 2
:
+

② 3x 2n(u) Sek
y
=
+ -

① = 2n5 3x + Usi(x) -

2cs(x)
y
-
Assignment - 01

1. A Body moves 6 m north. 8 m east and 10m vertically upwards, what is its resultant displacement
from initial position
10
(A) 10 2m (B) 10m (C) m (D) 10  2m
2
2. A person moves 30 m north and then 20 m towards east and finally 30 2 m in south-west
direction. The displacement of the person from the origin will be:
(A) 10 m along north (B) 10 m long south
(C) 10 m along west (D) Zero

3. An aeroplane flies 400 m north and 300 m south and then flies 1200 m upwards then net
displacement is:
(A) 1200 m (B) 1300 m (C) 1400 m (D) 1500 m

4. An athlete completes one round of a circular track of radius R in 40 sec. What will be his
displacement at the end of 2 min. 20 sec.
(A) Zero (B) 2R (C) 2R (D) 7R

5. A person travels along a straight road for half the distance with velocity v 1 and the remaining half
distance with velocity v2 The average velocity is given by

v22 v 1 + v2 2v 1v2
(A) v1v2 (B) (C) (D)
v21 2 v 1 + v2

6. A boy walks to his school at a distance of 6 km with constant speed of 2.5 km/hour and walks
back with a constant speed of 4 km/hr. His average speed for round trip expressed in km/hour, is:
(A) 24/13 (B) 40/13 (C) 3 (D) ½

7. A car travels the first half of a distance between two places at a speed of 30 km/hr and the second
half of the distance at 50 km/hr. The average speed of the car for the whole journey is:
(A) 42.5 km/hr (B) 40.0 km/hr (C) 37.5 km/hr (D) 35.0 km/hr

8. A train has a speed of 60 km/h. for the first one hour and 40 km/h for the next half hour. Its
average speed in km/h is:
(A) 50 (B) 53.33 (C) 48 (D) 70

9. A 150 m long train is moving with a uniform velocity of 45 km/h. The time taken by the train to
cross a bridge of length 850 meters is:
(A) 56 sec (B) 68 sec (C) 80 sec (D) 92 sec

Space for Rough Work

1
10. A particle is constrained to move on a straight line path. It returns to the starting point after
10 sec. The total distance covered by the particle during this time is 30 m. Which of the following
statements about the motion of the particle is false
(A) Displacement of the particle is zero (B) Average speed of the particle is 3 m/s
(C) Displacement of the particle is 30 m (D) Both (A) and (B)

11. A particle moves along a semicircle of radius 10m in 5 seconds. The average velocity of the particle
is:
(A) 2 ms−1 (B) 4 ms−1 (C) 2 ms−1 (D) 4 ms−1

12. A person travels along a straight road for the first half time with a velocity v 1 and the next half
time with a velocity v2 . The mean velocity V of the man is:

2 1 1 v 1 + v2 v1
(A) = + (B) V = (C) V = v 1v2 (D) V =
V v 1 v2 2 v2

13. If a car covers 2/5th of the total distance with v1 speed and 3/5th distance with v2 then average
speed is:
1 v 1 + v2 2v 1v2 5v 1v 2
(A) v 1v 2 (B) (C) (D)
2 2 v 1 + v2 3v 1 + 2v 2

14. A particle moves for 20 seconds with velocity 3 m/s and then velocity 4 m/s for another 20 seconds
and finally moves with velocity 5 m/s for next 20 seconds. What is the average velocity of the
particle
(A) 3 m/s (B) 4 m/s (C) 5 m/s (D) Zero

Space for Rough Work

2
Assignment - 02

1. A particle experiences a constant acceleration for 20 sec after starting from rest. If it travels a
distance S1 in the first 10 sec and a distance S2 in the next 10 sec, then
(A) S1 = S2 (B) S1 = S2 / 3 (C) S1 = S2 / 2 (D) S1 = S2 / 4

9
2. The initial velocity of the particle is 10 m / sec and its retardation is = 8  = 36 m . The distance
2
moved by the particle in 5th second of its motion is:
(A) 1 m (B) 19 m (C) 50 m (D) 75m

3. A motor car moving with a uniform speed of 20m / sec comes to stop on the application of brakes
after travelling a distance of 10m Its acceleration is:

(A) 20m / sec2 (B) −20m / sec2 (C) −40 m / sec2 (D) +2m / sec2

4. The velocity of a body moving with a uniform acceleration of 2 m. / sec2 is 10 m / sec . Its
velocity after an interval of 4 sec is:
(A) 12 m / sec (B) 14 m / sec (C) 16 m / sec (D) 18 m / sec

5. The initial velocity of a body moving along a straight line is 7 m / s . It has a uniform acceleration
of 4m / s2 . The distance covered by the body in the 5th second of its motion is:
(A) 25 m (B) 35 m (C) 50 m (D) 85 m

6. A particle moving with a uniform acceleration travels 24 m and 64 m in the first two consecutive
intervals of 4 sec each. Its initial velocity is:
(A) 1 m/sec (B) 10m / sec (C) 5 m/sec (D) 2 m/sec

7. A car moving with a velocity of 10 m/s can be stopped by the application of a constant force F in
a distance of 20 m. If the velocity of the car is 30 m/s, it can be stopped by this force in
20
(A) m (B) 20 m (C) 60 m (D) 180 m
3

8. A car moving with a speed of 40 km/h can be stopped by applying brakes after atleast 2 m. If the
same car is moving with a speed of 80 km/h, what is the minimum stopping distance
(A) 8 m (B) 2 m (C) 4 m (D) 6 m

Space for Rough Work

1
9. An elevator car, whose floor to ceiling distance is equal to 2.7 m, starts ascending with constant
acceleration of 1.2 ms–2. 2 sec after the start, a bolt begins fallings from the ceiling of the car. The
free fall time of the bolt is:
(A) 0.54 s (B) 6s (C) 0.7 s (D) 1 s

10. Two trains travelling on the same track are approaching each other with equal speeds of 40 m/s.
The drivers of the trains begin to decelerate simultaneously when they are just 2.0 km apart.
Assuming the decelerations to be uniform and equal, the value of the deceleration to barely avoid
collision should be:
(A) 11.8 m / s2 (B) 11.0 m / s2 (C) 2.1 m / s2 (D) 0.8 m / s2

11. A body moves from rest with a constant acceleration of 5m / s2 . Its instantaneous speed (in m / s)
at the end of 10 sec is:
(A) 50 (B) 5 (C) 2 (D) 0.5

12. A body starts from rest. What is the ratio of the distance travelled by the body during the 4th and
3rd second
7 5 7 3
(A) (B) (C) (D)
5 7 3 7

13. If a car at rest accelerates uniformly to a speed of 144 km/h in 20 s. Then it covers a distance of
(A) 20 m (B) 400 m (C) 1440 m (D) 2880 m

14. If a train travelling at 72 kmph is to be brought to rest in a distance of 200 metres, then its
retardation should be:
(A) 20 ms−2 (B) 10 ms−2 (C) 2 ms−2 (D) 1 ms−2

15. Two cars A and B at rest at same point initially. If A starts with uniform velocity of 40 m/sec and
B starts in the same direction with constant acceleration of 4m / s2 , then B will catch A after how
much time
(A) 10 sec (B) 20 sec (C) 30 sec (D) 35 sec

16. A particle travels 10m in first 5 sec and 10m in next 3 sec. Assuming constant acceleration what
is the distance travelled in next 2 sec
(A) 8.3 m (B) 9.3 m (C) 10.3 m (D) None of above

Space for Rough Work

2
17. The position of a particle moving along the x-axis at certain times is given below :
t (s) 0 1 2 3
x (m) –2 0 6 16
Which of the following describes the motion correctly
(A) Uniform, accelerated
(B) Uniform, decelerated
(C) Non-uniform, accelerated
(D) There is not enough data for generalization

18. A body A starts from rest with an acceleration a 1 . After 2 seconds, another body B starts from
rest with an acceleration a2 . If they travel equal distances in the 5th second, after the start of A,
then the ratio a1 : a2 is equal to
(A) 5 : 9 (B) 5 : 7 (C) 9 : 5 (D) 9 : 7

19. Speed of two identical cars are u and 4u at a specific instant. The ratio of the respective distances
in which the two cars are stopped from that instant is:
(A) 1 : 1 (B) 1 : 4 (C) 1 : 8 (D) 1 : 16

20. A man is 45 m behind the bus when the bus start accelerating from rest with acceleration 2.5
m/s2. With what minimum velocity should the man start running to catch the bus ?
(A) 12 m/s (B) 14 m/s (C) 15 m/s (D) 16 m/s

Space for Rough Work

3
Assignment – 03

1. A stone falls from a balloon that is descending at a uniform rate of 12 m / s . The displacement of
the stone from the point of release after 10 sec is:
(A) 490 m (B) 510 m (C) 610 m (D) 725 m

2. A body A is projected upwards with a velocity of 98m / s . The second body B is projected upwards
with the same initial velocity but after 4 sec. Both the bodies will meet after
(A) 6 sec (B) 8 sec (C) 10 sec (D) 12 sec

3. Two bodies of different masses ma and mb are dropped from two different heights a and b . The
ratio of the time taken by the two to cover these distances are
(A) a : b (B) b : a (C) a: b (D) a2 : b2

4. A body falls freely from rest. It covers as much distance in the last second of its motion as covered
in the first three seconds. The body has fallen for a time of
(A) 3 s (B) 5 s (C) 7 s (D) 9s

5. A body is released from a great height and falls freely towards the earth. Another body is released
from the same height exactly one second later. The separation between the two bodies, two
seconds after the release of the second body is:
(A) 4.9m (B) 9.8m (C) 19.6m (D) 24.5m

6. An object is projected upwards with a velocity of 100 m / s . It will strike the ground after
(approximately)
(A) 10 sec (B) 20 sec (C) 15 sec (D) 5 sec

7. A stone dropped from the top of the tower touches the ground in 4 sec. The height of the tower
is about
 1   1 
(A) = 7  5 + 4  (5)2  − 7  4 + 4  (4)2  = 25 m
 2   2 
(B) 40m
(C) 20 m
(D) 160 m

Space for Rough Work

1
8. A body is slipping from an inclined plane of height h and length l . If the angle of inclination is  ,
the time taken by the body to come from the top to the bottom of this inclined plane is:
2h 2l 1 2h 2h
(A) (B) (C) (D) sin 
g g sin  g g

9. A body falls from rest, its velocity at the end of first second is (g = 32ft / sec)
(A) 16 ft / sec (B) 32 ft / sec (C) 64 ft / sec (D) 24 ft / sec

10. A stone thrown upward with a speed u from the top of the tower reaches the ground with a
velocity 3u . The height of the tower is:
(A) 3u2 / g (B) 4u2 / g (C) 6u2 / g (D) 9u2 / g

11. A stone is dropped from a certain height which can reach the ground in 5 second. If the stone is
stopped after 3 second of its fall and then allowed to fall again, then the time taken by the stone
to reach the ground for the remaining distance is:
(A) 2 sec (B) 3 sec (C) 4 sec (D) None of these

12. A man in a balloon rising vertically with an acceleration of 4.9m / sec2 releases a ball 2 sec after
the balloon is let go from the ground. The greatest height above the ground reached by the ball is
(g = 9.8m / sec2 )
(A) 14.7 m (B) 19.6 m (C) 9.8 m (D) 24.5 m

13. A particle is dropped under gravity from rest from a height h(g = 9.8m / sec2 ) and it travels a
distance 9h / 25 in the last second, the height h is:
(A) 100 m (B) 122.5 m (C) 145 m (D) 167.5 m

14. A balloon is at a height of 81 m and is ascending upwards with a velocity of 12 m/s. A body of 2kg

weight is dropped from it. If g = 10m / s2 , the body will reach the surface of the earth in
(A) 1.5 s (B) 4.025 s (C) 5.4 s (D) 6.75 s

15. An aeroplane is moving with a velocity u . It drops a packet from a height h . The time t taken by
the packet in reaching the ground will be:
 2g   2u   h   2h 
(A)   (B)   (C)   (D)  
 h   g   2g   g 

Space for Rough Work

2
16. A ball is thrown vertically upwards from the top of a tower at 4.9 ms−1 . It strikes the pond near
the base of the tower after 3 seconds. The height of the tower is:
(A) 73.5 m (B) 44.1 m (C) 29.4 m (D) None of these

17. A rocket is fired upward from the earth's surface such that it creates an acceleration of
19.6 m/sec2. If after 5 sec its engine is switched off, the maximum height of the rocket from earth's
surface would be:
(A) 245 m (B) 490 m (C) 980 m (D) 735 m

18. A body starts to fall freely under gravity. The distances covered by it in first, second and third
second are in ratio
(A) 1 : 3 : 5 (B) 1 : 2 : 3 (C) 1 : 4 : 9 (D) 1 : 5 : 6

19. A body falling for 2 seconds covers a distance S equal to that covered in next second. Taking
g = 10m / s2 , S =
(A) 30 m (B) 10 m (C) 60 m (D) 20 m

20. With what velocity a ball be projected vertically so that the distance covered by it in 5th second is
twice the distance it covers in its 6th second (g = 10m / s2 )
(A) 58.8 m/s (B) 49 m/s (C) 65 m/s (D) 19.6 m/s

Space for Rough Work

3
Assignment – 04

1. A ball is dropped downwards. After 1 second another ball is dropped downwards from the same
point. What is the distance between them after 3 seconds
(A) 25 m (B) 20 m (C) 50 m (D) 9.8 m

2. A stone is thrown with an initial speed of 4.9 m/s from a bridge in vertically upward direction. It falls
down in water after 2 sec. The height of the bridge is:
(A) 4.9 m (B) 9.8 m (C) 19.8 m (D) 24.7 m

3. A stone is shot straight upward with a speed of 20 m/sec from a tower 200 m high. The speed
with which it strikes the ground is approximately
(A) 60 m/sec (B) 65 m/sec (C) 70 m/sec (D) 75 m/sec

4. A body freely falling from the rest has a velocity ‘v’ after it falls through a height ‘h’. The distance
it has to fall down for its velocity to become double, is:
(A) 2h (B) 4h (C) 6h (D) 8h

5. A body projected vertically upwards with a velocity u returns to the starting point in 4 seconds. If
g = 10 m/sec2, the value of u is:
(A) 5 m/sec (B) 10 m/sec (C) 15 m/sec (D) 20 m/sec

6. Time taken by an object falling from rest to cover the height of h1 and h2 is respectively t 1 and
t2 then the ratio of t 1 to t2 is:

(A) h1 : h2 (B) h1 : h2 (C) h1 : 2h2 (D) 2h1 : h2

7. A particle is thrown vertically upwards. If its velocity at half of the maximum height is 10 m/s, then
maximum height attained by it is (Take g = 10 m/s2)
(A) 8 m (B) 10 m (C) 12 m (D) 16 m

8. A body, thrown upwards with some velocity, reaches the maximum height of 20m. Another body
with double the mass thrown up, with double initial velocity will reach a maximum height of
(A) 200 m (B) 16 m (C) 80 m (D) 40 m

9. A balloon starts rising from the ground with an acceleration of 1.25 m/s2 after 8s, a stone is
released from the balloon. The stone will ( g = 10 m/s2)
(A) Reach the ground in 4 second (B) Begin to move down after being released
(C) Have a displacement of 50 m (D) Cover a distance of 40 m in reaching the ground

Space for Rough Work

1
10. A particle when thrown, moves such that it passes from same height at 2 and 10s, the height is:
(A) g (B) 2g (C) 5g (D) 10g

11. From the top of a tower, a particle is thrown vertically downwards with a velocity of 10 m/s. The
ratio of the distances, covered by it in the 3rd and 2nd seconds of the motion is (Take g = 10m / s2 )
(A) 5 : 7 (B) 7 : 5 (C) 3 : 6 (D) 6 : 3

12. Two balls A and B of same masses are thrown from the top of the building. A, thrown upward with
velocity V and B, thrown downward with velocity V, then
(A) Velocity of A is more than B at the ground
(B) Velocity of B is more than A at the ground
(C) Both A & B strike the ground with same velocity
(D) None of these

13. A cricket ball is thrown up with a speed of 19.6 ms–1. The maximum height it can reach is:
(A) 9.8 m (B) 19.6 m (C) 29.4 m (D) 39.2 m

14. A body falling from a high Minaret travels 40 meters in the last 2 seconds of its fall to ground.
Height of Minaret in meters is (take g = 10m / s2 )
(A) 60 (B) 45 (C) 80 (D) 50

15. A body falls from a height h = 200m (at New Delhi). The ratio of distance travelled in each 2 sec
during t = 0 to t = 6 second of the journey is:
(A) 1 : 4 : 9 (B) 1 : 2 : 4 (C) 1 : 3 : 5 (D) 1 : 2 : 3

16. Two balls are dropped from heights h and 2h respectively from the earth surface. The ratio of
time of these balls to reach the earth is:
(A) 1 : 2 (B) 2 :1 (C) 2 : 1 (D) 1 : 4

17. A body is thrown vertically upwards with velocity u. The distance travelled by it in the fifth and
the sixth seconds are equal. The velocity u is given by (g = 9.8 m/s2)
(A) 24.5 m/s (B) 49.0 m/s (C) 73.5 m/s (D) 98.0 m/s

18. A body, thrown upwards with some velocity reaches the maximum height of 50 m. Another body
with double the mass thrown up with double the initial velocity will reach a maximum height of
(A) 100 m (B) 200 m (C) 300 m (D) 400 m

19. A parachutist after bailing out falls 50 m without friction. When parachute opens, it decelerates
at 2 m/s2. He reaches the ground with a speed of 3 m/s. At what height, did he bail out ?
(A) 293 m (B) 111 m (C) 91 m (D) 182 m

Space for Rough Work

2
20. From the top of a tower two stones, whose masses are in the ratio 1 : 2 are thrown one straight
up with an initial speed u and the second straight down with the same speed u. Then, neglecting
air resistance
(A) The heavier stone hits the ground with a higher speed
(B) The lighter stone hits the ground with a higher speed
(C) Both the stones will have the same speed when they hit the ground.
(D) The speed can't be determined with the given data.

Space for Rough Work

3
KINEMATICS 2D

ASSIGNMENT-1

1. The coordinates of a moving particle at any time 't' are given by x = t3 and y = t3. The speed of
the particle at time 't' is given by:
(A) t2  +  (D) 3t2  + 
2 2 2 2
(B) 2 + 2 (C) 3t 2 + 2

2. If the position of the particle is x = 3 t2 − 5 t & y = 6 t − 2, then acceleration of the particle will be:
(A) 6 (B) − 5 (C) 2 (D) None of these

3. A body has an initial velocity of 3 ms–1 and has a constant acceleration of 1 ms–2 normal to the
direction of the initial velocity. Then its velocity, 4 second after the start is:
(A) 7 ms–1 along the direction of initial velocity
(B) 7 ms–1 along the normal to the direction of the initial velocity
(C) 7 ms–1 mid-way between the two directions
4
(D) 5 ms–1 at an angle of tan–1 3 with the direction of the initial velocity

4. The position vector of a particle is given as r = (t2 – 4t + 6) î + (t2) ˆj . The time after which the
velocity vector and acceleration vector becomes perpendicular to each other is equal to:
(A) 1sec (B) 2 sec (C) 1.5 sec (D) not possible

5. A point mass is projected, making an acute angle with the horizontal. If angle between velocity and
acceleration g is  at any time t during the motion, then  is given by:
(A) 0° <  < 90° (B)  = 90° (C)  < 90° (D) 0° <  < 180°

6. A projectile is thrown with a speed v at an angle  with the upward vertical. Its average velocity
between the instants at which it crosses half the maximum height is:
(A) v sin , horizontal and in the plane of projection
(B) v cos , horizontal and in the plane of projection
(C) 2v sin , horizontal and perpendicular to the plane of projection
(D) 2v cos , vertical and in the plane of projection

7. A ball is hit by a batsman at an angle of 37° as shown in figure. The man standing at P should run
at what minimum velocity so that he catches the ball before it strikes the ground. Assume that
height of man is negligible in comparison to maximum height of projectile.

(A) 3 ms–1 (B) 5 ms–1 (C) 9 ms–1 (D) 12 ms–1

8. A particle is projected from a point O with a velocity u in a direction making an angle  upward with
the horizontal. At P it is moving at right angles to its initial direction. Its velocity at P is:
(A) u tan  (B) u cot  (C) u cosec  (D) u sec 

9. A projectile is thrown with velocity v making an angle  with the horizontal. It just crosses the top
of two poles, each of height h, after 1 second and 3 second respectively. The time of flight of the
projectile is:
(A) 1s (B) 3s (C) 4s (D) 7.8s

1
KINEMATICS 2D
10. A cricketer hits a ball with a velocity 25 m/s at 60° above the horizontal. How far above the ground it
passes over a fielder 50 from the bat: (assume the ball is struck very close to the ground)
(g = 9.8 m/s2)
(A) 8.2 m (B) 9.0 m (C) 11.6 m (D) 12.7 m

11. A particle is projected from the ground with velocity u at angle  with horizontal. The horizontal
range, maximum height and time of flight are R, H and T respectively. They are given by,
u2 sin2 u2 sin2  2u sin 
R= ,H = and T =
g 2g g
Now keeping u as fixed,  is varied from 30° to 60°. Then,
(A) R will first increase then decrease, H will increase and T will decrease
(B) R will first increase then decrease while H and T both will increase
(C) R will decrease while H and T will increase
(D) R will increase while H and T will increase

12. A ball is thrown from a point on ground at some angle of projection. At the same time a bird starts
from a point directly above this point of projection at a height h horizontally with speed u. Given
that in its flight ball just touches the bird at one point. Find the distance on ground where ball
strikes.
h 2h 2h h
(A) 2u (B) u (C) 2u (D) u
g g g g

13. A body is projected horizontally from the top of a tower with initial velocity 18 ms –1. It hits the
ground at angle 45°. What is the vertical component of velocity when it strikes the ground?
(A) 18 3 ms–1 (B) 18 ms–1 (C) 9 2 ms–1 (D) 9 ms–1

14. An aeroplane is moving with horizontal velocity u at height h. The velocity of a packet dropped from
it on the earth’s surface will be: (g is acceleration due to gravity)
(A) u2 + 2gh (B) 2gh (C) 2gh (D) u2 − 2gh

15. A stone projected at angle ‘’ with horizontal from the roof of a tall building falls on the ground
after three second. Two second after the projection it was again at the level of projection. Then the
height of the building is:

(A) 5 m (B) 25 m (C) 20 m (D) 15 m

16. A particle moves on the xy-plane such that its position vector is given by r = 2t2ˆi − t3ˆj . The equation
of trajectory of the particle is given by:
4/3
 3x 
(A) 3x2 + 16y = 0 (B)   + 4y = 0
 2 
3/2
x
(C)   +y=0 (D) None of these
2
2
KINEMATICS 2D
17. On an inclined plane of inclination 30°, a ball is thrown at an angle of 60° with the horizontal from
the foot of the incline with a velocity of 10 3 ms–1. If g = 10 ms–2, then the time in which ball will
hit the inclined plane is:
(A) 1 sec. (B) 6 sec. (C) 2 sec. (D) 4 sec.

18. Two forces, F1 = (–6i – 4j) N and F2 = (–3i + 7j) N, act on a particle of mass 2.00 kg that is initially
at rest at coordinates (–2.00m, +4.00 m). (a) What are the components of the particle's velocity at
t = 10.0s? (b) In what direction is the particle moving at t = 10.0s? (c) What displacement does the
particle undergo during the first 10.0s? (d) what are the coordinates of the particle at t = 10.0 s?

19. A particle is projected in the X-Y plane with y-axis along vertical. Two sec after projection the
velocity of the particle makes an angle 45° with the X - axis. Four sec after projection, it moves
horizontally. Find the velocity of projection.

20. If at an instant the velocity of a projectile be 60 m/s and its inclination to the horizontal be 30°, at
what time interval (in sec) after that instant will the particle be moving at right angles to its former
direction. (g = 10 m/s2)

ANSWER KEY

(1) (D) (2) (A) (3) (D) (4) (A) (5) (D) (6) (A) (7) (B)
(8) (B) (9) (C) (10) (A) (11) (B) (12) (C) (13) (B) (14) (A)
(15) (D) (16) (C) (17) (C)
(18) (a) 45 m/s along negative x-axis and 15 m/s along positive y-axis
 1
(b)  = tan–1  − 3  from positive x-axis
 

(c) (-225iˆ + 75j)m


ˆ

(d) (-227iˆ + 79j)m


ˆ

(19) 20 5 m/s (20) 12

3
NLM Assignment

1. Statement-1: Speed of a frame 'A' is constant w.r.t. an inertial frame 'B'. Frame 'A' will be an inertial
frame.
Statement-2: If acceleration of a frame is zero w.r.t. to an inertial frame then that frame is an
inertial frame.
(A) Statement-1 is true, statement-2 is true and statement-2 is correct explanation for statement-1.
(B) Statement-1 is true, statement-2 is true and statement-2 is NOT the correct explanation for
statement-1.
(C) Statement-1 is true, statement-2 is false.
(D) Statement-1 is false, statement-2 is true.
2. Statement-1: For a body resting on level ground, normal reaction and weight of body form Newton's
3rd law’s action - reaction pair.
Statement-2: Action-reaction pair of Newton's 3rd law are equal and opposite and are of same
nature.
(A) Statement-1 is true, statement-2 is true and statement-2 is correct explanation for statement-1.
(B) Statement-1 is true, statement-2 is true and statement-2 is NOT the correct explanation for
statement-1.
(C) Statement-1 is true, statement-2 is false.
(D) Statement-1 is false, statement-2 is true.
3. In the figure shown, a balloon is pressed against a wall. It is in equilibrium and maximum compresed
state.
F1 = force of balloon on hand of man; F2 = force of balloon on wall; F3 = friction; F4 = weight of
balloon. Choose the correct statement.
(A) F1 and F2 are action reaction pairs.
(B) F3 and F4 are action reaction pairs.
(C) F2 and F3 are action reaction pairs.
(D) F4 and gravitational force exerted by balloon on earth are action
reaction pairs.
4. When a horse pulls a cart, the force needed to move the horse in forward direction is the force
exerted by
(A) the cart on the horse (B) the ground on the horse
(C) the ground on the cart (D) the horse on the ground

5*. Consider a cart being pulled by a horse with constant velocity. The horse exerts force FC/h on the
cart. The subscript indicates the force on the cart due to horse. The first subscript denotes the
body on which force acts and second due to which it acts.

Choose the correct statement(s):


(A) FC / g , NC/ g , Nh/ g are external forces on a system consisting of horse and cart.

(B) Fh / g + FC / g = 0 .

(C) NC/ g and FC /E are action reaction pairs.

(D) FC /h and Fh/C are action reaction pairs.

1
Draw the FBD for the following individual parts of the systems:
(Pulley are massless and frictionless)

6. 7. 8. 9.

10. 11. 12.

13. 14.

15.

2
1. In order to test the strength of a rope, one end of rope is tied to a large tree and the other end is
hitched to a team of 2 horses. The horse pull as hard as they can, but cannot break the rope. If the
rope is untied from the tree and attached to another team of 2 horses with equal strength, and the
two teams pull in opposite directions, the tension in the rope will:

(A) decrease by a factor of 2 (B) remain the same


(C) increase by a factor of 2 (D) increase by a factor of 4

2. A smooth cylinder of mass M and radius R is resting on two corner edges A and B as shown in figure.
The normal reaction at the edges A and B are NA and NB, then:

2 3N A
(A) NA = 2NB (B) NB =
5
NB
(C) N A = (D) NB = 3NA
2

3. Two men pull a rope from which a mass is hanging as shown. Both apply a force of 100 N each.
What force should each apply to make the rope horizontal

30° 30°

(A) 200 N (B) 400 N (C) 600 N (D) rope cannot become horizontal

4*. Select the correct statement(s) :

(A) The vertical string has a tension of 10 N.


(B) The horizontal string has a tension of 10 N.
(C) The inclined string has a tension more than 10 N.
(D) The inclined string has a tension less than 10 N.

3
5. A block of mass 'm' is placed on ground and an additional force F = mg is applied on the block as
as shown in figure. Calculate contact force between ground and block.

6. A sphere of mass 'm', radius 'R' is placed between two vertical wall having separations 'd' which is
slightly greater than '2R':

(i) Calculate force exerted by walls on the sphere.


(ii) Calculate force exerted by ground surface on the sphere.

7. If mass M is 2 kg then what is the tension in string AC? (in N)

8. A 27-kg chandelier hangs from a ceiling on a 4.0 m long vertical wire. What horizontal force should
be applied to the chandelier to displace its position 2.4 m to one side from equilibrium position?

9. A block of mass 0.3 kg is suspended from the ceiling by a light string. A second block of mass 0.2
kg is suspended from the first block through another string. Find the tensions in the two strings.
Take g = 10 m/s2.

10. A cylinder of weight w is resting on a fixed V-groove as shown in figure.

4
(a) Draw its free body diagram.
(b) Calculate normal reactions between the cylinder and two inclined walls.

5
1. What is the magnitude of force F if the tension in the string connecting blocks B and C is 60 N?
(All blocks are identical)
T 60 N
A B C F

(A) 60 N (B) 120 N (C) 180 N (D) None of these

2. The acceleration of m is 5 m/s2 downwards. What is the value of m?

2kg
m
(A) 4kg (B) 5kg (C) 6kg (D) 7 kg

3. Three masses of 1 kg, 6 kg and 3 kg are connected to each other with threads and are placed on table
as shown in figure. What is the acceleration with which the system is moving? Take g = 10m s–2.

(A) Zero (B) 1 m s–2 (C) 2 m s–2 (D) 3 m s–2

4. Three blocks of mass 1 kg, 4 kg and 2 kg are placed on a smooth horizontal plane as shown in the
figure. The contact force between 1 kg block and 4 kg block is:

(A) 100 N (B) 120 N (C) 50 N (D) 110 N


5. A block of mass m is placed in contact with one end of a smooth tube of mass M (see figure). A
horizontal force F acts on the tube in each case (i) and (ii). Then mark incorrect option :
M

F m (i)

F m (ii)

F F
(A) am = 0 and aM = in (i) (B) am = aM = in (i)
M M+m
F mF
(C) am = aM = in (ii) (D) Force on m is in (ii)
M+m M+m

6. A 5 kg block has a rope of mass 2 kg attached to its underside and a 3 kg block is suspended from
the other end of the rope. The whole system is accelerated upward by 2 m/s 2 by an external force
F0. What is F0?

6
F0
5 kg
2 kg
3 kg
(A) 60 N (B) 120 N (C) 40 N (D) 200 N

7. Figure shows a block of mass m placed on a bracket of mass M. Bracket and block system is moved
downward with constant velocity on an incline. What is magnitude of total force of bracket on block?
v = constant

m
M

(A) zero (B) mg sin  (C) mg cos  (D) mg

8*. Two blocks A and B of equal mass m are connected through a massless string and arranged as
shown in figure. The wedge is fixed on horizontal surface. Friction is absent everywhere. When the
system is released from rest :

Fixed
30° B

mg mg
(A) tension in string is (B) tension in string is
2 4
g 3g
(C) acceleration of A is (D) acceleration of A is
2 4
9*. A monkey of mass 40 kg climbs on a rope which can stand maximum tension of 600 N. In which of
the following case will the rope does not break ?
(A) The monkey climbs up with an acceleration of 4 m/s2
(B) The monkey climbs down with an acceleration of 4 m/s2
(C) The monkey climbs up with uniform speed of 5 m/s
(D) The monkey falls down the rope freely under gravity

10. In the diagram shown in figure, all pulleys are smooth and massless and strings are light. Match the
following:
F = 80 N

1 kg
3 kg
4 kg
2 kg
7
Column -I Column-II
(A) 1 kg block (P) will remain stationary
(B) 2 kg block (Q) will move down
(C) 3 kg block (R) will move up
(D) 4 kg block (S) 5 m/s2
(T) 10 m/s2

8
1. A 90 kg man stands in an elevator that has a downward acceleration of 1.5 m/s2. The force exerted
by him on the floor is about:
(A) zero (B) 90 N (C) 765 N (D) 880 N

2. A man is standing in a lift which goes up and comes down with the same constant acceleration. If
the ratio of the apparent weights in the two cases is 2 : 1, then the acceleration of the lift is
(A) 3.33 ms–2 (B) 2.50 ms–2 (C) 2.00 ms–2 (D) 1.67 ms–2

3. Two masses of 10 kg and 20 kg respectively are connected by a massless spring as shown in figure.
A force of 200 N acts on the 20 kg mass at the instant when the 10 kg mass has an acceleration of
12 ms–2 towards right, the acceleration of the 20 kg mass is :

(A) 2 ms–2 (B) 4ms–2 (C) 10ms–2 (D) 20ms–2

4. n the given figure, what is the reading of the spring balance ?

1kg 1kg

(A) 10 N (B) 20 N (C) 5 N (D) zero

5. Three springs with force constants k, 2k, 4k, respectively are connected in series. The effective
force constant of the spring will be
4k 2k
(A) 7k (B) 2k (C) (D)
7 5
6. A light particle of mass m is connected to another very heavy particle by means of a light
inextensible string that passes over a smooth pulley. The reading of the spring balance is
approximately -

(A) 4 mg (B) >> mg (C) Zero (D) 2 mg


7*. The apparent weight of a person in a lift moving vertically depend on
(A) the actual mass of the person (B) acceleration due to gravity
(C) acceleration of the lift (D) velocity of the lift
8. If the reading of spring balance as shown in figure is 2F newton
then value of F is.
(Assume that mass M is in equilibrium)

9. Figure shows a man of mass 50 kg standing on a light weighing


machine kept in a box of mass 30 kg. The box is hanging from a pulley fixed to the ceiling through

9
a light rope, the other end of which is held by the man himself. If the man manages to keep the
box at rest, the weight shown by the machine is ______kg.

10. Four blocks of masses 1 kg, 2kg, 4 kg, and 8kg are connected with three ideal springs. The whole
system is released from rest on a frictionless horizontal surface. After some time, the acceleration
of 1 kg and 4 kg are found to be zero while the middle spring is found to be compressed by 0.1 m.
At the same instant of time, the ratio of acceleration of 2 kg and 8 kg block is :
1kg 2kg 4kg 8kg
100N/m 200N/m 400N/m

10
1. The velocity of A at this instant is 4 m/s rightwards. Then the velocity of block B at this instant is:

B
(A) 4 m/s downwards (B) 2 m/s upwards
(C) 2 m/s downwards (D) 1 m/s upwards
2. Choose the relationship which governs the velocities of the four cylinders.

(A) vA + vB – 4vC + vD = 0 (B) 4vA – 8vB – 4vC + vD = 0


(C) 4vA – 4vB – 4vC + vB = 0 (D) vA + vB – vC + vD = 0

3. A boat sailing in a pond is being pulled with the help of a rope. At the moment, when the rope
makes an angle  with the water surface and the rope is pulled with velocity v, the velocity with
which the boat approaches the bank is

v

v
(A) v (B) (C) v cos  (D) v(1 – cos )
cos 
4. An elastic spring of natural length 10 cm and force constant 50 N/m2 is cut into two parts of lengths
1 and 2. The force constants of these parts are respectively: (Given: 1 : 2 = 2 : 3)

250
(A) 125 N/m2 and N/m2 (B) 20 N/m2 and 30 N/m2
3
250
(C) N/m2 and 125 N/m2 (D) 30 N/m2 and 20 N/m2
3

5. The system shown is in equilibrium. When string A is cut, the initial acceleration of
upper and lower blocks will be _________ & ________.
[ g = 10 m/s2 ]
[Hint: spring force does not change instantly]

6. Find extension of spring in equilibrium in Column-I and match them with Column-II.
[Assume ideal spring, light string and frictionless, pulley]
Column-I Column-II

11
k
mg
(A) (P)
k

4m g
(B) (Q)
3k

2m g
(C) (R)
k

mg
(D) (S)
4k

m
7. In the diagram strings, springs and the pulley are light and ideal. The system is in equilibrium with
the strings taut (T > 0), match the column. Masses are equal.

Column I Column II
(A) Just after spring W breaks (P) aA = 0
(B) Just after spring X breaks (Q) aB = 0
(C) Just after string Y breaks (R) aC = 0
(D) Just after spring Z breaks (S) aB = aC

Answer Q.8, Q.9 and Q.10 by appropriately matching the information given in the three columns of
the following table.
12
A block of mass 4 kg is placed on the inclined surface of wedge of mass 12 kg. A horizontal force
“F” is applied on the surface of wedge as shown in figure (Direction of force may be along the
positive and negative x-axis). All of the contact surfaces are smooth. Magnitude of force applied on
the wedge “F”, Normal contact force between wedge and block “N” and acceleration of block “a”
are given in Column-1, 2 and 3 respectively. Consider horizontal axis is along the x-axis and vertical
axis is along the y-axis.

Coloumn-1 Column-2 Column-3


Force applied (F) Normal contact force Acceleration of block
In Newton in Newton in m/s2
(I) 19.2 (i) 0 (P) 6
(II) 32.4 (ii) 32 (Q) 7.5
(III) 120 (iii) 40 (R) 8
(IV) 160 (IV) 50 (S) 10
8. In which of the following case acceleration of block is in horizontal direction?
(A) (III) (iii) (Q) (B) (IV) (i) (S)
(C) (III) (iv) (Q) (D) (I) (ii) (P)

9. In which of the following case the acceleration of wedge is equal to zero ?


(A) (II) (i) (S) (B) (I) (ii) (P)
(C) (I) (iii) (R) (D) (III) (ii) (P)
10. In which of the following case the acceleration of block is in vertically downward direction?
(A) (IV) (iii) (S) (B) (I) (ii) (R)
(C) (III) (iv) (Q) (D) (IV) (i) (S)

13
1. In the figure shown, the blocks A and B are connected with an inextensible string. If the block B is
pulled with a velocity of 5 m/s then it is observed that block A moves with 10 m/s. Find the angle
 (in degree) shown in the diagram at this instant.
5 m/s
B


A 10 m/s

(A) 60° (B) 30° (C) 45° (D) 37°

2. A block is dragged on smooth plane with the help of a rope which moves with velocity v. The
horizontal velocity of the block at the shown instant is:
V

//////////////////

////////////////////////////

v v
(A) v (B) (C) v sin  (D)
sin  cos 

3. In the arrangement shown in fig, the ends P and Q of an unstretchable string move downwards with
uniform speed U. Pulleys A and B are fixed. Mass M moves upwards with a speed.

(A) 2 U cos  (B) U cos  (C) 2 U/cos  (D) U/cos 


4. Find the acceleration of block of mass m. Assume pulleys are massless and frictionless.
g 2g
(A) (B) m
3 3
g
(C) (D) None
2
2m
5. In the arrangement shown, the pulleys and the strings are ideal. The
acceleration of block B is

m A

m B

(A) g/5 (B) g/2 (C) 2g/5 (D) 2g/3

14
6. Two trolley A and B are moving with accelerations a and 2a respectively in the same direction. To
an observer in trolley A, the magnitude of pseudo force acting on a block of mass m on the trolley
B is
(A) zero (B) ma
(C) 2 ma (D) None
7. A helicopter of mass M is carrying a box of mass m at the end of a
rope and is moving horizontally with constant acceleration 'a'. The acceleration due to
gravity is 'g'. Neglect air resistance. The rope is stretched out from the helicopter at a
constant angle  to the vertical. What is this angle?

(A) sin = a/g (B) cos = a/g (C) tan = a/g (D) sin = ma/(Mg)
8. What is the speed (in m/s) of block m1 of mass 2 kg (Figure) at the moment when
it has moved by 4.8 m starting from rest. Assume that the pulleys are weightless,
no friction, thread weightless and inextensible, and the mass of m 2 = 1 kg. m1
9. Both the blocks have equal mass m. String is massless and inextensible.
Acceleration of mass A is _______ m/s2. m2

A
B
10. An object of mass 2 kg moving with constant velocity 10iˆ m/s is seen in a frame moving with
constant velocity 10iˆ m/s. What will be the value of ‘pseudo force’ acting on object in this frame.

15

You might also like